Exam, TEAS Eng and language usage, TEAS Test Science Questions, teas test review 1, English study question teas, MATH SECTION, test, Reading2, Teas Reading, TEAS TEST (WHITE HOUSE SOLAR PANEL), TEAS Science Questions, TEAS READING: Assessment B, TEAS...

Pataasin ang iyong marka sa homework at exams ngayon gamit ang Quizwiz!

Which hormone decreases glucose level?

Insulin Beta cell

To determine the premium of a car an insurance company considers the following determine of the age of the car,the model of the car, and the mileage of the car. Which of the following is the dependent variable?

Insurance premium

Which of the following statements from the passage contains only facts?

When newly hatched, three of the birds can fit inside a nest the size of a large thimble.

The table shows a survey of auto mobile color preference. Which of the following statements about the survey responses is correct?

12 survey respondents preferred white or red

What is the missing coefficient needed to balance this double replacement reaction? 4 FeS + _____ O2 ------ 2 Fe2O3 + 4 SO2 a. 2 b. 5 c. 7 d. 14

7

In this sentence, which verb tense should be used in the blank? Brian ____ to Montreal only two times before. A. has driven B. has drove C. had drove D. drove

A

Convert each percent to a decimal. A. 73.7% B. 138%

A. 73.7/100= 0.737 B. 138/100= 1.38

Describe the role of actin in skeletal muscle contraction

- Makes up thin filaments - Contains myosin binding site that myosin head attaches to during contraction

Ligaments

Connect bone to bone

Stimulus question 3 of 3 which of the following is the writer's purpose for writing this passage?( three candidates showe up to a interview)

To analyze

Which of the following is the writer's purpose for writing this passage?

To analyze

Arteries

carry blood away from the heart

The most recognized globular protein is?

hemoglobin.

Which of the following is the % equivalent of 0.0016? a. 16% b. 160% c. 1.6% d. 0.16%

0.16%

Which of the following options represents 27% of 54?

14.58

Where is urine store?

Bladder

where are blood cells reproduce?

Bone marrow

A cup-like end of the nephron, surrounds the glomerulus,

Bowman's capsule

Where does the central nervous system processes information?

Brain

What does the medulla regulates?

Breathing, swallowing, and the beating of the heart.

What does coronary arteries do?

Bring oxygenated blood to the heart.

What is osteoblasts?

Build bone

fallopian tubes

Carries the egg from the ovary to the uterus where fertilization takes place

What is found in both Prokaryotes and Eukaryotes cells?

Cell membrane/Plasma membrane, DNA, and Ribosomes

Which of the following can be inferred from information in the passage? ( CHIMNEY SWIFTS)

Changes in home design and home heating occurred in the 1970s

NAD(+) +Ze(-)+H(+)⇀NADH Which of the following statements correctly describes the reaction above?

Combustion of NAD(+)

I before E except after C:

Conceive Deceive Perceive

Tendon

Connects muscle to bone

Which of the following types of membrane is associated with the skin

Cutaneous

which of the following chemicals is released by one type of immune cell to directly activate another immune cell?

Cytokines

. Running Log

Day 9

Function of Insulin:

Decreases blood sugar by promoting uptake of glucose by cells.

Which of the following portions of the neuron transmits neurotransmitters

Dentrite

Which layer of the skin is where sebaceous gland found?

Dermis

Which of the following hormones increases the heart rate in response to stress?

Epinephrine

Which of the following is a hormone that mediates the fight-or-flight response? A. Insulin B. Glucagon C. Epinephrine D. Endorphin

Epinephrine

The passage from Russian blue cats , what kind of passage is it ?

Explository; inform

In which of the following organs does the meiosis occur?

Fallopian tube

Breakdown of which of the following begins in the small intestine? a. fats b. fiber c. protein d. carbohydrates

Fats

Which of the followingm is likely suggestion the city manager's office will recieve from the public?

Fremont should increase available parking spaces downtown

An equal mass of which of the following states of matter could completely fill either a 100ml or 500ml container?

Gas

The process known as gastrulation results in the formation of which embryonic structures?

Germinal layers

A network of capillaries that contains large pores, making it highly permeable to certain ions like Na+ and K.

Glomerulus

What does the anterior pituitary secretes:

Growth hormone Prolactin hormone follicle-stimulating hormone luteinizing hormone thyroid-stimulating hormone adrenocorticotrpic hormone

which of the following sentences is an example of the third-person narrative voice?

He was scared when he got on the plane

Example of a simple sentence?

Judy watered the lawn Judy and Alan watered the lawn.

Which of the following is the protein secreted by hair follicles in the integument? a. collagen b. fibrin c. elastin d. Keratin

Keratin

The length of a rectangle is 3 units greater than its width, which expression correctly represents the rectangle?

2w+2(w+3)

Which cells produces testosterone?

Leydig cells in the presence of LH (Interstitial cell)

what produces bile?

Liver

Which of the following statements is the hypothesis? A. Mice finish mazes faster when the environment is quiet. B. Noise affects the concentration of mice. C. Mazes are an effective way to test mice. D. Loud music makes mice finish mazes faster.

Loud music makes mice finish mazes faster

The digested fats are absorbed into which system?

Lymphatic system

which of the following apples is the most economical to buy?

Macintosh

Buffers

Maintain acid base balance

Testosterone

Male sex hormone production is affected by diseases that harm the testes

Chimney Swifts Which of the following conclusions can the reader make from the passage?

Metal flues are inserted in masonry chimneys to prevent birds, such as chimney swifts, from nesting in them.

What organelle transport energy that from comes from food?

Mitochindria

What is organ system?

Multiple organs work together to perform a larger function.

Which of the following resources would help a reader gather information economic?

News article and textbooks

What kind of immunity response is Cytokine-Mediated Inflammation?

Non-Specific Immune Response.

Which of the following is the smallest region in space where two electrons with opposite spins are paired? a. Shell b. Orbital c. Nucleus d. Period

Orbital

what is peroxisomes?

Oxidation and detoxification

lung

Oxygenates blood (does not filter it)

Parathyroid gland hormone:

Parathyroid hormone

Which of the following hormones regulates blood calcium levels?

Parathyroid hormone

After crossing two tall pea plants, a scientist notes that approximately ¾ are tall, while approximately ¼ are short. The scientist is noting the offsprings'

Phenotypes

What is the master gland?

Pituitary

White house solar panels Which of the following is a logical conclusion based on this passage?

Presidents Obama and Carter have similar ideas on energy conservation.

Which of the following correctly shows the order in which the filtrate moves through the structures of the nephron?

Proximal tubule→loop of henle→distal tubule→collecting duct PT-LH-DT-CD

Hair follicles cooling the body:

Relax

People should read a __diagram before attempting to fix a broken appliance. Which of the following words correctly words completes the sentence above?

Schematic

What do amylase break down?

Starch

What does carbohydrates convert?

Starch and glycogen into sugar.

What is Valence electrons?

The electrons in the outermost shell

Of pathogens by phagocytes is a nonspecific cell?

The internal aspect of the innate immune system.

Hair protects the body from:

UV radiation by preventing sunlight from reaching the surface of the skin.

What does the body make when expose to sunlight?

Vitamin D

what is osteoclasts?

absorbs/breaks down

Examples of coordinating conjunctions:

and, but, yet, or, nor, for, and so

coordinating conjunctions includes:

and, but, yet, or, nor, for, and so

Testes hormone:

testosterone

Martin and dons: Job inquiries

thorn and thomas

Insulin

A hormone produced by the pancreas type 1 diabetes is a disease that is caused by the absence of insulin

what has sex hormone and their role is axillary & pubic air growth & libido:

Adrenal cortex

What is hypotonic?

Concentration of solutes is greater on the insider of the cell than outside of the cell and the cell will swell.

Nervous system

Includes the brain, spinal cord, & peripheral nerves controls the actions of other body systems

Function of Glucagon:

Increases blood sugar by stimulating breakdown of glycogen in the liver.

What is it called when gametes produce a zygote?

Fertilization

As a person cycling increases speed, the rate of calories burned, distance travelled, and energy expanded also increases. Which of the following is the independent variable?

Speed

Which of the following is the author's main purpose in the third paragraph?

To inform

Enzymes are ____ that work best at ____ pH or temperature. biological catalysts, optimum chemical substances, moderate organic catalysts, minimum inorganic catalysts, maximun

biological catalysts, optimum

what is the largest physical barrier?

the skin

solve for: 8/5x=6

(5/8)(8/5)x=6(5/8) 6/1*5/8=3/1*5/4=15/4

What is interphase?

-Cell prepares for division by replicating genetic /cytoplasmic material. -Longest phrase

What are the major layers of the skin?

-Outer epidermis -Inner dermis

solve the following problem: 4+3X(9-6)

13 how? 9-6=3 3X3=9 9+4=13

two hunderand students at a local high school answered a poll asking their favorite sport. The results are displayed in the graph above.how many students choose football or swimming as their favorite sport compared to those who choose soccer?

19

Which of the following is the best decimal approximation for the positive square root of 7?

2.6

A temperature gauge reads 95͑ F. Which of the following is the correct conversion to degrees celsius? (note: C=(5/9)(F-32))

35 C

What is a covalent bond?

A bond created by the sharing of electrons

What is double displacement reaction?

A chemical reaction in which two elements or molecular ions switch places.

Which of the following is the best source to consult for information about the famous jazz saxophonist charlie "yardbird" parker's childhood in Kansas City in the 1940s?

A published magazine interview with charlie parker

What is a controlled variable?

All of the variables held constant between the experiment and control groups.

Which of the following compound sentences is punctuated correctly?

Aziz was astonished; he had never seen horses in the wild before

Which of the following best summaries the passage?

Chimney swifts rely entirely on masonry chimneys to survive during their migrations.

What is an example of condensation?

Dew drops from a leaf

/each man in our club__to sing all the time/ - Which of the following options correctly completes the sentence above?

Has

Which part of the brain is responsible for memory?

Hippocampus

A person who is sick may have?

Increase in WBCs

Which of the following are lymphatic capillaries that absorb fats? a. lacteals b. nodules c. bronchioles d. axons

Lacteals

What does bile do?

Neutralizes stomach acid

What is intramolecular forces?

The bonds that hold a molecule together.

Which of the following best summarize the passage?

The diminishing number of chimney swifts can be reversed by simple conservation efforts.

colon is use for explanations or to give a quote:

The man started with an idea: "We are able to do more than we imagine."

Which of the following is a logical conclusion based on this passage?

The white house was the first building in America to have solar panels

Which of the following best rephrases the topic of the passage?

The wide variety of hummingbirds and their attributes

eccrine glands

These glands produce sweat.

Which of the following sentences best summarizes the passage?

They are marvels of speed, agility, and beauty

Memorandum Which of the following sequences reflect the correct sequence found in the memo?

This winter will be extraordinarily severe. Employees who need equipment to work from home may request it. Employees will be notified by email if the building closes

Which of the following best describes the author's purpose in the first sentence of the passage?

To connect the passage to the reader's frame of reference.

spleen function

To remove old fragmented red blood cells and pathogens from the blood

what is the function DNA:

To synthesize RNA and protein.

What is the job of the circulatory system?

To transport endocrine hormones

Which of the following describes the function of a neuron?

Transmits sensory information

what is active immunity?

Vaccinations

What demonstrate a chemical reaction of water?

Water produces gas with sodium metal

What is an example of chemical reaction of water?

Water produces gas with sodium metal

Which of the following sentences correctly uses a complex sentence structure?

Whenever the boys played in the rain, they came home with muddy clothes.

A

Which of the following excerpts from Joseph Stalin's 1941 radio broadcast would best be classified as incendiary? Choice Feedback A. The enemy is crafty, unscrupulous, experienced in deception. B. We must reckon with all this and not fall victims to provocation. C. Men of the Red Army are displaying unexampled valor. D. Is it really true that the German Fascist troops are invincible

D

Which sentence most accurately evaluates the spoken tone of Churchill's "Their Finest Hour"? Choice Feedback A. Churchill's confident tone will inspire other nations to fight alongside Great Britain. B. Churchill's honesty about consulting Britain's colonial governments reinforces his credibility. C. Churchill's defensive tone makes it appear that other leaders have questioned his leadership. D. Churchill's lack of enthusiasm makes it seem as if he is uninterested in the outcome of the war.

Define Ceaseless

Without stop or pause

Part five of the classic dramatic structure is the denouement, which ties up loose ends and reveals character in his or her new circumstances. a. Conclusion b. Renewal c. Narrative d. Prediction

a

what is glycerol?

a component of a triglyceride, which is a lipid used in long-term energy storage.

DNA appears in the body as two chins of nucleotides called?

a double helix.

what is Marfan syndrome?

a genetic disorder that affects the connective tissue.

Which of the following is the purpose of using a control groups in an investigation? a) to establish a standard for data comparison b)to provide the correct answer to an investigation c) to determine the dependent variables d) to function as a third variable group

a) to establish a standard for data comparison

what is hydrolysis?

addition of a water molecule to split apart the two monomers and release energy.

Which of the following are found in the hypodermis? a. apocrine glands b. hair follicles c. pores d. adipose tissue

adipose tissue

Examples of subordinate conjunction?

after, although, because, before, in order that, since, so that, unless, until, when, whenever, where, wherever, whether, and while.

subordinating conjunctions:

after, although, because, before, in order that, since, so that, unless, until, when, whenever, where, wherever, whether, while

femoral artery

artery that supplies oxygenated blood to the lower limb (leg)

Which of the following transitional expressions should be used to show cause and effect? a. Likewise b. Consequently

b

Which valve is found between the left atrium and the left ventricle? aortic valve bicuspid (mitral) valve tricuspid AV valve tricuspid (mitral) valve

bicuspid (mitral) valve

what is the parasympathetic system responsible for?

bodily functions when we are at rest: it stimulates digestion, activates various metabolic processes and helps us to relax.

Which of the following best supports the author's point of view that Hummingbirds are "marvel of speed, agility, and beauty"? a. A citation from a magazine article about migration b. A citation from a pamphlet on the wildlife of Florida c. A citation from a zoologist d. A citation from an archaeologist

c. A citation from a zoologist

In plants, ________ involves the formation of a _____? A cytokinesis, cleavage furrow B telophase, cell plate C cytokinesis, cell plate D prophase, DNA

c. cytokinesis, cell plate

What type of cells result from mitosis? A one diploid cell B two haploid cells C two diploid cells D four haploid cells

c. two diploid cells

Blood returns to the heart from the:

capillaries via venules, which merge to form veins.

The semifluid mass of partly digested food that moves from the stomach to the small intestine is?

chyme

Capillaries

connect arteries and veins

nonspecific immune response:

cytokine-mediated inflammation

What is NOT true about RNA? A It contains a sugar called ribose. B Thymine is not one of its base pairings. C It is unstable in alkaline conditions. D Its sugar is more reduced than DNA's sugar.

d. Its sugar is more reduced than DNA's sugar

blood vessels cooling of the body:

dilate

what is in the revision stage?

edit the work, check for spelling, punctuation, grammar and formatting

what does lipase break down?

fat

the cerebrum is divided into 4 lobes called:

frontal, parietal, occipital, and temporal lobes.

His expression seemed to "intimate" that he was growing impatient.

imply

where are protein found?

in hair, muscle, bone, and nearly all other tissues and cells.

Which of these is an example of protein-related malnutrition? kwashiorkor rickets anemia scurvy

kwashiorkor

The arteriole entering the glomerulus, the afferent arteriole:

larger than the arteriole exiting the glomerulus.

Which of these is not a lymphoid organ? liver tonsils thymus spleen

liver

Waste products are absorbed into the:

lymph

What protects the skin from UV?

melanin

pineal gland hormone:

melatonin

The monomer for nucleic acids is called?

nucleotide

Which of the following is the region of the kidney that contains the glomerulus of the nephron? a. medulla b. pelvis c. cortex d. adrenal

Cortex

Tarsal

Flat bones

Which of the following sentences correctly uses a compound sentence structure?

After they played basketball all day, the girls were very tired.

Today I had lunch with my friend. I called her yesterday and asked her to bring our high school yearbook to the restaurant. She forgot to bring it. After lunch, she promised to bring it to me in the morning.Based on the passage above, which of the following events occurred first?

Asked her to bring the yearbook

Which of the following sources contains copyright information of a book?

Publication page

Which of the following information sources contains copyright information for a given book?

Publication page.

Which of the following options most accurately summerizes the passage?

Ik the answer of it!

Bile, salt & acids are also absorbed in the:

Ileum

what is codominance?

both alleles are independently expressed.

15) Which of the following types of bonds link base pair nucleotides within a double strand of DNA? a) Covalent b) Ionic c) Hydrogen d) Peptide

c

21) Which of the following is a hinge joint? a) Wrist b) Shoulder c) Jaw d) Thumb

c

what is the synonym of necessitate?

entail, involve, require, demands, warrants.

what does estrogen stimulates:

female secondary sex characteristics the role in menstrual cycle and pregnancy.

what is synthesis reaction?

also known as direct combination a chemical reaction in which more than one reactant component combines to form the product.

A family wants to make a 20% downpayment on a house that costs $20,000, which of the amount is the downpayment?

$40,000

A consumer shops for a sofa. The store is having a 20% sale of all furniture. What is the discount before taxes of a sofa that is originally $510?

$408

At the beginning of the month, a consumer had$437.52 in his bank account. During the month he made deposits of $256 and $217.14 and withdrawal of $80 and white checks for $115.98 and $108.90. Which of the following is left?

$578.18

adrenal glands

(Produce cortisol and stress hormones ) a pair of endocrine glands just above the kidneys. the adrenals secrete the hormones epinephrine (adrenaline) and norepinephrine (noradrenaline), which help to arouse the body in times of stress.

what is the cerebrum?

- The largest part of the brain - Responsible for thought for thoughts, perception, visual, & auditory processing.

Describe the role of myosin in skeletal muscle contraction

- makes up thick filaments - contains myosin heads that attach to myosin binding site during muscle contraction - Hydrolyze ATP into ADP and P(I)

autosomal dominant:

- males & females are equally likely to have the trait. - traits do not skip generation - the trait is present if the corresponding gene is present -there is male to male and female to female transmission.

interstitial endocrine cell:

- surrounds the seminiferous tubules. - produce androgen, such as testosterone and secrete it into the interstitial fluid.

The endocrine system controls the internal environment by regulating what?

-Chemical composition and volume.

what is prophase?

-Chromatin thickens into chromosomes and the nuclear membrane begins to disintegrate. -Pairs of centrioles move to opposite sides of the cell and spindle fibers form.

What are the 4 lobes of the corpus callosum called?

-Frontal -parietal -occipital, and temporal

A 12 2/4 door board is cut into 4 equal parts. Each part is then divided in half. Which of the following measurement is the final length of each?

1 19/32

Brandon wants to determine the density of a certain liquid. He selects an empty 50 mL container that has a mass of 30 g. Brandon then pours the liquid into the container, but he only has enough to fill the container to 80% of capacity. The mass of the container plus the liquid is 74 g. What is the density of the density of the liquid? a. 0.88 g/mL b. 1.00 g/mL c. 1.10 g/ mL d. 1.28 g/ mL

1.10 g/mL Why? The volume of the liquid is 80% of a 50 ml container: 0.80 X 50= 40 ml The mass of the container plus the liquid is 74g, but he container alone has a mass of 30g. Therefore, the mass of the liquid is 74g - 30g= 44g. Density is m/v, so the density of this is 44g/40ml- 1.1g/1ml

(3/7),(⅖),(½),(⅓) Using the numbers above, which of the following numbers is the least?

1/3

A child has a bottle full of pennies, nickles, dimes, and quarters. There are twice as many quarters as pennys, four times as many nickles as pennies and five times as many dimes. How many more dimes does the child have than quarters?

10 times as many

A consumer makes a $400 down payment on television that costs $1,570. Which of the following is the number of months it will take to pay off the television with monthly payments of 100?

12

solve the following problem: 6X5+4/2-2X5

22 How? 6X5= 30 4/2= 2 2X5= 10 30+2= 32-10= 22

Corpus callosum is subdivided into?

4 lobes

What is the life span and breakdown product of red blood cells? 1 year, oxyhemoglobin 2 months, biliverdin 6 months, ammonia 4 months, bilirubin.

4 months, bilirubin.

_4(⅙)+1(⅓)_ Simplify the expression above. Which of the following is correct?

4(1/2)

10. Of these compound words, which is spelled incorrectly? A. Stock-broker B. Ex-husband C. Mother-in-law D. Twofold

A

15. Which of the following options is correctly punctuated? A. The runway truck, which had been parked on a steep hill, swerved and teetered before jumping a curb and slamming into a tree. B. The runway truck, which had been parked on a steep hill swerved and teetered, before jumping a curb and slamming into a tree. C. The runway truck which had been parked on a steep hill, swerved and teetered before jumping a curb, and slamming into a tree. D. The runway truck which had been parked on a steep hill swerved, and teetered before jumping a curb and slamming into a tree.

A

Extending her arms over the left-field wall, the baseball plummeted toward the fan, and she caught it cleanly on the fly before displaying the home run ball for all to see. Which of the following phrases is misplaced in the sentence above? A. Extending her arms over the left-field wall B. toward the fan C. on the fly D. before displaying the home run ball for all to see

A

The store will stay open later during the holiday season. The store will stock a variety of seasonal merchandise. The store will have many discounts available for holiday shoppers. When revising the paragraph above, the writer should address which of the following? A. Sentence fluency B. Punctuation C. Paragraph organization D. Writing clarity

A

What are histones associated with? A. DNA B. Digestive system C. Circulatory system D. Reproduction

A

Which of the following sentence correctly follows the rules of punctuation and capitalization? A. There are two main sources of water pollution: Sewers and factories B. There are two main sources of water pollution: sewers and factories C. There are two main sources of water pollution; Sewer and Factories D. There are two main sources of water pollution; sewers and Factories

A

Which word would be used correctly in the blank in this sentence? "Tiffany, Arlena, and Jane worked as a team for ____ senior project." A. Their B. Ther C. There D. They're

A

Rapport:

A close and harmonios relationship in which people or groups concerned understand each others feelings

What is the spinal cord?

A cylindrical column of nerves that runs through the center of the spine.

Mustache: Which of the following statements describes the topic of the passage? A. Mustaches are a constant through different time periods. B. People who have mustaches are less likely to be famous. C. Mustaches were more popular in the early 1900s than now. D. Having a mustache means a person is creative.

A. Mustaches are a constant through different time periods.

Based on the passage, which of the following statements is true? A. The author provides facts to support the main idea B. The author uses an informal style that appeals to readers C. The author has an extensive background in bird studies D. The author relies on first-hand observation of chimney swift behavior

A. The author provides facts to support the main idea

Which of the following best summarize the passage? A. The diminishing numbers of chimney swift can be reversed by simple conservation efforts B. Chimney swift rely entirely on masonry chimneys to survive during their early migration C. Prefabricated chimneys are the one thing that threatens the future of the chimney swift D. The number of chimney swifts in the world has been slowly declining since the 1970s

A. The diminishing numbers of chimney swift can be reversed by simple conservation efforts

A human egg will develop into a female if it is fertilized by sperm containing? a. an X chromosome b. a Y chromosome c. an XY chromosome d. an XXY chromosome

A: A female has two X chromosomes, and a male has one X chromosome and one Y chromosome. For a human egg to develop into a female, it must be fertilized by a sperm containing an X chromosome.

Which of the following equations represents aerobic respiration? a. C6H12O6 + 6O2 > 6CO2 + 38ATP b. C6H12O6 + C6H12O6 > C12H22O11 + H2O c. C6H12O6 > 2CO2 + 2C2H5OH + 2ATP d. 6CO2 + 12H2O > C6H12O6 + 6O2 + 6H2O

A: Aerobic respiration is the breakdown of glucose and the formation of energy. The reaction shown in equation D is the reverse of aerobic respiration: photosynthesis. Equation C represents fermentation (anerobic), and equation B shows dehydration synthesis.

To be absorbed by cells, proteins must be changed to? a. amino acids b. sucrose c. fatty acids d. glycerol

A: Amino acids are the end products of protein digestion and are small enough to be absorbed.

A compound that can act like either an acid or a base is described as? a. amphoteric b. isomeric c. hydrolytic d. polymeric

A: An amphoteric substance can behave either as an acid or a base. Water is an example of an amphoteric substance because it can act as an acid by furnishing a proton, or it can act as a base by accepting a proton.

Two atoms are isotopes if they have the same atomic number but a different number of? a. neutrons b. mesons c. electrons d. protons

A: An example of isotopes are carbon-12 and carbon-14. Both have the same atomic number (number of protons), but although carbon-12 contains 6 neutrons, carbon-14 contains 8 neutrons.

Which of these biological processes includes the other three? a. cell respiration b Krebs cycle c. electron transport chain d. anaerobic splitting of glucose

A: Cellular respiration begins with the splitting of glucose. The products then enter the Kreb's cycle and then the electron transport chain.

Which of these substances is a product of cellular respiration in animals? a. ATP b. RNA c. ethanol d. oxygen

A: Cellular respiration is an energy-yielding process that releases ATP (adenosine triphosphate) from organic molecules.

A student studies a structure microscopically and notes that it contains glandular cells. It has blood vessels entering and leaving it, but it has no duct leading out of it. The structure must be? a. an endocrine gland b. an excretory gland c. a digestive gland d. a tear gland

A: Endocrine glands are ductless glands with blood vessel associations. Exocrine glands, such as digestive and tear glands, empty their secretions through ducts.

Which of the following terms are included in the scientific name of an organism? a. genus and species b. phylum and species c. family and genus d. kingdom and phylum

A: Genus and species make up the scientific name of an organism.

Shivering to maintain a 98.6F Temperature in cold weather is an example of? a. homeostasis b. synthesis c. hydrolysis d. transpiration

A: Homeostasis is the maintenance of a stable internal environment.

Which of the following statements best describes the functioning of a battery? a. it changes chemical energy into electrical energy b. it stores electricity c. it creates electrons d. it changes current into voltage

A: In an electrochemical cell, which makes up a battery, chemical energy is converted into electric current.

Compounds with different molecular structures but but the same formula are called? a. isomers b. isobars c. isotopes d. isotherms

A: Isomers are compounds that have the same formula but different molecular structure. Isobars indicate the same pressure. Isotopes are atoms with the same atomic number but different mass numbers, and isotherms indicate the same temperature.

Ohm's law is used tin the study of? a. electricity b. mechanics c. thermodynamics d. optics

A: Ohm's law applies to the study of electricity. It states that voltage (V) is proportional to current (I) and resistance (R):V=IR.

Which of the following terms refers to the capacity of a microscope to distinguish between objects that lie very close to each other? a. resolving power b. staining c. micro-dissection d. magnifying power

A: Resolving power is the capacity to distinguis between objects that lie very close to each other.

The solubility of a solid in a liquid generally increases with? a. an increase in temperature b. an increase in pressure c. a decrease in temperature d. a decrease in pressure

A: Solubility of a solid in a liquid increases with increasing temperature but is unaffected by pressure. Pressure will only affect the solubility of a gas.

the vaporization of solid dry ice to gaseous carbon dioxide by heating is an example of? a. sublimation b. evaporation c. precipitation d. condensation

A: Sublimation is the direct vaporization of a solid by heating, without passing through a liquid state.

The idea that new varieties of organisms are evolving today is best supported by the? a. increasing need for new antibiotics b. diminishing size of the ozone layer c. climate changes resulting in global warming d. increasing size of the human population

A: The developement of resistant strains of bacteria, thus prompting the need for new antibiotics, is an example of evolution occurring today. The ozone layer, global warming, and increasing population size are changes in selective pressure, but not examples of evolution.

According to this pyramid to support 100 lb of birds, which of the following is needed? a. more then 100 lb of insects b. more than 100 lb of snakes c. less than 100 lb of green plants d. 100 lb of insects

A: The diagram increases in size in the direction of birds to insects, indicating that more insects are required to support a certain amount of birds. Thus, 100 pounds of birds will consume an amount of insects excedding 100 pounds.

An overdose of a muscle relaxant may cause a person to cease breathing because? a. the diaphragm will be unable to contract b. the lung muscles will cease to function c. the muscles around the trachea will constrict d. the alveoli will constrict

A: The diaphragm must contract for inhalation to occur, and, as a muscle, the diaphragm is affected by the muscle relaxant.

Most of the work done by the human kidney occurs in the? a. nephron b. neuron c. ureter d. alveolus

A: The functional unit of the kidney is the nephron. Urine is formed in the nephron by the processes of filtration, re-absorption, and secretion.

Which of the following terms is the SI unit for work? a. joule b. watt c. ampere d. meter

A: The joule is the unit of energy equal to the work done by a force of 1 N acting over a distance of 1 m.

A student placed a stone in a gradulated cylinder containing 30cm3 of water. She then noted that the water level rose to 55cm3 mark. What is the volume of the stone? a. 25cm3 b. 30cm3 c. 55cm3 d. 85cm3

A: The stone causes the volume in the cylinder to increase by 25cm3 (55cm3-30cm3). The volume of a liquid displaced by a submerged object is equal to the volume of the object itself. Therefor, the volume of the stone is 25 cm3.

Which of these nitrogenous bases may be found in RNA, but not DNA? a. uracil b. adenine c. thymine d. guanine

A: Uracil is found in RNA but not DNA. Thymine is found in DNA but not in RNA. Guanine, cytosine and adenine are found in both.

As the angle of incidence of a ray of light striking a mirror increases, the angle of reflection will? a. increase b. decrease c. remain the same d. first increase, then remain constant

A: ccording to the law of reflection, the angle of incidence and the angle of reflection will be equal.

According to the recipe above, when should the onion be cooked?

After heating the oil in the pan

Ball and socket joints, pivot joints, plane joints, and hinge joints are which type of joints? Ball and socket and pivot joints are synovial joints. Plane and hinge joints are cartilaginous joints. All of them are cartilaginous joints. Ball and socket joints are synovial joints. The rest are fibrous joints. All of them are synovial joints.

All of them are synovial joints.

Which of the following details best supports the main idea of the passage, "He must stay here a full hour to win the bet."?

An hour in this place just was not worth it

A patient walks into a clinic with a sore leg... Which of the following is the definition of "bursitis" as used in the passage above?

An inflammation of a joint.

What is the Law of Dominance?

An organism with alternate forms of a gene will express the form that is dominant.

Which of these statements is correct? A reducing agent removes oxygen from, or removes electrons to, another substance. An oxidizing agent removes oxygen or electrons from other substances. A reducing reagent either adds oxygen to, or removes electrons from, another substance. An oxidizing agent either adds oxygen to, or removes electrons from, other substances.

An oxidizing agent either adds oxygen to, or removes electrons from, other substances.

The first four steps of the scientific method are as follows: I. Identify the problem II. Ask questions III. Develop a hypothesis IV. Collect data and experiment on that data Which of the following is the next step in the scientific method? a. Observe the data b. Analyze the results c. Measure the data d. Develop a conclusion

Analyze the results

"Upon receiving her minutely corrected essay, Melanie decided that her teacher's teaching style was far too pedantic." Using context clues in the sentence above, determine the meaning of the word "pedantic". A. prone to incorrectly identifying errors B. prone to focusing on small, potentially insignificant errors or details C. given to flights of fancy and whimsy D. excessively generous or free in correction

B

1. The rules for lighting are not always this simple, but the possibilities are endless. 2. The secret to taking beautiful photographs is an understanding of composition and lighting. 3. When both composition and lighting are well considered, a fantastic photo is just a click away. 4. With a proper understanding, lighting can be used to capture detail, provide contrast, and express moods. 5. A simple rule of composition is to mentally divide the frame of your view into thirds, both horizontally and vertically, and center your focal point on one of the intersections. If sentence number 2 is the topic sentence, which of the following sequences of the remaining sentences best forms an organized, local paragraph? A.3,4,1,5 B.4,1,5,3 C.4,5,1,3 D. 3,5,1,4

B

3. I had to wrack my brain and plum the depths of my resources the foul consequences of my previous vice. Which of the following homophones should be used to correct the sentence above? A. Fowl B. Plumb C. Rack D. Vise

B. plumb

In humans, if the diaphragm is pushed upward there is a decrease in chest volume. this decrease is followed by? a. an increase in pressure in the chest cavity and inhalation b. an increase n pressure in the chest cavity and exhalation c. a decrees in pressure in the chest cavity and inhalation d. a decrease in pressure in the chest cavity and exhalation

B: A decrease in volume results in an increase in pressure (Boyle's law) and thus exhalation.

When red food coloring is added to a beaker of water, the coloring slowly spreads until the water is all colored pink. This is best explained by the process of? a. peristalsis b. diffusion c. active transport d. osmosis

B: A drop of food coloring will slowly diffuse through the beaker of water until it reaches equilibrium when it will be fully dispersed. It moves from the area of greater concentration to the area of lower concentration.

A decrease in acetylcholine synthesis in humans would most likely lead to? a. an increase in protein digestion b. a decrease in nerve impulse transmission c. an increase in muscular activity d. a decrease on white blood cells

B: Acetylcholine is a common neurotransmitter. Neurotransmitters act as chemical messengers and spread informtation from one neuron to the next.

Machines may be used for all of the following purposes except to? a. multiply force b. increase energy d. transform energy d. multiply speed

B: Conservation of energy forbids a machine from increasing energy. Machines are designed to use energy efficiently. Machines cannnot be designed to increase energy; they use energy to do work.

Which of the following is not used as a means of separating mixtures? a. filtration b. electrolysis c. chromatography d. distillation

B: Filtration, chromatography, and distillation are means of separatiing componets of mixtures (physical means). Electrlysis is used to decompose compounds, as in the electrolysis of water.

In a food chain, an organism that feeds on green plants is known as a? a. decomposer b. producer c. first-order consumer d. second-order consumer

B: Green plants are producers because they produce their own food by means of photosynthesis. Consumers depend on other organisms for their food. A first-order (primary) consumer is an herbivore and eats plants and alge. A second-order (secondary) consumer is a carnivore and eats herbivores.

How much of 12g of a radioactive isotope with a half-life of 20 years will be left after 40 years? a. 0g b. 3g c. 6g d. 8g

B: Half-life is the amount of time it takes for one half of the mass of a sample of radioactive substance to decay. In 20 years a 12-g isotope with a half-life of 20 years will be 6g. Twenty years after the the isotope will be 3g.

Which are herbivores? a. green plants b. insects c. birds d. snakes

B: Herbivores are animals that feed on plants. Thus, insects are herbivores because they consume green plants.

Which of the following phenomena is most likely to speed the evolutionary rate of a species? a. lack of environmental changes b. increase in mutations c. decrease in migration d. increase in death rate

B: Mutations of DNA are the orginal source of variation and alter the gene pool in a population. An increase in the number of mutations increases the variation in the gene pool and thus speeds up the evolutionary rate of a species.

Undisturbed layers of rock were examined. Mammal, fish, and bird fossils were found only in the upper layers. Fish were in layers beneath those, and neither fish, birds, nor mammals were in the deepest layers. This would tend to support the hypothesis that? a. vertebrates always coexisted with invertebrates b. fish were the first vertebrates to evolve c. all vertebrates evolved at about the same time d. birds evolved before fish

B: New layers of sediment cover old ones and provide a record of emergence of flora and fauna. the fish were the only fossils found in the layers beneath the upper layers, indicating that they evolved first.

During the 1930s and through the 1950s, thousands of American children received x-ray treatment for swollen tonsils and adenoids. X-rays are no longer used to treat this condition because large doses of radiation? a. were found to stimulate excessive hair growth b. are thought to cause certain types of cancer c. are thought to increase the severity of acne during adolescence d. were found to increase the number of cases of tonsillitis.

B: Radiation exposure has been linked to cancer.

An equal-arm balance is balanced when 20 washers are on one side and 10 bolts are on the other. Four bolts are added to one side. How many washers must be added to the other side o maintain balance? a. 14 b. 8 c. 4 d. 2

B: Since the equal-arm balance is balanced at a ration of 20 washers to 10 bolts, the weight of the bolts must be double that of the washers. Two washers are equivalent in mass to 1 bolt. Thus, 4 bolts are equivalent in mass to 8 washers. An increase of 4 bolts on one side will require 8 washers on the other side to maintain the balance.

In a food chain involving grass, grasshoppers, birds and mammals, the orginal source of energy is? a. glucose b. sunlight c. chlorophyll d. ATP

B: Sunlight is absorbed by the chlorophyll contained in plants. The absorbed energy fuels photosynthesis which produces glucose. In this food chain, grass is the producer.

A woman standing at a bus stop hears the siren of an approaching ambulance. As the ambulance passes by her, she observes a shift in the frequency of the siren. The effect she heard is known as the? a. photoelectric effect b. Doppler effect c. phase shift effect d. Einstein effect

B: The Doppler effect occurs whenever there is a relative morion between the source of the sound waves and the observer. The pitch of a siren becomes higher as it approaches you and lower as it drives away. the siren has not changed its frequency, byt the motion of the siren toward you increases the frequency of the sound waves you hear.

Nerve impulses form the retina are transmitted to the brain by the? a. olfactory nerve b. optic nerve c. Eustachian tube d. auditory nerve

B: The Eustachian tube and auditory nerve are associated with the ear, and the olfactory nerve is involved with the sense of smell.

On top of a mountain, the boiling point of water is? a. lower then at sea level, because the atmospheric pressure is higher on the mountain b. lower then at sea level, because the atmospheric pressure is lower on the mountain c. the same as at sea level, because the atmospheric pressure is the same in both locations d. higher then at sea level, because the atmospheric pressure is lower on the mountains

B: The boiling point of a solution is the temperature at which the vapor pressure is equal to the pressure of the atmosphere.

In the equilibrium system A + B <> C + 30 kcal we can say that the? a. reverse reaction is exothermic b. forward reaction is exothermic c. reverse reaction requires a catalyst d. forward reaction is endothermic

B: The forward reaction is exothermic because heat is shown as one of the products. The reverse reaction is endothermic since heat is shown as a product. We do not know about a catalyst from the information given.

Compound A reacts with compound B to give compound C plus compounds D and E, as shown by the following equation: A+B> C+D+E If in a closed system, 7g A react with 4g B to give 2g C plus 3g E, how many grams of D are produced? a. 5 b. 6 c. 11 d. 16

B: The law of conservation of mass says that mass may be neither created nor destroyed. Therefor, the mass of the reactants must be equal to the mass of the products of a reaction.

In a flask containing 500 mL of NaCl(aq), the solvent is? a. Na+ b. H2O c. Cl- d. NaCl(s)

B: The notation "(aq)" refers to an aqueous solution, indicating that the solvent is water.

More energy is used in pushing a box up along an inclined plane to a height of 4 m then in liftting the box straight up to the same height. This is because? a. more force is needed in pushing the box then in lifting it b. work is done against friction in pushing the box along the plane c. the box moves a greater distance when pushed along the plane then when lifted d. it takes more time to push the box along the plane then to lift it

B: Whenever a box is along a surface, energy goes into counteracting friction. The amount of energy required is determined by the coefficient of friction.

Based on the passage, which of the following can be inferred as the reason why Steven accepts David's $5 wager?

Based on his daily routine, he assumed he could easily win the bet

the passage stimulus question 2 of 6 based on the passage, which of the following can be inferred as the reason why steven accepts david's 5 dollar wager? (Steven walked through the graveyard)

Based on his daily routine, he assumed he could easily win the bet

Which of the following inferences can the reader draw according to the passage?

Being on time for a job interview and dressing appropriately is important to securing a job.

Prefix for "inter":

Between

Which of the following describes one responsibility of the integumentary system? a. distributing vital substances (such as nutrients) throughout the body b. blocking pathogens that cause disease c. sending leaked fluids from cardiovascular system back to the blood vessels d. storing bodily hormones that influence gender traits

Blocking pathogens that cause disease

What makes up the central nervous system?

Brain and spinal cord

Which gland from the men's body secretes a fluid into the urethra to neutralize the acidity in the urethra?

Bulbourethral

12. The top-secret blueprints were for an intergalactic spaceship. Which of the following is the meaning of the prefix "inter-"as used in the sentence above? A.Past B.Into C.Among D. Below

C. among

Which of the following hormones are secreted directly by the hypothalamus? A . melatonin B oxytocin C dopamine D calcitonin

C. dopamine

When looking through a microscope, a student observes centrosomes in a group of cells. The cells are most likely from the? a. root of a bean plant b. leaf of a moss c. skin of a mouse d. stem of a fern

C: A centrosome is an organelle found in animal cells.

A student wanted to test the effects of different sugars on the growth of a certain mold. The sugars to be tested were glucose, fructose, and sucrose. One gram of each sugar was placed in a tube of 10 mL of water, and a bit of the mold was put in each of the three solutions. To add a control to the experiment, the student should also have? a. made two

C: A control for this experiment would be to grow the mold in just water so that the results of the mold growth with sugar can be compared to the mold growth without sugar.

Quantities having both magnitude and direction are called? a. line segments b. scalars c. vectors d. directrixes

C: By definition, vectors are quantities with magnitude and direction.

Which Group of chemicals is not normally found in most living things? a. carbohydrates b. proteins c. silicates d. nucleic acids

C: Carbohydrates, proteins, and nucleic acids (RNA and DNA) are all very important chemicals in living things.

As the eardrum is made to vibrate more rapidly, the sound is perceived as? a. louder in intensity b. softer in intensity c. higher in pitch d. lower in pitch

C: High-pitch sounds are heard when the receptor cells closer to the oval window are stimulated, and low-pitch sounds result from the stimulation of the cells that are farther down the cochlea.

An excess of which of these ions tends to make a solution acidic? a. chloride b. hydroxyl c. hydronium d. sodium

C: Hydronium ions, or H3O+, are essentially hydrogen ions combined with a nearby water molecule. In essence, acids are characterized by the presence of hydrogen or hydronium ions.

If a car's rate of change of velocity is negative, the car is? a. speeding up b. maintaining a constant speed c. slowing down d. stopped

C: If a car's rate of change of velocity is negative, the car's acceleration is negative. this means that the car is decelerating (slowing down).

What types of body cells are most directly involved when a person walks up hill? a. smooth muscle cells b. subcutaneous cells c. striated muscle cells d. epithelial cells

C: Muscles are the tissues mostly involved when a person is exercising. Smooth muscle is found in the lining of organs such as the uterus. Skeletal muscle, such as that found in the muscles of the legs and arms is striated.

Certain seaweeds accumulate iodine in a concentration as much as a million times greater than that of the surrounding ocean. How must this intake be accomplished? a. osmosis b. diffusion c. active transport d. passive transport

C: Since the concentrations are so much greater in seaweed than in the surrounding water, iodine must have to move against a concentration gradient.

If a person's gallbladder were removed by surgery, which of the following substances would he have the most difficulty digesting? a. carbohydrates b. nucleic acids c. fats d. proteins

C: The gallbladder stores bile, which is the substance produced by the liver. Bile contains bile salts, which aid in the digestion and absorption of fats.

A student wants to grow a bacterial culture. Which of these environments is best suited for growing most most kinds of bacteria? a. a lighted window at 72 F b. a refrigerator at 45 F c. an incubator at 37 C d. a freezer at 10 C

C: The optimal temperature at which bacteria grows is 37C.

Bean plant seeds are germinated in a dark closet. The seedlings lack the color green. Which of the following statements is the best explanation for this? a. Bean plants are heterotrophs b. Bean seedlings lack nitrogen in their seed leaf c. The absence of an environmental factor limits the proper genetic expression d. Bean plants cannot be classified as green plants

C: The seedlings lacked light, which interfered with their development.

The specific heat of water is 1 cal/g/C. How many calories of heat are needed to raise the temperature of 30g of water from 25 C to 75 C. a. 50 Cal b. 750 Cal c. 1,500 Cal d. 2,250 Cal

C: The specific heat capacity of a substance is the heat required to raise the temperature of 1g of a specific substance by 1C. In the case of water, the specific heat capacity is 1 Cal/g/C. In this situation, you want to change 30g of water by 50C.

The scientific name of a beaver is Castor Canadensis. Similarly, a coyote is Canis latrans, a mountain lion is Felis concolor, a wolf is Canis lupus, and a cougar is Puma concolor. Which two of these animals belong to the same genus? a. beaver and wolf b. cougar and mountain lion c. coyote and wolf d. mountain lion and cougar

C: The wolf and the coyote both belong to the genus Canis. the scientific name consists of the genus name and the species name.

Which of these substances has the highest boiling point? a. ether b. ethanol c. water. d. glycerol

C: Water molecules are attracted to each other by hydrogen bonds. The others are not dipoles, so the attraction between the molecules is minimal.

Twenty-five milliliters of a saline solution contains 5g of NaCl. What percentage solution is it? a. 5% b. 12.5% c. 20% d. 25%

C: We assume that 1 mL of water has a mass of 1g. So 25 mL equals 25g. Divide 5 by 25 and convert to percentage.

A 5-year-old and her father each lifted identical chairs from the floor to the table top. Which person did the most work? a. the father b. the 5-year-old c. they both did the same amount d. not enough information is given

C: Work done is independent of who is doing it. Since both people move the object over the same distance and use the same force, they do the same amount of work.

What are the two major parts of the nervous system?

CNS (Central nervous system) PNS (Peripheral nervous system)

Which of the following sentence from the passage includes biased language?

Carter said that the solar panels were "a small part of one of the greatest and most exciting adventures ever undertaken by the American People."

Which of the following sentence from passage includes biased language.

Carter said that the solar panels were a small part of one of the greatest and most exciting adventures ever undertaken by American people

White house solar panels Which of the following sentences from the passage includes biased language?

Carter said that the solar panels were a small part of one of the greatest and most exciting adventures ever undertaken by the American people.

Which substance makes up the pads that provide support between the vertebrae? a. bone b. cartilage c. tendon d. fat

Cartilage

Which of the following arteries supplies the heart with blood? a. carotid b. coronary c. subclavian d. brachiocephalic

Cartoid

What connects the large intestine to the appendix?

Cecum

What two criteria for classifying epithelial tissue are cell layers and _________. Which of the following completes the sentence above? a. cell composition b. cell absorption c. cell shape d. cell stratification

Cell shape

Organized from high to low, the hierarchy of the human body's structure is as follows: organism, organ systems, organs, tissues. Which of the following comes next? a. molecules b. atoms c. cells d. muscle

Cells

What are the 2 major parts of the nervous system?

Central and peripheral nervous system

Which part of the brain is responsible for balance, movement, and coordination?

Cerebellum

The police assumed the thieves must have had to "scale" the seven-foot fence in order to escape capture. What "scale" means?

Climb over

Each group of three DNA letters is called a _______; each codon codes for an amino acid, and he amino acids are linked together as translation takes place.

Codon

/part five of the classic dramatic structure is the denouement, which ties up loose ends and reveals the main character in his or her new circumstances/ - Which of the following terms best defines the word "denouement" as used in the sentence above?

Conclusion

Which of the following lists support the main idea of the passage? A. Chimney swifts appear in North America in summer, migrate to the tropics in winter, and give birth in the spring B. Logging and deforestation, changes in the ways we heart our buildings and chimney capping have robbed the chimney swift of its preferred roosting sites C. The small, slender bird known as the chimney swift migrates in large numbers, can be seen only at dusk, and raises its young in the tropics D. Metal-lined chimneys, chimney towers, and reforestation efforts will ultimately result in the growth of the chimney swift population

D. Metal-lined chimneys, chimney towers, and reforestation efforts will ultimately result in the growth of the chimney swift population

6.The gist of the saying went something like this: If a person wants to get ahead in life, you have to be willing to work for it. Which of the following language conversions is violated in the sentence above? A.Use of a colon B.Punctuation of an introductory clause C.Verb tense D.Pronoun-antecedents' agreement

D. Pronoun-antecedent agreement

Which of the following can be inferred from the information in the passage? A. Logging industry production dramatically increased around 1970 B. Changes in home design and home heating occurred in the late 1970s. C. They way we tracked chimney swift populations changed in the 1970s D. The chimney swift's adaptability was severely hindered around 1970s

D. The chimney swift's adaptability was severely hindered around 1970s

Mustache: Which of the following information does the author use in the passage to support the claim that the mustache has influenced other creations? A. The existence of shaving in ancient times. B. The meanings associated with mustaches. C. The presence of famous people with mustaches. D. The invention of the mustache cup.

D. The invention of the mustache cup.

A neuron that transmits impulses from the receptors to the spinal cord is called? a. motor neuron b. an associative neuron c. an interneuron d. a sensory neuron

D: A sensory neuron conveys a message from a sensory organ to the spinal cord, which transmits the message via a motor neuron to the effector muscle.

A closed container of hydrogen gas is warmed from 20 C to 25 C. If the volume remains the same, what will happen to the pressure in the container? a. it will remain the same b. it will decrease c. it will fluctuate d. it will increase

D: According to Gay-Lussac's law of gases, as temperature increases at a constant volume, pressure increases proportionally. The pressure will increase.

Which of the following structures is the functional unit of the human lung? a. bronchus b. Malpighian tubule c. nephron d. alveolus

D: Alveoli are the functional units of lungs. The oviduct, or fallopian tube, conveys the released egg to the uterus. Malpighian tubules are excreatory organs found in insects, and the nephron is the functional unt of the kidneys.

Which of the following organisms has a nutritive process most similar to that of animals? a. seaweed b. oak tree c. grass d. bread mold

D: Bread mold is a fungus. Fungi are unable to make their own food and are dependent on other life forms for nutrition. Fungi are heterotrophs. The other choices are all autotrophs.

Which of the following would be classified as an example of connective tissue? a. striated muscle b. epidermis c. nerve d. tendon

D: Connective tissue binds and supports other tissues. For example, tendons attach muscle to bone. Muscle is muscular tissue, epidermis is epithelial tissue, and nerve is nervous tissue.

Suppose cube A is 10cm along each edge and cube B is 5 cm along each edge. What is the relationship of the volume of cube A to that of Cube B? a. cube A has two times the volume of cube B b. cube A has four times the volume of cube B c. cube A has six times the volume of cube B d. cube A has eight time the volume of cube B

D: Cube A has a volume of 1,000 cm3, whereas cube B has a volume of 125 cm3. Cube A has 8 times the volume of cube B.

An object is traveling at a constant velocity of 100m/sec. How far will it travel in 20 sec? a. 5m b. 120m c. 200m d. 2,000m

D: Distance traveled is directly proportional to velocity and time. Distance = velocity x time, distance = 100m/secx20sec=2,00m

In Einstein's equation, E=mc2, which does C represent? a. the quantum numbers of the atoms involved b. the number of coulombs c. the number of calories of heat d. the speed of light

D: Einstein's equation, E=MC2, means energy = mass x (speed of light)2

Which of the following equations represents a neutralization reaction? a. 2Na+MgCl2 > Mg+2NaCl b. CaCO3+4C > 3CO+CaC2 c. 4NH3+5O2 > 6H2O+4NO d. H2CO3+Ca(OH)2 > 2H2O+CaCO3

D: Equation d is a reaction between an acid (carbonic acid) and a base [(Ca(OH)2] to form water and a salt. It is the only neutralization reaction among the choices.

The law of chemical equilibrium states that in the reversible reaction N2 + 3H2 > 2NH3 the equilibrium constant is expressed as? (the equations are long so I am only putting the answer down.)

D: K=(NH3)2/(N2)(H2)3

In humans, which of these blood components carries most of the oxygen? a. plasma b. white blood cells c. platelets d. red blood cells

D: Red blood cells contain hemoglobin, which binds, transports and delivers oxygen throughout the body.

Which of the following formulas is most probably for a compound formed from aluminum and sulfur? (Aluminum has an oxidation number of 3+. Sulfur has an oxidation number of 2-.) a. AIS b. Al3S2 c. Al2S d. Al2S3

D: The ions that make up a compound form in such a ratio as to produce a neutral compound. Thus, 2 aluminum ions (+3) are needed for 3 sulfur inos (-2each): Al2O3

Which of the following organelles is not involved in protein synthesis? a. ribsome b. mitochondrion c. rough endoplasmic reticulum d. nucleus

D: The nucleus contains DNA. The ribosomes, located on the rough endoplasmic reticulum, are the site for protein synthesis.

Which of the following sentences from the passage presents the topic of the passage?

Despite the fact that sleepwalking or noctambulation occurs frequently, this odd ailment remains a baffling, uncanny - if not frightening - phenomenon.

Cyotoxic T cells

Destroy pathogens and infected cells

White house solar panels (Solar panels are a popular choice for homeowners...) Which of the following sentences best summarizes the text?

Diverse presidential philosophies have affected white house energy resource use.

Which of the following best summarizes the text:

Diverse presidential philosophies have affected white house energy.

what does the Brachiocephalic artery do?

Divides into the right common carotid and right subclavian arteries.

Intransitive verbs:

Do not point to a receiver of an action. in other words, the action of the verb does not point to a subject or object

stimulus question 1 of 2: Which of the following can be inferred based on this memo? (Winter will soon be here)

Employees who need a laptop to work from home during winter weather will be provided one

Memorandum (winter will be soon here...) Which of the following can be inferred based on the memo?

Employees who need a laptop to work from home during winter weather will be provided one.

Stimulus question 3 of 3 which of the following sources in this passage most likely to appear? (Henry Tudor)

Encyclopedia

What responds to environmental changes?

Endocrine system

Blooding clotting involves which of the following proteins

Fibrinogen

What is the Law of Independent Assortment?

Genes for different traits are sorted separately form one another so that the inheritance of one trait is not dependent on the inheritance of another.

Which of the following statements best supports the hypothesis that viruses can cause cancer?

Genes that regulate cell division are found in some virus

S & O are in the same periodic group and share chemical properties, but H2S is a gas at room temperature. What is the main reason for this?

H2O has a stronger intermolecular bonds than H2S

which of the following reactions would take place after walking into a very cold room? a. sebaceous glands would decrease excretion. b. sudoriforous glands would increase excretion. c. blood vessels would increase blood flow to the skin. d. Hair follicles would decrease air flow across the skin

Hair follicles would decrease air flow across the skin

example intransitive verb:

He plays/ John writes well

Something the author doesn't state directly, but readers can assume base on what the author does say is?

Implication

Stimulus 5 of 6 which of the following sentences from the passage includes biased language ( WHITE HOUSE SOLAR PANEL)

In 2010, president barak obama decided to install solar panels on the white house as part of his administration's focus on environmental issues

The Welsh kingdom of Gwynedd existed as an independent state from the early 5th century, when the Romans left Britain, until the late 13th century, when the king of England took control of Wales. which of the following functions as an adjective in the sentence above? a. independent b. century c. government d. control

Independent

The students in a science class conducted an experiment to see whether several samples of different materials were attracted by a magnet. Which of the following type of property were they identifying with their experiment? a. Intensive physical property b. Extensive physical property c. Chemical property d. Reactivity property

Intensive physical property

Uvula

Is found in the back of the throat and prevents food entry into the nasal passages

marrow

Is inside the bone and does not connect to other bones

Trachea (windpipe)

Is the large tube containing cartilaginous rings through which air passes into and out the lungs

The gonadal artery

Is the primary artery that supplies oxygenated blood to the gonads and male reproductive system. It call the testicular artery in male and the ovarian artery in female

Which of these statements regarding isotopes is not true? Radio-isotopes are isotopes with unstable nuclei that emit radiations. Isotopes have the same number of protons and electrons in each atom, but a different number of neutrons in their nucleus. Isotopes are atoms of the same element with different mass numbers. Isotopes of an element have the same chemical and physical properties.

Isotopes of an element have the same chemical and physical properties.

What is the function of amylase in the body? It breaks down starch into simpler sugars. It breaks down fat into fatty acids. It breaks down fat into glycerol. It breaks down proteins into amino acids.

It breaks down starch into simpler sugars.

Which of these statements regarding facilitated diffusion across a cell membrane is true? It is a kind of active transport (e.g., ATP hydrolysis) that requires transmembrane/carrier proteins. It is a kind of active transport (e.g., ATP hydrolysis) that requires protein pumps. It is a kind of passive transport (i.e., does not use energy such as hydrolysis of ATP) that requires transmembrane/carrier proteins. It is a kind of passive transport (i.e., does not use energy such as hydrolysis of ATP) that does not require transmembrane/carrier proteins.

It is a kind of passive transport (i.e., does not use energy such as hydrolysis of ATP) that requires transmembrane/carrier proteins.

Which of these statements regarding the process of fermentation is true? It is an example of an anaerobic process where yeast acts on a sugar solution to produce carbon dioxide and alcohol. It is an example of an anaerobic process where bacteria act on a protein solution to produce carbon dioxide and alcohol. It is an example of tissue respiration where yeast acts on carbohydrates to produce oxygen and alcohol. It is an example of an aerobic process where yeast acts on a sugar solution to produce oxygen and alcohol.

It is an example of an anaerobic process where yeast acts on a sugar solution to produce carbon dioxide and alcohol.

Which of the following is a hinge joint?

Jaw

What is responsible for speech?

Larynx

Chimney Swifts Which of the following lists support the main idea of the passage?

Logging and deforestation, changes in the ways we hear our buildings, and chimney capping have robbed the chimney swift of its preferred roosting sites.

Which of the following list support the main idea of the passage of the Chimney Swift Passage?

Logging and deforestation, changes in the ways we heat our buildings, and chimney capping have robbed the chimney swift of its preferred roosting sites.

Which would be the appropriate unit to express the length of the arm bones of a Deinocheirus mirificus? a. millimeter b. centimeter c. meter d. kilometer

Meter

A student is comparing four different types of cells and observes that one contains many more mitochondria than the others. The students should conclude that the cell with the greatest number of mitochondria is most likely which of the following?

Muscle cell

What happens if acetylcholinesterase is inhibited by the synapse?

Muscle contraction/ spasms

Despite the lack of rapport among the members of my group in science class, we were still able to produce a valuable report, lead a good classroom discussion and receive high grades on the project. Which of the following is the meaning of "rapport" as used in the sentence above?

Mutual understanding

Innate immune system cells that attack host cells harboring an intracellular pathogen:

Natural killer cells

Which innate immune system cells attack the host cells harboring an intracellular pathogen?

Natural killer cells

Where is the parathyroid gland located? a. neck b. back c. side d. brain

Neck

what does the word infamous mean?

Notorious

Which of these statements regarding the "oxygen debt" created during heavy exercise is true? Oxygen debt is created due to build-up of acetic acid in the muscles, which needs a high level of oxygen to be oxidized. Oxygen debt is created due to build-up of lactic acid in the liver, which needs a high level of oxygen to be stored. Oxygen debt is created due to build-up of lactic acid in the muscles, which needs a high level of oxygen to be oxidized. Oxygen debt is created due to build-up of lactic acid in the muscles, which needs a high level of carbon dioxide to be oxidized.

Oxygen debt is created due to build-up of lactic acid in the muscles, which needs a high level of oxygen to be oxidized.

What cell produce HCL (hydrochloric acid) ?

Parietal cells

what is an enzyme that is capable of digesting proteins in an acid PH?

Pepsin

What is extensive physical property?

Physical properties that depend upon the amount of matter being measured

What is intensive physical property?

Physical properties that do not depend upon the amount of substance.

Which gland stimulates the secretion of the other glands?

Pituitary gland

There are many tall tales from old world sailors and farmers, but some of them about weather prediction are true. Here are three general rules you can follow to predict precipitation. If there is a ring around the moon at night, rain or snow may be coming. Rain or snow may be imminent when puffy cumulus clouds change and begin to flatten. And finally, if a north wind shifts in a counter-clockwise manner, rain or snow could be on its way.What is the main idea of the paragraph?

Precipitation can often be forecasted by a few simple observations.

Which of the following inferences can be made from the following section from the passage ?It was ahead of times .He set this example for the american people as a part of a campaign.

President Carter's conservation efforts significantly reduced energy waste in America

White house solar panels Which of the following inferences can be made from the following section from the passage? Carter was ahead of the times. He set this example for the American people as part of a campaign to conserve energy.

President Carter's conservation efforts significantly reduced energy waste in America.

According to the passage, which of the following events occurred the second?

President Reagan took office in 1981

White house solar panels According to the passage, which of the following events occurred second?

President Reagan took office in 1981.

What are the two types of measurement important in science? a. quantitative and numerical b. qualitative and descriptive c. numerical and scientific d. quantitative and qualitative

Quantitative and qualitative

If the weather forecast suggests that the current north wind will shift in a counter-clockwise direction to a southwesterly wind by afternoon, what might you predict?

Rain

A

Read the following excerpt from A Room of One's Own by Virginia Woolf: And here, I said, opening a book about music, we have the very words used again in this year of grace, 1928, of women who try to write music. 'Of Mlle. Germaine Tailleferre one can only repeat Dr. Johnson's dictum concerning, a woman preacher, transposed into terms of music. "Sir, a woman's composing is like a dog's walking on his hind legs. It is not done well, but you are surprised to find it done at all." So accurately does history repeat itself. Which statement best states the idea that this excerpt helps to develop? Choice Feedback A. In the 20th century, women still face hostility as they endeavor to create. B. The number of disciplines from which women are barred is ever shrinking. C. Women must create a room to call their own rather than seek it out. D. Criticism of women from men is often less honest and gentler.

A

Read the following excerpt from A Room of One's Own by Virginia Woolf: Occasionally an individual woman is mentioned, an Elizabeth, or a Mary; a queen or a great lady. But by no possible means could middle-class women with nothing but brains and character at their command have taken part in any one of the great movements which, brought together, constitute the historian's view of the past. Which statement best states the idea that this excerpt helps to develop? Choice Feedback A. Women have been historically held back from greatness. B. Women's lives have been deliberately left out of historical record. C. Women have traditionally been encouraged to write about their lives. D. Women's contributions have been erased from history.

A

Read the following excerpt from A Room of One's Own by Virginia Woolf: [A]ny woman born with a great gift in the sixteenth century would certainly have gone crazed, shot herself, or ended her days in some lonely cottage outside the village, half witch, half wizard, feared and mocked at. For it needs little skill in psychology to be sure that a happily gifted girl who had tried to use her gift for poetry would have been so thwarted and hindered by other people. Which statement best states the idea that this excerpt helps to develop? Choice Feedback A. Society has held women back from greatness. B. Men have conspired to keep women powerless. C. Historians have glossed over women's achievements. D. Emotions have prevented women from success.

A

Read the following excerpt from Emperor Quian Long's Letter to King George III in England in 1793: But as the tea, silk and porcelain which the Celestial Empire produces, are absolute necessities to European nations and to yourselves, we have permitted, as a signal mark of favour, that foreign hongs should be established at Canton, so that your wants might be supplied and your country thus participate in our beneficence. Based on this excerpt, what can most logically be concluded about Emperor Quian Long's point of view? Choice Feedback A. Trade with China is an enormous privilege. B. China highly values British goods. C. The British are better trade partners than other countries. D. Sharing with others is a moral principle.

A

Read the following excerpt from Emperor Quian Long's Letter to King George III in England in 1793: If you assert that your reverence for Our Celestial dynasty fills you with a desire to acquire our civilisation, our ceremonies and code of laws differ so completely from your own that, even if your Envoy were able to acquire the rudiments of our civilisation, you could not possibly transplant our manners and customs to your alien soil. Therefore, however adept the Envoy might become, nothing would be gained thereby. Based on this excerpt, what can most logically be concluded about Emperor Quian Long's point of view? Choice Feedback A. The British are too crude to adapt the more refined Chinese customs. B. The British need to behave more like the Chinese in the future. C. The Chinese people can learn many things from British culture. D. British culture should be highly valued by all nations.

D

Read the following excerpt from Emperor Quian Long's Letter to King George III in England in 1793: In consideration of the fact that your Ambassador and his deputy have come a long way with your memorial and tribute, I have shown them high favour and have allowed them to be introduced into my presence. Based on this excerpt, what can most logically be concluded about Emperor Quian Long's point of view? Choice Feedback A. The Ambassador is a worthy opponent. B. Britain is equal to China. C. Britain had sent too many troops. D. China is a superior nation

A

Read the following excerpt from Franklin Delano Roosevelt's Address to the Nation: The attack yesterday on the Hawaiian islands has caused severe damage to American naval and military forces. I regret to tell you that very many American lives have been lost. In addition, American ships have been reported torpedoed on the high seas between San Francisco and Honolulu. Yesterday, the Japanese government also launched an attack against Malaya. Last night, Japanese forces attacked Hong Kong. Last night, Japanese forces attacked Guam. Which sentence most accurately evaluates the rhetoric of Franklin Delano Roosevelt's speech? Choice Feedback A. The president uses parallelism to emphasize how widespread the attacks were. B. The president relies on repetition to ensure that his listeners do not forget the names of locations. C. The president uses incendiary language to evoke anger from the American listeners. D. The president relies on sarcasm to make Japan's invasion seem less than it was.

B

Read the following excerpt from Joseph Chamberlain's speech, "The True Conception of Empire," given in 1906: Here also the sense of possession has given place to a different sentiment—the sense of obligation. We feel now that our rule over these territories can only be justified if we can show that it adds to the happiness and prosperity of the people, and I maintain that our rule does, and has, brought security and peace and comparative prosperity to countries that never knew these blessings before. Which phrase from the excerpt most clearly shows an imperialistic attitude? Choice Feedback A. sense of obligation B. never knew these blessings C. comparative prosperity D. a different sentiment

A

Read the following excerpt from Joseph Chamberlain's speech, "The True Conception of Empire," given in 1906: We began to be, and we ultimately became, a great imperial Power in the eighteenth century, but, during the greater part of that time, the Colonies were regarded, not only by us, but by every European Power that possessed them, as possessions valuable in proportion to the pecuniary advantage which they brought to the mother country, which, under that order of ideas, was not truly a mother at all . . . Which phrase from the excerpt most clearly shows an imperialistic attitude? Choice Feedback A. possessions valuable in proportion B. under that order of ideas C. not truly a mother at all D. we ultimately became

B

Read the following excerpt from Joseph Chamberlain's speech, "The True Conception of Empire," given in 1906: [I]n almost every instance in which the rule of the queen has been established and the great Pax Britannica has been enforced, there has come with it greater security to life and property, and a material improvement in the condition of the bulk of the population. No doubt, in the first instance, when these conquests have been made, there has been bloodshed, there has been loss of life among the native populations, loss of still more precious lives among those who have been sent out to bring these countries into some kind of disciplined order . . . Which phrase from the excerpt most clearly shows an imperialistic attitude? Choice Feedback A. almost every instance B. loss of still more precious lives C. there has been bloodshed D. rule of the queen

C

Read the following excerpt from Mary Shelley's Frankenstein in which the monster tells his story: "You are in the wrong," replied the fiend; "and, instead of threatening, I consent to reason with you. I am malicious because I am miserable; am I not shunned and hated by all mankind? You, my creator, would tear me to pieces, and triumph; remember that, and tell me why I should pity man more than man pities me? . . ." Which phrase from the passage best supports the theme that monstrous appearances can be deceiving? Choice Feedback A. am I not shunned B. hated by all mankind C. consent to reason D. I am malicious

B

Read the following excerpt from Mary Shelley's Frankenstein in which the monster tells his story: I demand a creature of another sex, but as hideous as myself; the gratification is small, but it is all that I can receive, and it shall content me. It is true, we shall be monsters, cut off from all the world; but on that account we shall be more attached to one another. Our lives will not be happy, but they will be harmless, and free from the misery I now feel. Which phrase from the passage best supports the theme that monstrous appearances can be deceiving? Choice Feedback A. hideous as myself B. harmless, and free C. we shall be monsters D. demand a creature

A

Read the following excerpt from Mary Shelley's Frankenstein in which the monster tells his story: [I]t was a portrait of a most lovely woman. In spite of my malignity it softened and attracted me. For a few moments I gazed with delight on her dark eyes, fringed by deep lashes, and her lovely lips; but presently my rage returned: I remembered that I was forever deprived of the delights that such beautiful creatures could bestow . . . Which phrase from the passage best supports the theme that monstrous appearances can be deceiving? Choice Feedback A. it softened and attracted me B. fringed by deep lashes C. beautiful creatures could bestow D. portrait of a most lovely woman

A

Read the following excerpt from Mary Shelley's Frankenstein, which is narrated by Dr. Frankenstein: But I am a blasted tree; the bolt has entered my soul: and I felt then that I should survive to exhibit, what I shall soon cease to be, — a miserable spectacle of wrecked humanity, pitiable to others, and abhorrent to myself. Which phrase from the passage best supports the theme that humans possess the ability to be monstrous? Choice Feedback A. spectacle of wrecked humanity B. bolt has entered my soul C. shall soon cease to be D. pitiable to others

A

Read the following excerpt from Mary Shelley's Frankenstein, which is narrated by Dr. Frankenstein: Yet, at the idea that the fiend should live and be triumphant, my rage and vengeance returned, and, like a mighty tide, overwhelmed every other feeling. After a slight repose, during which the spirits of the dead hovered round, and instigated me to toil and revenge, I prepared for my journey. Which phrase from the passage best supports the theme that humans possess the ability to be monstrous? Choice Feedback A. my rage and vengeance B. spirits of the dead hovered C. that the fiend should live D. instigated me to toil

A

Read the following excerpt from Mary Shelley's Frankenstein, which is narrated by Dr. Frankenstein: [B]ut now, for the first time the wickedness of my promise burst upon me; I shuddered to think that future ages might curse me as their pest, whose selfishness had not hesitated to buy its own peace at the price, perhaps, of the existence of the whole human race. Which phrase from the passage best supports the theme that humans possess the ability to be monstrous? Choice Feedback A. wickedness of my promise B. peace at the price C. whole human race D. shuddered to think

B

Read the following excerpt from T. N. Makharji's A Visit to Europe: Perhaps, no symptom being visible in my external appearance of the cannibalistic tendencies of my heart, or owing probably to the notion that I must have by that time got over my partiality for human flesh, or knowing at least that the place was safe enough against any treacherous spring which I might take into my head to make upon them, or owing to whatever other cause, the party gradually grew bolder, began to talk in whispers and actually tried to attract my attention . . . Based on this excerpt, what can most logically be concluded about Makharji's point of view? Choice Feedback A.. People are shy when they meet new people. B. Europeans do not see Indians as civilized people C. He prefers cannibalism over vegetarianism. D. He has to be careful in Europe to avoid getting arrested.

A

Read the following excerpt from T. N. Makharji's A Visit to Europe: They were as much astonished to see the Indians produce works of art with the aid of rude apparatus they themselves had discarded long ago, as a Hindu would be to see a chimpanzee officiating as a priest in a funeral ceremony and reading out Sanskrit texts from a palm leaf book spread before him. Based on this excerpt, what can most logically be concluded about Makharji's point of view? Choice Feedback A. Europeans believe Indians are an ignorant, uncultured people. B. Indians are not producing fine art because they use primitive tools. C. It is amazing that Indians are so artistic. D. Europeans have a limited experience with interpreting Sanskrit.

A

Read the following excerpt from T. N. Makharji's A Visit to Europe: You could tell any amount of stories on this subject [of the number of wives Indians have] without exciting the slightest suspicion. Once, one of our number told a pretty waitress—"I am awfully pleased with you, and I want to marry you. Will you accept the fortieth wifeship in my household which became vacant just before I left my country?" She asked—"How many wives have you altogether?" "Two hundred and fifty, the usual number," was the ready answer. Based on this excerpt, what can most logically be concluded about Makharji's point of view? Choice Feedback A. Europeans' stereotypes of Indians make them gullible. B. Indian people like to travel the world. C. Storytelling provides amusement as part of the Indian culture. D. The number of wives Indian men have is astonishing.

B

Read the following excerpt from Winston Churchill's "Their Finest Hour" speech presented during World War II: I am not reciting these facts for the purpose of recrimination. That I judge to be utterly futile and even harmful. We cannot afford it. . . . Now I put all this aside. I put it on the shelf, from which the historians, when they have time, will select their documents to tell their stories. We have to think of the future and not of the past. What is most likely the intended effect of beginning the speech this way? Choice Feedback A.. The audience will realize that assigning blame is for children. B. The audience will be convinced to focus on hope for the future. The C. The audience will conclude that Churchill doesn't want to discuss failure. D. The audience will view Churchill as a proud leader, unwilling to accept failure.

D

Read the following excerpt from Winston Churchill's "Their Finest Hour" speech presented during World War II: Of this, I am quite sure, that if we open a quarrel between the past and the present, we shall find that we have lost the future. Therefore, I cannot accept the drawing of any distinctions between Members of the present Government. It was formed at a moment of crisis in order to unite all the Parties and all sections of opinion. It has received the almost unanimous support of both Houses of Parliament. Its Members are going to stand together, and, subject to the authority of the House of Commons, we are going to govern the country and fight the war. What is most likely the intended effect of beginning the speech this way? Choice Feedback A.. The audience will realize that past mistakes of the government led to the war. B. The audience will be convinced that Churchill will accept criticism. C. The audience will decide to be subject to authority of the House of Commons. D. The audience will perceive the government as strong and united. Global Incorrect Feedback The correct answer is: The audience will perceive the government as strong and united.

A

Read the following excerpt from Winston Churchill's "Their Finest Hour" speech presented during World War II: The disastrous military events which have happened during the past fortnight have not come to me with any sense of surprise. Indeed, I indicated a fortnight ago as clearly as I could to the House that the worst possibilities were open; and I made it perfectly clear then that whatever happened in France would make no difference to the resolve of Britain and the British Empire to fight on, "if necessary for years, if necessary alone." What is most likely the intended effect of beginning the speech this way? Choice Feedback A. The audience will gain a clear understanding of the wisdom and resolve of their leader to see the conflict to an end. B. The audience will be convinced that the battle in France was unfortunate, but it really does not matter to Churchill. C. The audience will blame Churchill for knowing how bad things were for two weeks and not taking action to prevent it. D. The audience will decide Churchill is stubbornly fastened to his decision, even if people will be harmed.

A

Read the following excerpt from the closing of Winston Churchill's "Their Finest Hour" speech presented during World War II: But if we fail, then the whole world, including the United States, including all that we have known and cared for, will sink into the abyss of a new Dark Age made more sinister, and perhaps more protracted, by the lights of perverted science. Let us therefore brace ourselves to our duties, and so bear ourselves that, if the British Empire and its Commonwealth last for a thousand years, men will still say, "This was their finest hour." What is most likely the intended effect of ending the speech this way? Choice Feedback A. The audience will see that it must have courage because the stakes are high for the whole world. B. The audience will despair because it will realize that it is living in a new Dark Age. C. The audience will believe that the perversion of science is the culprit behind the world's woes. D. The audience will decide to protest the war efforts, bringing the dark abyss to a conclusion.

A

Read the following excerpt from the closing of Winston Churchill's "Their Finest Hour" speech presented during World War II: What General Weygand called the Battle of France is over. I expect that the Battle of Britain is about to begin. Upon this battle depends the survival of Christian civilization. Upon it depends our own British life, and the long continuity of our institutions and our Empire. The whole fury and might of the enemy must very soon be turned on us. Hitler knows that he will have to break us in this Island or lose the war. What is most likely the intended effect of ending the speech this way? Choice Feedback A. The audience will be prepared for the worst for the sake of life and country. B. The audience will be provoked to fear by the idea of the evil Hitler will unleash upon them. C. The audience will be discouraged by the idea of its civilization being destroyed. D. The audience will be convinced to fight a guerrilla war against Germany should it invade Great Britain.

A

Read the following excerpt from the closing of Winston Churchill's "Their Finest Hour" speech presented during World War II: [W]e in this Island and in the British Empire will never lose our sense of comradeship with the French people. If we are now called upon to endure what they have been suffering, we shall emulate their courage, and if final victory rewards our toils they shall share the gains, aye, and freedom shall be restored to all. What is most likely the intended effect of ending the speech this way? Choice Feedback A. The audience members will be inspired to hope for victory and protect their allies. B. The audience members will understand their duty to endure suffering like the French did. C. The audience members will realize that they are comrades with all Europeans. D. The audience members will view their country's defeats as necessary evils.

A

Read the following excerpts from two speeches given by Franklin D. Roosevelt at the onset of America's entrance into World War II in 1941. The first one was given to Congress and the second one was a radio address to the nation: 1. With confidence in our armed forces, with the unbounding determination of our people, we will gain the inevitable triumph — so help us God. 2. We are now in the midst of a war, not for conquest, not for vengeance, but for a world in which this nation, and all that this nation represents, will be safe for our children. Which most accurately describes a major difference in the emphasis between the two speeches? Choice Feedback A. Unlike Speech #2, Speech #1 promises perseverance to gain victory. B. Both speeches use hyperbole to inflate the need for war with Japan. C. Unlike Speech #2, Speech #1 relies on FDR's integrity to persuade Americans. D. Unlike Speech #1, Speech #2 stirs up fear to muster support.

A

Read the following excerpts from two speeches given by Franklin D. Roosevelt at the onset of America's entrance into World War II in 1941. The first speech was given to Congress and the second was a radio address to the nation: 1. I believe that I interpret the will of the Congress and of the people when I assert that we will not only defend ourselves to the uttermost, but will make it very certain that this form of treachery shall never again endanger us. 2. We are now in this war. We are all in it — all the way. Every single man, woman and child is a partner in the most tremendous undertaking of our American history. We must share together the bad news and the good news, the defeats and the victories — the changing fortunes of war. Which most accurately describes a major difference in the emphasis between the two speeches? Choice Feedback A. Unlike Speech #1, Speech #2 emphasizes the need to unify for a common cause B. Both speeches employ logos to make the point that war is inevitable. C. Unlike Speech #1, Speech #2 uses language that blames Japan for acting deceitfully. D. Unlike Speech #2, Speech #1 uses pathos to highlight the need to fight back.

A

Read the following passage from Joseph Stalin's radio broadcast shortly after Germany invaded the USSR in 1941: Collective farmers must drive off all their cattle and turn over their grain to the safekeeping of the State authorities for transportation to the rear. All valuable property including non-ferrous metals, grain and fuel which cannot be withdrawn must without fail be destroyed. In areas occupied by the enemy, Guerilla units, mounted and foot, must be formed, diversionist groups must be organized . . . Which argument is most clearly developed by the rhetoric used in the passage? Choice Feedback A. Nothing of value will be overtaken by Germany. B. Germany's army doesn't have similar resources as Russia. C. Seniors and teenagers will be conscripted to service. D. Stalin is confident that German occupation is inevitable.

D

Read the following passage from Joseph Stalin's radio broadcast shortly after Germany invaded the USSR in 1941: Collective farmers must drive off all their cattle and turn over their grain to the safekeeping of the State authorities for transportation to the rear. All valuable property including non-ferrous metals, grain and fuel which cannot be withdrawn must without fail be destroyed. In areas occupied by the enemy, Guerilla units, mounted and foot, must be formed, diversionist groups must be organized . . . Which argument is most clearly developed by the rhetoric used in the passage? Choice Feedback A. Stalin is confident that German occupation is inevitable. B. Germany's army doesn't have similar resources as Russia. C. Seniors and teenagers will be conscripted to service. D. . Nothing of value will be overtaken by Germany.

A

Read the following passage from Joseph Stalin's radio broadcast shortly after Germany invaded the USSR in 1941: Such popular levies must be raised in every city which is in danger of an enemy invasion, all the working people must be roused to defend our freedom, our honor, our country — in our patriotic war against German fascism. In order to insure a rapid mobilization of all forces of the peoples of the USSR, and to repulse the enemy who treacherously attacked our country, a State Committee of Defense has been formed in whose hands the entire power of the State has been vested. Which argument is most clearly developed by the rhetoric used in the passage? Choice Feedback A. Russia will commandeer the nation's resources to ensure defense. B. Russia may be forced to yield its power to Germany in the near future. C. Russia is confident that its democratic government will overcome fascism. D. The working people of Russia have to be awakened from their peaceful slumber.

A

Read the following passage from Mary Shelley's Frankenstein: "The child still struggled and loaded me with epithets which carried despair to my heart: I grasped his throat to silence him and in a moment he lay dead at my feet. I gazed on my victim, and my heart swelled with exultation and hellish triumph: clapping my hands, I exclaimed, 'I too, can create desolation: my enemy is not impregnable; this death will carry despair to him, and a thousand other miseries shall torment and destroy him.' . . ." Which element of Gothic literature is most evident in this excerpt from Frankenstein? Choice Feedback A. Physical terror B. Psychological terror C. A grim setting D. A motif of death

B

Read the following passage from Mary Shelley's Frankenstein: After days and nights of incredible labor and fatigue, I succeeded in discovering the cause of generation and life; nay, more, I became myself capable of bestowing animation upon lifeless matter. The astonishment which I had at first experienced on this discovery soon gave place to delight and rapture. Which common notion from the historical context surrounding Frankenstein does this excerpt most clearly show? Choice Feedback A. Some doctors were unscrupulous, selling false medicinal remedies and experimenting at the risk of their patients. B. Scientists were willing to interfere with the natural order that was ordained by God, becoming god-like in their explorations C. Enlightenment thinking promoted learning and advanced education for the betterment of human life. D. The Romantics de-emphasized the influence of reason, and instead emphasized human emotion.

C

Read the following passage from Mary Shelley's Frankenstein: But these philosophers whose hands seem only made to dabble in dirt and their eyes to pore over the microscope or crucible, have indeed performed miracles. They penetrate into the recesses of nature, and show how she works in her hiding-places. They ascend into the heavens; they have discovered how the blood circulates, and the nature of the air we breathe. They have acquired new and almost unlimited powers . . . Which common notion from the historical context surrounding Frankenstein does this excerpt most clearly show? Choice Feedback A. The Industrial Revolution brought about the scientific method, leading to more scientific quests. B. Horror and suspense, haunted forests, and castles were common settings in the Gothic literature of the day. C. Scientists stepped into the domain of religion, presuming to discover and recreate the essence of life itself. D. The Romantic period brought about an appreciation for beauty in opposition to the intellectual quest for reason.

D

Read the following passage from Mary Shelley's Frankenstein: I had desired it with an ardor that far exceeded moderation; but now that I had finished, the beauty of the dream vanished, and breathless horror and disgust filled my heart, Unable to endure the aspect of the being I had created, I rushed out of the room, and continued a long time traversing my bed-chamber, unable to compose my mind to sleep. Which element of Gothic literature is most evident in this excerpt from Frankenstein? Choice Feedback A. Physical terror B. A dark, ruinous setting C. The specter of death D. Psychological terror

D

Read the following passage from Mary Shelley's Frankenstein: I saw how the fine form of man was degraded and wasted; . . . I saw how the worm inherited the wonders of the eye and brain. Which element of Gothic literature is most evident in this excerpt from Frankenstein? Choice Feedback A. A spooky setting B. Physical terror C. Psychological terror D. An emphasis on death and decay

A

Read the following passage: It is no secret that we live in the greatest country in the world. Every nation need only look to us to see that ours is a free and happy society. It is now time to spread the ideals of our nation, through diplomacy or by force, so that everyone may share in our freedom and happiness. Which phrase from the passage most clearly shows an imperialistic attitude? Choice Feedback A. spread the ideals B. It is no secret C. freedom and happiness D. everyone may share

C

Read the following passage: We had traveled across the sea to the new land, hoping to find an untamed wilderness. However, we found an inhabited land that needed discipline. We pacified the population and made it fit for civilization. For this we thank our great explorers and warriors. Which phrase from the passage most clearly shows an imperialistic attitude? Choice Feedback A. inhabited land B. untamed wilderness C. fit for civilization D. explorers and warriors

A

Read this excerpt from Franklin Delano Roosevelt's "Fireside Chat 19: On the War with Japan": In 1940, Italy attacked France and later Greece — without warning. And this year, in 1941, the Axis Powers attacked Yugoslavia and Greece and they dominated the Balkans — without warning. In 1941, also, Hitler invaded Russia — without warning. And now Japan has attacked Malaya and Thailand — and the United States — without warning. Which sentence most accurately evaluates the rhetoric of Franklin Delano Roosevelt's speech? Choice Feedback A. The president repeats the phrase "without warning" to emphasize that these events were unexpected. B. The president relies on repetition to ensure that his listeners remember the order the events occurred C. The president uses the words "attacked" and "invaded" to exaggerate the significance of the events. D. The president refers to when each event occurred to emphasize his credibility with the listeners.

Which blood vessel carry blood directly from the heart via the abdominal aorta?

Renal arteries

Which of the statements restates the main idea of the passage

Right or left-handedness doesn't really affect the outcomes of people lives.

Which of the following is a logical conclusion based on the passage of walking in the graveyard ?

Steven becomes most frightened by the atmosphere of the graveyard at night

What is the meaning of "uncanny" as used in the 4th paragraph of the passage

Surreal

The part of the human excretory system most responsible for maintaining normal body temperature is the: a. kidney b. bladder c. liver d. sweat glands

Sweat glands

apocrine glands

Sweat glands in the groin and underarm areas that secrete thicker sweat, that produce odor when come in contact with bacteria on the skin

Which of the following is occurs when the thyroid gland releases calcitonin?

The building of the bones occurs

Chimney Swifts (chimney swifts are small, slender birds...) Which of the following can be inferred from information in the passage?

The chimney swift's adaptability was severely hindered around 1970

Which of the following can be inferred from the information in the passage?

The chimney swift's adaptability was severely hindered around 1970s.

Which of the following occurs if the epiglottis does not function properly? a. The client is unable to recall recent events. b. The client coughs because food goes into the trachea. c. The client cannot produce hormones from the pancreas. d. The client becomes reproductively sterile.

The client coughs because food goes into the trachea.

During inhalation, what changes occur in the diaphragm? The diaphragm muscles relax, making it dome-shaped. The diaphragm muscles contract or tighten and the diaphragm moves downward. The abdominal muscles contract, making the diaphragm move upward. The diaphragm muscles contract or tighten and move upward.

The diaphragm muscles contract or tighten and the diaphragm moves downward

circulatory system (cardiovascular system)

The heart pumping blood through the arteries, capillaries,and veins provides the means of transporting substances throughout the body

A part of which body system controls fluid loss, protects deep tissues, and synthesizes vitamin D? a. The skeletal system b. The muscular system c. The lymphatic system d. The integumentary system

The integumentary system

Today I had lunch with my friend. I called her yesterday and asked her to bring our high school yearbook to the restaurant. She forgot to bring it. After lunch, she promised to bring it to me in the morning.Based on the passage above, which of the following events occurred first?

The narrator asks his friend to bring the yearbook

Which of the following occurs when the diaphragm contracts?

The thoracic cavity increases in volume

A ball is tied to a pole with a rope and is moving in a circle with constant speed. Which of these statements about the ball is true? Force is acting away from the center of circle. Force is acting at a tangent to the circle. There is force acting toward the center of circle. There is no acceleration.

There is a force acting toward the center

Which of these make up a molecule of fat? three molecules of fatty acid and one molecule of glycerol one carbon, one hydrogen, and one oxygen molecule one glycerol molecule and three stearic acid molecules three molecules of glycerol and one molecule of fatty acid

Three molecules of fatty acid and one molecule of glycerol

How does gas exchange occur?

Through Diffusion

what is alter?

To change something such as cutting and hemming clothes.

stimulus question #2 of 2 which of the following is the purpose of the editorial downtown fremont?

To engage residents in the discussion about increasing business revenue

White house solar panels Instead of following a chronological sequence, the author discusses Obama's reinstallation of solar panels first. Which of the following is the purpose of this order of the events?

To engage the reader with the current event.

Instead of following a chronological sequence, the author discusses obama's reinstallation of solar panels first .Which purpose of this order of events

To engage the reader's political stance

Instead of following chronological sequence, the author discusses Obama's reinstallation of solar panels first. Which of the following is the purpose of this order of the events?

To engage the reader's political stance

Which of the following is the purpose of the passage?

To inform

Which of the following was a key reason that scientists began working on better treatments for malaria?

To protect soldiers in combat

Which of these statements regarding the gastrointestinal tract is false? Villi present in the large intestine absorb water and mineral salts. The large intestine secretes no enzymes. The ileum absorbs B12 and bile salts. Villi contain lacteals for the absorption of fats.

Villi present in the large intestine absorb water and mineral salts

Which vitamin is synthesized within the human body? vitamin A vitamin E vitamin C vitamin D

Vitamin D

A

Which of the following excerpts from Joseph Stalin's 1941 radio broadcast would best be classified as incendiary? Choice Feedback A. [Germany] has lost politically by exposing herself in the eyes of the entire world as a bloodthirsty aggressor. B. [T]he non-aggression pact between Germany and the USSR is precisely such a pact. C. [T]his peace treaty does not infringe . . . on the territorial integrity, independence and honor of the peace-loving State. D. All our work must be immediately reconstructed on a war footing . . .

A

Which of the following two themes are most developed in Frankenstein? Choice Feedback A. Selfish pursuit and revenge as a consequence of grief B. The pursuit of excellence and the power of truth C. The vanity of pride and the power of relationships D. The curse of isolationism and the beauty of nature

B

Which of the following two themes are most developed in Frankenstein? Choice Feedback A. The meaninglessness of beauty and the joy of friendship B. The pursuit of technology and the loss of humanity C. The cost of selfishness and the need for gentleness D. The discovery of nature and the power of happiness

12) The insulating sheath that facilitates rapid movement of the action potential down the axon is known as which of the following? a) Myelin b) Myosin c) Actin d) Sarcomere

a

18) Which of the following distinguishes endocrine and exocrine glands from one another? a) The presence or absence of ducts b) The Biochemistry of their secretions c) The presence or absence of nerve innervation d) The effect their secretions have on target cells

a

A pudding recipe for 50 people calls for 4 cups of sugar. Each bag of sugar contains 6 cups. How many bags of sugar will be needed to make this recipe for 300 people? a. 4 bags b. 13 bags c. 24 bags d. 8 bags

a

Covert 27 to a Roman numeral a. XXVII b. IIIXXX c. CCCXII d. MMXVV

a

In which of the following places can you find the thesis statement of an essay? a. Introduction b. Body c. Reference d. Conclusion

a

Which of the following is punctuated correctly? a. While some people love cats, others prefer dogs. b. While some people love cats: others prefer dogs. c. While some people love cats. others prefer dogs. d. While some people love cats others prefer dogs.

a

Which of the following statements about a paragraph's topic sentence is true? a. It should be as general as possible b. It should be as detailed as possible c. It summarizes the main idea of the paragraph d. It summarizes the main idea of the essay

a

Which of the following words means the act of withdrawing, reducing, or abolishing military forces or weapons? a. Disarmament b. Subversion c. Antiestablishment d. Noncombatant

a

the smaller the diameter of the efferent arteriole resist:

a steady floe of blood increasing the blood pressure within the glomerulus

Threshold

a strip of wood, metal, or stone forming the bottom of a doorway and crossed in entering a house or room.

Skeletal muscle contraction is stimulated by the neurotransmitter ______, which is released by nerves at the neuromuscular junction a) acetylcholine b)nitric oxide c) dopamine d) glycine

a) acetylcholine

Which of the following hormones is produced in the ovaries? a) estrogen b) vasopressin c) prolactin d) oxytocine

a) estrogen

Which of the following food sources provides the majority of the urea that is filtered from the blood via the kidneys? a) meat and some plant products, such as legumes b) fruits, such as pineapple and pear c) lard and various oils, such as vegetables and canola d) grain products, such as breads and pasta

a) meat and some plant products, such as legumes

Which of the following statements correctly matches a body system with its function? a) the digestive system facilitates absorption of substances into the bloodstream b) the respiratory system transports oxygenated blood throughout the lungs and body c) The excitatory system prevents harmful substances from entering the body d) The endocrine system transports hormones from place to place within the body

a) the digestive system facilitates absorption of substance into the blood stream

During photosynthesis, which two compounds are combined to create the output of glucose and oxygen? A Carbon dioxide and water. B Carbon dioxide and bicarbonate. C Bicarbonate and water. D Carbon dioxide and multiple alkaline substances.

a. carbon dioxide and water

________ is a two-step process, and the four cells that result each have half the normal genetic material of the original cell. a. meiosis b. mitosis c.replication d. codification

a. meiosis

which of the atoms described below is an anion? a. an atom with 9 protons, 10 neutrons, and 10 electrons b. an atom with 9 protons, 10 neutrons, and 9 electrons c. an atom with 9 protons, 11 neutrons, and 9 electrons d. an atom with 11 protons, 12 neutrons, and 10 electrons

an atom with 9 protons, 10 neutrons, and 10 electrons

common linking verbs include:

appear, be become, feel, grow, look, seem, smell, sound, and taste.

16) A researcher is gathering large amounts of data about patients who have a common disease and the effectiveness of various medications in comparison to a placebo. Which of the following types of graphs is the best way for the researcher to present the data? a) Flow chart b) Bar graph c) Pie Chart d) Line Graph

b

22) Which of the following structures is severed and tied off as part of a vasectomy procedure? a) Seminal vesicle b) Vas Deferens c) Epididymis d) Testes

b

26) In which of the following structures of the respiratory system does the majority of gas exchange between the systems occur? a) Bronchi b) Alveoli c) Bronchioles d) Trachea

b

A farmer needs to buy fertilizer for his farmland. The instructions on the by of fertilizer say 49.5 pounds of fertilizer is needed for 1 acre of land. The farmer buys 2,000 pounds of fertilizer. Which of the following is the approximate number of acres he can cover with the fertilizer he bought? a. 1,000 b. 40 c. 125 d. 300

b

All the 1,020 students at a local high school must enroll in a foreign language course. If 269 students are enrolled in French, 663 are enrolled in Spanish, And the remaining students are enrolled in German, which of the following is the percent of students enrolled in German? (Round the answer to the nearest whole number.) a. 91% b. 9% c. 65% d. 26%

b

Seven more than twice a number is equal to twenty-one. Which of the following equations expresses the phrase above? a. 7(2x)=21 b. 7+2x=21 c. 2(x+7)=21 d. 2(7)+x=21

b

The class is nervous about _______ first exam? a. They're b. Its c. Its' d. It's

b

Which of the following options is spelled correctly? a. Accommedate b. Accommodate c. Accommadate d. Accommidate

b

Which of the following sentences contains formal, rather than informal language? a. The clerk could've contacted that manager if she wanted to b. If the situation becomes tense, contact your supervisor c. I'm okay with doing it your way, as long as you take responsibility d. The last chapter was way too long. I could barely finish it.

b

Which of the following sentences contains informal, rather than formal language? a. Bright colors appear to capture the attention of children b. The lead engineer found out the cause of a lot of the problems. c. The article examined the history of the women's movement d. The new program accumulates the data at the end of the month

b

Which of the following conditions is related to an abnormality in the function of red blood cells? a) arrhythmia b) Anemia c) hypertension d) leukemia

b) anemia

a blood sample taken from a patient fighting an infection is expected to show a) reduced white blood cell count b)elevated white blood cell count c) elevated red blood cell count d)reduced red blood cell count

b) elevated white blood cell count

Which of the following connects bones to muscles? a)ligaments b) tendons c) synovium d) suture

b) tendons

Which of the answer choices provided best defines the following statement? For a given mass and constant temperature, an inverse relationship exists between the volume and pressure of a gas? a. Charles' Law b. Boyle's Law c. Stefan-Boltzmann Law d. Ideal Gas Law

b. Boyle's Law

In which part of the heart does oxygenated blood flow out of? A Right ventricle B Left ventricle C Right atrium D Left atrium

b. Left ventricle

Which of the following is a possible cause of herniation of discs? A hepatitis B whiplash C plantar fasciitis D ulcer

b. whiplash

What is NOT excreted through the integumentary system?

blood

What is hypothermia?

body temperature drops below normal, the integumentary system prevents heat loss.

25) After crossing two tall pea plants, a scientist notes that approximately ¾ are tall, while ¼ are short. The scient .... the offspring' a) Wild types b) karyotypes c) phenotypes d) genotypes

c

4(2x-6)=10x-6 Solve for x above. Which of the following is correct? a. 5 b. -7 c. -9 d. 10

c

Although Americans are dedicated to dieting and healthy eating, they often succumb to the temptation for snacks and desserts. Which of the following is a synonym for "succumb" as used in the sentence above? a. Ignore b. Deny c. Surrender d. Supplant

c

How many layers of liquid will the finished drink have? a. Four b. Two c. Three d. Five

c

Numerous studies have confirmed that a brisk walk 30 minutes each day is adequate exercise for most healthy adults. Two of the most beneficial aspects are increased metabolism and joint flexibility. Equally important, however, are the mental benefits. A daily walk provides a chance to decompress from the stresses of the work day. A change in scenery and fresh air can brighten your mood. Which of the following sentences would be the best topic sentence for the paragraph? a. Exercise does not have to be difficult; consider walking b. One of the easiest ways to lose inches is to take a walk on a daily basis c. There are physical and psychological reasons for a daily walk d. If you want to get in shape the first step is to get moving

c

Singing is my favorite activity. "singing" functions in which of the following parts of speech in the sentence above? a. Preposition b. Adjective c. Noun d. Adverb

c

The girl wore headphones to avoid the ceaseless jabber of the other students on the bus. Which of the following is the meaning of the word "ceaseless" as used in the sentence above? a. Temporary b. Periodic c. Nonstop d. Recurrent

c

The graceful dancer moved across the stage as the music began to play. Which of the following is the meaning of the suffix "-ful" as used in the sentence above? a. State of b. Worthy c. Characterized by d. Able

c

The number of bicycles (x) in a shop is 4 more than 3 times the number of snowmobiles (y). Which of the following equations symbolically represents the number of bicycles in terms of the number of snowmobiles? (assessment) a. 3x=y+4 b. X+4=3y c. X=3y+4 d. X=4y+3

c

To determine the insurance premium of a car, an insurance company considers the following determinants: the age of the car, the model of the car, and the mileage of the car. Which of the following is the dependent variable? a. Mileage b. Age c. Insurance premium d. Model

c

Which of the following correctly rounds the value below to the hundredths place? 1,782.96471 a. 1,800 b. 1,782.96400 c. 1,782.96 d. 1,792.97

c

Which of the following demonstrates proper subject-verb agreement? a. The boys and girl in our club is in the café'. b. The boys and girl visits my house. c. The boy or the girls come to school late. d. The boys or the girls sings in the choir.

c

Which of the following is the correct conversion of 2 ¾ yards into inches? a. 11 inches b. 33 inches c. 99 inches d. 75 inches

c

Which of the following is the solution to the equation -9(k-17)=54? a. -11 b. 23 c. 11 d. -23

c

235 U . ---> . X + 4HE 92 . 2 which of the following is "x" in the reaction above? a) 239 Pu 94 b) 231 Pu 94 c) 231 Th 90

c) 231 Th 90

which of the following explains what happens to the circulatory system during hyperventilation? a) oxygen levels increase, causing an increase in blood pH b) Oxygen levels decrease, causing a decrease in blood pH c) Carbon dioxide levels decrease, causing an increase in blood pH d) Carbon dioxide levels increase, causing a decrease in blood pH

c) Carbon dioxide levels decrease, causing an increase in blood pH

Which of the following glands is exclusively found in males? a) adrenal b)parathyroid c)Cowper's d) pituitary

c) Cowper's

if an intensive property in independent of the amount of the material being measured, which of the following is an example of intensive property? a)Length b)mass c)volume d) density

c) Density

which of the following glands is found on top of the kidneys? a) thyroid b) pituitary c) adrenal d) pancreas

c) adrenal

Which of the following best summaries the passage? ( CHIMNEY SWIFTS)

chimney swifts rely entirely on masonry chimneys to survive during their yearly migrations

While playing basketball, a player injures her Achilles tendon, which types tissue was injured?

connective

When MgCO3 is dissolved in water, it separates into MG2+ and CO3^2- ions. What type of bond exists between carbon (C) and oxygen (O) a. neutron b. ionic c. proton d. covalent

covalent

Which of these is not a part of the central nervous system? tract (bundle of nerve fibers/axons) brain spinal cord cranial nerves

cranial nerves

provides a practical means of warming the body via integumenary thermoregulation:

cutaneous vasoconstriction

The glycolysis phase of cellular respiration takes place in;

cytosol

17) An equal mass of which of the following states of matter could completely fill either a 100 mL or 500 mL container? a) solid b) mixture c) liquid d) gas

d

36) Which of the following describes how RNA and DNA are used in mitosis? a) Moves vesicles to the middle of the cell to form the cell plate b) Forms spindle fibers that attach to the chromosome c) Produces energy for cell division d) Leads to synthesis of proteins that assist in cell replication

d

According to the prefix "pre-", which of the following words defines preliminary? a. Concluding b. Repeating c. Anticipatory d. Introductory

d

As a true bibliophile, she spent a great deal of time in the library. Which of the defines the word "bibliophile" as used in the sentence above? a. A university professor b. A collector of rare maps c. A research scientist d. A lover of books

d

Which of the following sentences demonstrates proper subject-verb agreement? a. The teams meets in their respective locker rooms before the football game. b. The cheerleading group are going to the football game. c. The students in our band is going to the football game. d. The group of fans is making a banner for the football game.

d

Which of the following sentences is written correctly? a. The lizard has the ability to altar it's coloration to blend in with the foliage. b. The lizard has the ability to altar its coloration to blend in with the foliage. c. The lizard has the ability to alter it's coloration to blend in with the foliage. d. The lizard has the ability to alter its coloration to blend in with the foliage.

d

1. preservation of genes of best - adapted offspring 2. Overproduction of offspring with variation in traits 3. Differential survival in the face of environmental challenges Which of the following is the sequence in which these steps take place? a) 3,2,1 b) 1,3,2 c) 2,1,3 d) 2,3,1

d) 2,3,1

A researcher decides to repeat an experiment and reassess results first established in the 1930's about why plants grow toward light. Which of the following is the best reason for conduction this experiment? a)plant behavior could have modified due to evolutionary processes since the experiments were first done b)new breeds of plants discovered since the 1930's have not yet been shown to have to same light responses c)shifts in average temperatures produced by global warming may affect plants in ways that are not yet understood d)equipment and methods developed since the 1930's may produce better data and provide mechanistic detail.

d) equipment and methods developed since the 1930's may produce better developed data and provide mechanistic detail

The cause of cell differentiation is that different cells a) contain different DNA b) have different chromosomes c) contain similar protein information d) express alternate genetic information

d) express alternate genetic information

Which of the following glands produces melatonin, the hormones that regulates sleep? a) thyroid b)hypothalamus c)pituitary d) pineal

d) pineal

The lungs are covered by a membrane called the a) pericardium b) periosteum c) symphysis d) pleura

d) pleura

Pancreatic cells have an abundance of _______ to meet the protein production demands of the cell a) nuclei b)mitochondria c)lysosomes d) rough endoplasmic reticulum

d) rough endoplasmic reticulum

Which of the following contains the testes? a) epididymis b) Mesentery b)prostate d) scrotum

d) scrotum

Which of the following layers of the epidermis contains cells that provide protection against UV radiation? a) stratum corneum b) stratum lucidum c) stratum spinosum d) stratum basale

d) stratum basale

Two weeks ago, I went fishing on the Kaw River with my grandfather. We found a secluded spot where the fish were abundant. After a while, several police officers appeared on the opposite shore. Grandfather hollered across and discovered that they were looking for a stolen car that had been reportedly dumped in the Kaw. I suddenly had a large tug on my line, but I could not pull the fish in without breaking my line. A police officer in scuba gear volunteered to help. After several moments, the diver surfaced and said, "You have a whopper, but he is in one of those old cars at the bottom. I would have better luck helping you, but every time I approach him, he rolls up the window!" Which of the following sentences in the text could support the inference that it is common practice to dispose of old vehicles in the Kaw River? a. "We found a secluded spot where the fish were abundant." b. "After a while, several police officers appeared on the opposite shore." c. "Grandfather hollered across and discovered that they were looking for a stolen car that had been reportedly dumped in the Kaw." d. "After several moments, the diver surfaced and said, 'You have a whopper, but he is in one of those old cars at the bottom."

d. "After several moments, the diver surfaced and said, 'You have a whopper, but he is in one of those old cars at the bottom."

If you have 200 mL of a 60% sodium chloride solution, how much NaCl is dissolved in the solution? A . 20 g B 50 g C 80 g D 120 g

d. 120g

How many carbon dioxide molecules are required to balance the following reaction: 4C6H12O6 + 24O2 → _CO2 + 24H2O + Energy A . 4 B 8 C 16 D 24

d. 24

Which of the following sources contains copyright information for a given book? a. Encyclopedia b. Table of contents c. Index d. Library online catalog

d. Library online catalog

Despite our entreaties, her objection to our plan STANDS. Define stands a. Resists a hostile action b. Halts or comes to a stop c. Rises to an upright position d. Remains in effect

d. Remains in effect

which layer of the skin contains oil glands? a. epidermis b. dermis c. hypodermis d. subcutaneous

dermis

which of the follow layers of skin contains glands that secrete sodium? a. epidermis b. hypodermis c. dermis d. exodermis

dermis

Mitosis is a process of cell division that results in two identical daughter cells from a single parent. The number of chromosomes remains the same as the parent, 46. This a ________ cell.

diploid (46)

The glittering lights in the garden were meant to "entrance" the guests as they arrived. What is the meaning of the homograph "entrance" as used in the sentence above?

enchant (glittering lights in the garden)

rypsinogen, an enzyme secreted by the pancreas, is activated into trypsin in the duodenum by which enzyme? pepsin enterokinase hydrochloric acid protease

enterokinase

Which of the following is a true statement about enzymes? a. enzymes are proteins that increase the rate of biological reactions. b. enzymes are form by joining two nitrogenous bases with a peptide bond. c. enzymes catalyze reactions by binding to may different types of molecules d. enzymes are fibrous proteins made of amino acids.

enzymes are proteins that increase the rate of biological reactions.

an increase in what will lead to a decrease in the amount of oxygen in the blood?

exercise rate

The ventricles collects and:

expel blood from the heart.

Steroids don't contain what?

fatty acids but are hydrophobic like other lipids.

Lone bones are largely part of the appendicular skeleton and includes:

femur, tibia, fibula, metatarsals, phalanges, humerus, radius, and metacarpals

which kind of protein fiber is NOT found in connective tissue?

fibrin

When a shell reaches its capacity, additional electron start to?

fill in the next shell

Your current salary is $50,000. Next year you will get a 4% increase in your salary. How much more money will you make next year?

first, 4% divided by 100 to give decimal form. which equals 0.04. second, 0.14 multiply $50,000 equals 2,000 answer= $2,000

Suppose scientists have invented a new substance that can be compressed to fit into smaller containers or expanded to fill large spaces. In what state of matter must this mew substance exist? A. Solid B. Liquid C. Gas D. Crystal

gas

which hormone is produce by the stomach that induces stomach secretion?

gastrin

Which of the following does NOT contain protein? a. cell membrane b. muscle tissue c. genetic material d. skeleton

genetic material why? it contains nucleic acids and not amino acids

Dark hair is a dominant trait. if D symbolizes the allele for dark hair, which option shows the heterozygous genotype and its matching phenotype? a. genotype: Dd; phenotype: dark hair b. genotype: Dd; phenotype: lighter hair c. genotype: DD; phenotype: dark hair d. genotype: DD; phenotype: lighter hair

genotype: Dd; phenotype: dark hair

what is a descriptive passage?

give physical details or provide unique characteristics

which structure is the passageway that allows air to travel pass the vocal cords:

glottis

what is an alpha cell? a. nephron b. oxytocin c. insulin d. glucagon

glucagon

adrenal cortex gland hormones:

glucocorticoids aldosterone

some polysaccharides are compose only of what?

glucose, which includes cellulose, starch, and glycogen.

Triglycerides contain 3 fatty-acid chains that are bound to what kind of molecule?

glycerol

when the barriers are breached, immune cells from the:

innate and adaptive arms target pathogens for destruction.

what is beta cell? a. insulin b. glucagon c. goblet cells d. mitochondria

insulin

which hormone decreases glucose level?

insulin beta cell

A small chemical messenger, to signal to nearby cells the presence of a foreign pathogen & activate innate defenses in cells are:

interferons

According to the prefix "pre-", which of the following words defines "preliminary"

introductory

The first paragraph in the passage on chimney swifts includes this description: "...large colonies of hundreds, or even thousands, of individuals form tornado-like funnels." Why does the author include this in the passage?

it provide the reader engagged with image.

Which of these statements is true for boiling, but not true for evaporation? It takes place at a specific temperature. Gas forms at the surface of the liquid. The warmer the liquid is, the faster the process. The larger the surface area of liquid, the faster the process.

it takes place at a specific temperature

What is conjunctions?

join words, phrases, or clauses, and they show the connection between the joined pieces.

Hair and nails are comprised of which of the following? a. collagen b. keratin c. melanin d. elastin

keratin

where is vitamin K and water stored? a. duodenum b. liver c. large intestine d. stomach

largest intestine

what is the myocardium? a. layer of the heart that tells bp b. layer of the heart that contains striated muscle fibers for contraction. c. the area between the lungs and the heart. d. a cavity in the heart.

layer of the heart that contains striated muscle fibers for contraction.

which heart ventricle is he most muscular? a. left b. right

left ventricle

A clear plasma-like fluid high in WBCs, & filtered through one of several lymph nodes:

lymph

Which group of major parts and organs make up the immune system? a. lymphatic system, spleen, tonsils, thymus, and bone marrow. b. Brain, spinal cord, and nerve cells. c. Heart, veins, arteries, and capillaries d. Nose, trachea, bronchial tubes, lungs, alveolus , and diaphragm.

lymphatic system, spleen, tonsils, thymus, and bone marrow.

1.Success on the job entails a number of skills; it encompasses strong analytical, communication. Which of the following is a synonym for 'entails' as used in the sentence above? A. Necessitates B. Interjects C. Accommodates D. Solicits

necessitate

what is anion?

negative charge ion. the number of electrons must be greater than protons.

Where does blood filtration and absorption occurs in the kidney?

nephron

Information about the body's condition or environment is sent to the brain along sensory neurons:

obtained using the 5 senses

What is a simple sentence?

one independent clause with no subordinate clauses.

Food moves in the gastrointestinal tract by which of these processes? peristalsis of the circular and longitudinal muscles voluntary waves of contraction peristalsis of the longitudinal muscles only. peristalsis of the voluntary muscles.

peristalsis of the circular and longitudinal muscles

Social media is what kind of passage?

persuasive

which gland regulates sleep? a. melatonin b. pineal c. pituitary d. thymus

pineal

which gland is the stimulating gland?

pituitary gland

"I had to wrack my brain and plum the depths of my resources the foul consequences of my previous vice." Which of the following homophones should be used to correct the sentence above? a. fowl b. plumb c. rack d. vise

plumb

what are larger chains of carbohydrates called?

polysaccharides

what is cation?

positive charge ion. more protons than electron

The cerebellum coordinates voluntary movements such as:

posture, balance, coordination, and speech, resulting in smooth and balanced muscular activity.

Brown the meat

process of partially cooking the surface of meat to help remove excessive fat

ceruminous glands

produce ear wax (cerumen) although the produce sebum they do not primarily supply hair shafts and external skin with oily secretions

sebaceous glands

produce sebum (oil) which supplies hair shafts and skin with oily secretions

If someone is lying belly-down, their positioning is said to be what? lateral prone supine posterior

prone

Where is hemoglobin synthesized

red blood cells

54) NAD+ 2e' + H2O NADH What kind of reaction is this?

redox

Function of thyroxin?

regulated metabolized and temperature.

A student is conducting an experiment to study the trait of fur color in rabbits. Black fur is dominant, and white fur is recessive. She crosses a black male with a white female and is surprised when one of the offspring has white fur. What fact would account for this result? a. the white female is heterozygous b. the black male is heterozygous c. the offspring with white fur is heterozygous d. all of the offspring are homozygous

the black male is heterozygous

Which structure is not considered a part of the rib cage? the lumbar vertebrae costal cartilage sternum the thoracic vertebrae

the lumbar vertebrae

Chimney swifts Which of the following best summarizes the passage?

the number of Chimney Swifts in the world has been slowly decreasing since 1970s

which gland is called the master gland:

the pituitary gland

what is dehydration synthesis?

the removal of a water molecule to create a covalent bond.

A coach shows up to football practice one day and all the equipment is missing, before he notifies the principal he tries to figure out the most plausible reason for this.

the rival team from the next town over is playing a prank

Past perfect:

the second action happened in the past. The first action came before the second

where does the cerebellum receives information from?

the sensory systems, the spinal cord, and other parts of the brain and then regulates motor movements.

The hairless parts of the body include:

the soles of the feet, lips, inner labia, and glans penis.

Which of the following is the purpose of editorial about downtown Fremont?

to determine how residents feel about adding more parking downtown.

what is autosomal dominant?

trait or disorder can be passed down through families.

For a protein to be expressed, the DNA must first be ________to mRNA.

transcribed

What are the 3 main categories of lipids?

triglycerides, phospholipids, and steroids.

What is dihybrid cross?

two individuals who are heterozygous for both genes.

According to the prefix "sub", which of the following words defines "subterranean"?

underground

Hydrophillic?

water loving phosphate group

plural first person pronoun?

we, us, our, ours

"Having been discovered, Rover looked up at his owner with puppy-dog eyes" Which words/ phrases are misplaced in this sentence? a. having been discovered b. rover looked up c. at his owner d. with puppy dog eyes

with puppy dog eyes

Having been discovered, Jack looked up at his owner with puppy-dog eyes. which of the following groups of word is misplaced? 1) been discovered 2) Jack looked up 3) with puppy-dog eyes 4) his owner

with puppy-dog eyes

Your watering can with the large spout is just perfect for the garden. Which is a prepositional phrase?

with the large spout

what is preposition?

word placed before a noun or pronoun that shows the relationship between an object and another word in the sentence.

The distinguished lexicologist had been with the university for many years. In the sentence, the prefix lexi- indicates that the researcher studies which of the following? A. literature B. words C. plants D. rocks

words

The number of bicycles (x) in shops is 4 or more than 3 times the number of snowmobiles (y). Which of the following equation symbolically represents the number of bicycles in terms of snow mobile?

x=3y+4

what is corpus luteum?

yellow hormone-secreting body in the female reproductive system. It is formed in an ovary at the site of a follicle, or sac, that has matured and released its ovum, or egg, in the process known as ovulation.

The breakdown of food by enzymes for absorption is called?

Enzymatic digestion

Liquid to gas is?

Evaporation

respiratory system

Exchange gases with the outside environment,bringing oxygen in and letting carbon dioxide out

Chimney swifts The first paragraph in the passage on chimney swifts includes this description "...large colonies of hundreds, or even thousands of individuals form tornado-like funnels". Why does the author include this in the passage?

It provides the reader with an engaging image

Which sentence from a paragraph about sun-dried tomatoes is a transition sentence?

Once you have decided what tomatoes to use, there are three different methods for drying them.

Which of the following summarizes the text?

Diverse presidential philosophies have affected the White House energy resource use

example of future perfect:

When she comes for the supplies, I will have walked to the store.

Microvilli in the small intestine secretes what?

They are brush border enzymes.

Deoxygenated blood flow from:

The right ventricle through the pulmonary arteries to the lungs, where the blood picks up oxygen.

Examples of coordinating conjunction in a sentence:

The rock was small, but it was heavy. She drove in the night, and he drove in the day.

Which of the following sentences demonstrates proper subject-verb agreement? a.The boys or the girls sings in the choir b. The boys and girls visits my house c. The boys and girl in our club is in the cafe d. The boy or girls come to school late

d

what kind of passage is the Chili recipe?

expository - read carefully the Recipe and the questions. one of the answers is to rinse the beans first.

In a hypertonic solution, water flows through aquaporins embedded in the plasma membrane of the cell. This type of transport is best known as?

facility diffusion

what are lipids?

fats

what does the enzyme lipase break down?

fats into fatty acid and glycerol

which of the following words is spelled correctly? Which of following words is spelled correctly? A. Foreign B Vien C. Hieght D. Reveive

foreign

Which of the following is the result of meiosis beginning with one human cell? a. two cells, each with 23 chromosomes b. two cells, each with 46 chromosomes c. four cells, each with 23 chromosomes d. four cells, each with 46 chromosomes

four cells, each with 23 chromosomes

what is imperative sentence?

gives an order or command.

what is a declarative sentence:

gives information for makes a statement.

what does cellulose act as in the human body?

helps move material through the digestive tract.

There are three insects that are being compared under a microscope. As a scientist, you decide that measuring them would be an important part of recording their data. Which unit of measurement would best for this situation? a. centimeters b. meters c. micrometers d. kilometers

micrometers

which of the following words most closely matches the meaning of the word "infamous" as used in the passage

notorious

What is the function of the placenta? a. regulate the mother's uterus b. regulate the mother's heart rate during pregnancy c. nourish the baby and gets rid of wastes d. nourish the mother and gets rid of wastes from the developing baby

nourish the baby and gets rid of wastes

what kinds of food are high in starch?

potatoes, grains, rices, and bean products.

Which of the following features of an alamanc would be most useful to a person trying to decide whether to throw a fourth of july party over summer?

predict the weather using the alamanc or sunrise times

what are stages of writing process?

prewriting, writing, revision

John followed the doctors _______ to get more rest. how to spell

recommendation

What is the proper spelling below? a. reccmmendation b. rekomendation c. recommendaion d. recommendation

recommendation

function of testosterone:

regulates sperm cell production and secondary sex characteristics.

Pancreas produces

insulin and glucagon

Which of the following sentences is correctly punctuated? a. A working student's best friend is her phone b. The ponys' saddle is so pretty c. Several car's bumpers were crushed after the collision d. Tom's and Anne house is beautiful

a

what is the largest artery in the body?

aorta

what muscles cause hair to stand up and causes goosebumps? a. arrector pili b. diaphragm c. myosin d. triceps

arrector pili

what carries blood flow away from the heart? a. oxygen b. aorta c. veins d. arteries

arteries

Strong, elastic vessels adapted to the high pressure of blood as it leaves the heart:

arteries.

The spleen functions are part of the _____ system. Which of the following completes the sentence above? A digestive B lymphatic C endocrine D circulatory

b. lymphatic

Which group of major parts and organs make up the immune system? a. brain, spinal cord, and nerve cells b. lymphatic system, spleen, tonsils, thymus, and bone marrow c. nose, trachea, bronchial tubes, lungs, alveolus, and diaphragm d. heart, veins, arteries, and capillaries

b. lymphatic system, spleen, tonsils, thymus, and bone marrow

Prokaryotic and eukaryotic cells are similar in having which of the following? a. Presence of a nucleus. b. Membrane-bound organelles c. Protein-studded DNA d. Integral membrane proteins in the plasma membrane

b. membrane - bound organelles

When does the nuclear division of somatic cells take place during cellular reproduction? a. cytokinesis b. mitosis c. interphase d.meiosis

b. mitosis

norepinephrine hormone:

stimulates the flight response in "fight or flight" response

What are the components of the respiratory system?

-Nose -Throat -Lungs

Which of the following is unaffected by a cellular differentiation? a. DNA b. size c. shape d. metabolism

DNA

Healthcare spending per capita

Eritrea

nonpathogenic microbes that compete for resources and thus prevent pathogen occupancy is:

Flora

singular third person pronoun?

He, she, it, him, her, his, hers, its

What is the characteristic of the human organisms

Human consume rather than manufacture nutrients molecules (heterotrophic) and human genes are encoded in DNA

What does Villa do?

Increase surface area absorption.

This is produced by the stomach and required for vit B12 absorption:

Intrinsic factor

The process of cephalization allows for which of the following?

Localizing sensory organs

Which unit is used to indicate mass?

Mg

Which of the following is the main idea of the third paragraph?

Sleepwalking is a common phenomenon.

Which of the following is the sphincter muscle that regulates the movement of chyme into the small intestine?

Pyloric

The teacher announces to the class _____ which of the following completes the sentence above?

"The English test is scheduled for Wednesday."

Which of the following sentences correctly follows the rules of punctuation regarding dialogue?

"Which of you," she asked, "is going to the store with me?" I said that I would go if we would be back by six.

Each month an employee withholds $468 from her gross paycheck of $2,350. Which of the following will be her net annual salary?

$22584

which of following is the solution to the equation -9(k-17)=-54

-11

Given the unbalanced equation __KClO3 > ___KCl + ___O2 What is the coefficient of O2 in the balanced equation? a. 1 b. 2 c. 3 d. 4

C: A balanced equation contains the same number of atoms of each element on both sides of the written reaction: 2KClO3 > 2KCl + 3O2.

Which group of organisms help prevent the accumulation of organic wastes in nature? a. rabbits b. mosses c. bacteria d. ferns

C: Bacteria comprise the decomposer portion of the food chain.

8. which of the following sentence is punctuated correctly? A. A carpenter must use certain tools - hammers, saws, chisels to finish a job. B. A carpenter must use certain tools hammers, saws, chisels - to finish a job. C. A carpenter must use certain tools, hammers, saws, chisels, to finish a job. D. A carpenter must use certain tools - hammers, saws, chisels - to finish a job.

D.

Which formula represents a carbohydrate? a. C6H6 b. C2H5OH c. CHCOOH d. C6H12O6

D: A carbohydrate is made up of carbon, hydrogen and oxygen with the ration of hydrogen to oxygen 2:1.

A cloud chamber is a device used for? a. studying the formation of weather systems b. recreating the atmosphere of the primitive earth c. treating a respiratory infection d. studying charged particles

D: A cloud chamber makes the paths of ionizing radiation rays visible. the device reveals the paths of high-speed charged particles.

In pea plants, tallness is dominant over shortness. If 25% of one generation of pea plants is short, what were the probable genotypes of the parents? a. tt x tt b. Tt x tt c. TT x Tt d. Tt x Tt

D: A cross in which both of the parents are heterozygotes yields 25% TT, 50% Tt, and 25% tt.

Which of the following statements best describes how pressure and volume are affected as a sample of ideal gas is heated in a rigid, sealed container? a. neither pressure nor volume will increase b. pressure will remain constant, and volume will increase c. both pressure and volume will decrease d. pressure will increase, and volume remains constant

D: A rigid container implies no change in volume. According to Gay-Lussac's law, at constant volume an increase in temperature cause and increase in pressure and vice versa.

Which of these values could be the ph of an acid rain sample? a. 12.0 b. 8.0 c. 7.0 d. 4.0

D: Acid rain has an acidic pH. A substance with a pH between 0 and 7 is an acid.

Which of these formulas represents an alcohol? a. C2H5OC2H5 b. C3H8 c. CH3COOH d. C2H5OH

D: An alcohol is usually characterized structurally by the presence of an OH group attached to a hydrocarbon.

Which organisms are classified as protists? a. protozoa, bacteria, and worms b. fungi, ferns, and mollusks c. monkeys, apes, and man d. protozoa and algae

D: Bacteria are in the kingdom Monera. Worms, mollusks, humans, apes and monkeys are in the kingdom Animalia. Ferns are in the kingdom Plantae, and fungus is in the kingdom Fungi. Protozoa and algae are Protists..

An unknown substance is boiled with Benedict's solution The brick red color that results indicates the presence of? a. protein b. acid c. starch d. glucose

D: Benedict's solution is used to indicate the presence of monosaccharides, glucose.

Blue litmus paper will turn red when placed in a solution having which of the following ph values? a. 14 b. 12 c. 7 d. 3

D: Blue litmus paper turns red when placed in an acid solution. A ph below 7 indicates an acid.

The diameter of the low-power field of a certain microscope is 1.6 mm. The diameter of a cell that is half the diameter of the field is? a. 500 um b. 800 um c. 1000 um d. 1600 um

D: One half of the field = 800 um: 1 mm = 1,000 um, and 1.6 mm = 1,600 um.

In pea plants, plant height and seed color obey the law of independent assortment. If seeds are hybrid for both of these traits, how will their offspring most likely appear? a. all tall, with green seeds b. all short, some with yellow and some with green seeds c. some tall and some short, with all green seeds d. some tall and some short, with green or yellow seeds

D: The law of independent assortment states that traits are passed onto offspring independently of each other. Thus, a seed may contain either of the height alleles and either of the color alleles.

The entry and exit of materials in the animal cell are regulated by which of these cell organelles? a. nucleolus b. cell wall c. endoplasmic reticulum d. plasma membrane

D: The plasma mermbrasne, which is make of carbohydrates, lipids and proteins, is selectively permaeable and regulates traffic into and out of a cell.

Which glands are sweat glands?

Eccrine and sudoriferous glands

Convert each decimal to a fraction. A. 0.516 B. 1.07

A. 516/1000 B. 107/100

Which of the following sentences in the text could support the inference that it is common practice to dispose of old vehicles in the Kaw River?

"After several moments, the diver surfaced and said, 'You have a whopper, but he is in one of those old cars at the bottom.'

what is androgen (sex):

Testosterone (adrenal cortex)

What is malleability?

The ability of a material to be manipulated onto different shapes.

based on the passage which of the following is true?(CHIMNEY SWIFTS)

The arthur provides facts to support the main idea

If the outer shell is exactly at its full capacity of electrons:

The atom will be stable and will not react easily

which nervous system is cerebellum part of?

central nervous system

What connects the vagina to the outside?

cervix

what connects the vagina to the uterus?

cervix

what is chitin?

contains glucose and amino acids and forms the exterior skeleton of arthropods.

Of the following, which is not a type of chemical bond? A ionic B covalent C polar D magnetic

d. magnetic

Which molecule groups is chemically digested for the first time in the stomach?

protein

A ring-shaped muscle that tightens to separate the stomach from the small intestine:

pyloric sphincter

During hyperthemia the sudoriferous glands deliver:

water to the skin in the form of sweat, which evaporates from the skin's surface. as the water evaporates, it absorbs and carries away heat with it, cooling the body.

Is " Measles" singular?

yes

which of the following percentages is equivalent to 8/5 A. 1.6% B. 160% C. 0.625 D. 62.5%

160% How? 8/5=1.6 1.6 X 100= 160%

Which of the following fractional representations is equivalent to 0.897?

897/1000

Osteoporosis result from what?

A decline of osteoblast activity while osteoclast activity continues at expected level.

what is oxidation reaction?

A reaction involving a combination with oxygen.

What are shells?

All the electrons across certain subshells that share the same principle energy level.

Where would a student look to find out more information about historical weather predictions and weather patterns?

Almanac

Example of a complex sentence:

Although he had the flu, Harry went to work. Marcia got married after she finished college.

Salts from blood

Are removed by the kidney

Which of these temperatures could not occur? a. -0 C b. -10 K c. 2,000 F d. 104 C

B: Zero Kelvin is absolute zero, which is the lowest possible temperate; therefor, -10 Kelvin does not exist.

Prefix for "mal":

Bad

What is the yellow pigment of bile?

Billirubin

Which part of the brain is responsible for speech?

Broca area

"The man was harshly punished for his egregious mistake." Based on the sentence above, what is the correct meaning of the word "egregious"? A. misunderstood, or false B. simple, rudimentary C. terrible, exceedingly bad D. tragic, unforeseen

C

What organ secretes the thick fluid that nourishes and protects sperm? A. Vas deferens B. Epidemys C. Prostate gland D. Seminal vessivles

C

We could have__ lunch with them Which of the following options correctly completes the sentence above? A. ate B. eat C. eaten D. had ate

C. eaten

the surgeon _____ operated on the patient that has a greater reputation A. whom B. that C. who D. which

C. who

which of the following tasks is appropriate during the final stages of the writing process?

Checking for punctuation errors

What is the function of the synapse?

Chemical transmission involves release of chemical messengers known as neurotransmitters. Neurotransmitters carry information from the pre-synaptic—sending—neuron to the post-synaptic—receiving—cell

The cerebrum is divided into 2 halves, connect by nerve fibers that form?

Corpus callosum

density formula:

Density=mass/volume

What is the name of the process in the lungs by which oxygen is transported from the air to the blood? a. osmosis b. diffusion c. dissipation d. reverse osmosis

Diffusion

what are the two types of sweat glands:

Eccrine and apocrine glands.

what is the uterus connected to:

Fallopian tubes, the cervix, and via cervix to the vagina.

glucose is stored as

Glycogen in the liver and muscles of mammals

Which of the following bones supports the tongue and is the only bone in the body not anchored to other bones? a. patella b. coccyx c. Hyoid d. scapula

Hyoid

macrophages cells

Invest and digest both no-self cells and dead cells

Which element is essential for normal production of thyroxine? A. Calcium and VIT D B. Phosphorus C. Magnesium D. Iodine

Iodine. Iodine is required by thyroid gland for the production of the hormone thyroxine.

soft palate

Is found in the back of the buccal cavity. It helps in the swallowing and prevents food entry into the nasal passages

The insulating sheath that facilitates the rapid movement of action potential down the axon is which of the following?

Myelin

Pulmonary artery is the blood vessel that contains the least:

Oxygenated blood

Subshell designation:

S=2 P=6 D=10 F=14

Which part of the skeletal system is made up of 126 bones in the following regions of: upper limbs, lower limbs, pelvic girdle, and pectoral (shoulder) girdle.

The appendicular skeleton

Cervix

The external opening to the uterus

Use the bibliography below to answer the question.Doe, Jane. " The Assassination of Archduke Franz Ferdinand. " Beginnings of the World War I. Web. Smith, John " Who Started the First World War?" History Questions. Web. Thomas, Jack. Poetry from the Onset of World War I. New York . 2000. PrintAbove is a sample of a bibliography from a student research paper. Based on the sources listed above, what is the likely subject of the paper?

The start of World War I

Which of the following statements from the passage provides evidence that hummingbirds have the "ability to amaze"?

Their wings beat faster than those of any other bird - from 60 to 70 times per second.

Which of the sentence indicates compare and contrast structure which was used in the passage of walking in the graveyard?

Things happening in the evening are different

Which of the following cannot exist in RNA? a. uracil b. thymine c. cytosine d. guanine

Thymine

Which gland of the endocrine system is responsible for controlling an individual's metabolism?

Thyroid gland

what are the thyroid gland hormones?

Thyroxin calcitonin

Which of the following sources contains copyright information for a given book?

Title page

"Which of the following is the purpose of this paragraph of Kaw RIver?

To entertain

Stimulus question 2 of 3 Which of the following is the purpose of the passage( Henry Tudor)

To inform

Which of the following structures is shared by the male urinary and genital systems?

Urethra

Which of the following is part of the small intestine? a) ileum b) colon c) Pylorus d) Cecum

a) ileum

The stomach is _____ to the small intestine. Which of the following correctly completes the sentence above? A lateral B superior C posterior D proximal

b. superior

Adrenal gland has the adrenal medulla that releases:

epinephrine and norepinephrine

ovaries hormones:

estrogen progesterone

what controls the hormone secreted by the pituitary gland? a. pineal b. thalamus c. hypothalamus d. thyroid

hypothalamus

The medication should be used to treat mild respiratory infection

if you have a respiratory problem, take this medication only if directed by doctor

Myelinated levels of thyroxine will do what to the heart?

increase heart rate

sudoriferous glands cooling the body:

increase sweating

Hair follicles are columns in the skin that have large concentration of:

keratin-producing cells. these cells form the hair while adjacent melanin-producing cell give hair it's pigmentation.

which organ excess solutes form the blood:

kidney

which organs remove nitrogenous waste from the body?

kidney

what is a fact?

something that can be proved to be true.

what are carbohydrates?

sugars or starches that contain carbon, oxygen, and hydrogen .

what is orbital?

the probability of the electron's location at any point in time

A patient in a sleep clinic has a prescription for a continuous positive airway pressure machine for sleep apnea. She would like a lifetime warranty. She is deciding between the four models listed in the chart above. Which of the following is the most economical buy for her situation?

Product A

When are the parasympathetic nerves active within the nervous system? a. when an individual experiences a strong emotion, such as fear or excitement b. When an individual feels pain or heat c. When an individual is either talking or walking d. When an individual is either resting or eating

When an individual is either resting or eating

A

Which of the following excerpts from Joseph Stalin's 1941 radio broadcast would best be classified as incendiary? Choice Feedback A. [O]ur country has come to death grips with its most malicious and most perfidious enemy — German fascism. B. [T]he chief virtue of the Soviet people must be courage, valor, fearlessness in struggle. C. We must organize all-round assistance to the Red Army, insure powerful reinforcements for its ranks. D. Collective farmers must drive off all their cattle and turn over their grain to the safe keeping of the state authorities.

13) Which of the following hormones increases the heart rate in response to stress? a) Epinephrine b) Testosterone c) Glutamate d) Insulin

a

27) Which of the following portions of the neuron transmits neurotransmitter? a) Axon b) Ganglion c) Dendrite d) Cell body

a

Having been discovered, Rover looked up at his owner with puppy-dog eyes. Which of the following group of words is misplaced in the sentence above? a. With puppy-dog eyes b. Been discovered c. His owner d. Rover looked up

a

what is the function of the medulla oblongata?

a portion of the hindbrain that controls autonomic functions such as breathing, digestion, heart and blood vessel function, swallowing, and sneezing.

Which of the following illustrates a negative feedback mechanism that occurs in the human body? a)the release of insulin from the pancreas in response to an increase in blood sugar levels b) the release of glucagon from the pancreas in response to an increase in blood- sugar levels c) the release of oxytocin resulting in uterine contractions as childbirth is underway d) chemical reactions that result in coagulation due to fibrin being released

a) the release of insulin from the pancreas in response to an increase in blood - sugar levels

What is the role of the urethra? a) to transport urine out of the body b) to transport the egg to the uterus c) to connect the kidneys to the bladder d) to connect the uterus to the outside of the body

a) to transport urine out of the body

Which of the following is a logical conclusion based on the passage? a. A hummingbird may attack other birds that come near a hummingbird feeder. b. Hummingbirds are important for their role in pollinating plants. c. Hummingbirds live long lives relative to other birds. d. A hummingbird will rarely leave its own territory.

a. A hummingbird may attack other birds that come near a hummingbird feeder.

Which of the following has a similar theme to the Hummingbirds passage? a. A nature documentary highlighting the many different varieties of peacock spider in Australia and their unique traits b. A editorial article about the declining population of Antarctic penguins that advocates for their conservation c. A novel about a young woman who takes in a wounded squirrel as a pet d. A television advertisement for an automatic fish feeder

a. A nature documentary highlighting the many different varieties of peacock spider in Australia and their unique traits

Which of the following correctly lists the cellular hierarchy from the simplest to the most complex structure? a.cell, tissue, organ, organ system, organism b.tissue, cell, organ, organ system, organism c.organism, organ system, organ, tissue, cell d. organ system, organism, organ, tissue, cell

a. cell, tissue, organ, organ system, organism

The adrenal glands are part of the a. endocrine system. b. respiratory system. c. immune system. d. emphatic system.

a. endocrine

Why does the human eye perceive a red colored dress as the color red? A The molecules of the dress do not absorb red light wavelengths. B The molecules of the dress absorb green and blue light wavelengths. C A red dress will primarily absorb red light wavelengths. D A red dress will not absorb light wavelengths from non-red colors.

a. the molecules of the dress do not absorb red light wavelengths

The opening of the rectum from which solid waste is expelled is?

anus

5) Which of the following is the primary function of red blood cells? a) Fighting infection in the body b) Carrying oxygen to other body cells c) Helping create blood clots d) Responding to antigens

b

9) Which of the following organic compounds are used to manufacture steroid hormones in the body? a) Polysaccharides b) Glucose c) Cholesterol d) Protein

b

A worker is paid $2,350 monthly and has $465 withheld from each paycheck. Which of the following is the annually net salary? a. $33,816 b. $22,584 c. $27,732 d. $28,200

b

Surely, he thought, I can't trust this enticing but dubious offer, although I could use the opportunity more that I'd like to admit. Which of the following parts of the speech is the word "although"? a. Interjection b. Conjunction c. Adverb d. Participle

b

Which of the following statements demonstrates proper pronoun-antecedent agreement? a. If people want to travel to the moon, you must get a job at a space technology station. b. If a person wants to travel to the moon, you must get a job at a space technology station. c. If a person wants to travel to the moon, she must get a job at a space technology station. d. If people want to travel to the moon, he must get a job at a space technology station.

b

Cystic Firbrosis is a homozygous recessive condition. A genetic counselor should advise concerned parents that, if each of them has a parent who has cystic fibrosis yet neither of the concerned parents is affected, the probability of their potential child having cystic fibrosis is a) 0% b) 25 % c) 50% d) 100%

b) 25%

Which of the following explains why damage to the spleen reduces the production of granulocytes? a)damage to the spleen reduces the production of granulocytes b) damage to the spleen causes massive blood loss c) damage to the spleen results in loss of clotting ability d) damage to the spleen results in decreased red blood cell production and maturation

b) damage to the spleen causes massive blood loss

what is monohybrid cross?

both parents are heterozygous for the given trait.

A farmer plants 4 apple trees for every 3 pecan trees in the orchard. She plants 2 peach trees for every 5 pecan trees. If the farmer plants 24 peach trees, how many apple trees are in the orchard? a. 48 apple trees b. 60 apple trees c. 80 apple trees d. 96 apple trees

c

To create a layered effect, at which of the following times should ice be added to this drink recipe? a. Before the chilled water b. Before the mint leaves c. After the orange juice d. After the sports drink

c

Which of the following sentences uses the correct capitalization? a. The Red cross is a Liaison for the families of Soldiers stationed in other countries, acting as a go-between for separated spouses, parents, and children. b. The Red Cross is a liaison for the families of Soldiers stationed in other countries, acting as a go-between for separated spouses, parents, and children. c. The Red Cross is a liaison for the families of soldiers stationed in other countries, acting as a go-between for separated spouses, parents, and children. d. The Red cross is a Liaison for the families of soldiers stationed in other countries, acting as a go-between for separated spouses, parents, and children.

c

Bile is secreted into the duodenum through the:

common bile duct

During hypothermia the blood vessels constrict to reduce:

the amount of blood flow through the skin, limiting the amount of heat lost to the external environment through radiation.

A class of 28 students has 13 boys and 15 girls. What is the ratio of boys to girls in the class

13:15

Simplify the expression above which of the following is correct?

4 33/40

Which of the following formulas best summarizes a single replacement reactions? a. A+B------ AB B. AB+C-----AC+B c. AB+CD---AD+BC d. AB----A+B

AB+C-----AC+B

4. In the race for student body government, both students were coequal. Their speeches each pointed to their records of service and academic excellence. Which of the following options is the meaning of 'coequal' used in the sentence above? A.The students gave speeches that were the same length. B.The students had similar accomplishments C.The students shared the same opinion on the issues. D.The students were running for the same office.

B.

In the classification of two organisms, which of the following is the best evidence of a close relationship? a. similar number and arrangement of bones b. similar sequence of amino acids in their proteins c. similar methods of sexual reproduction d. similar methods of cell respiration

B: A similarity in amino acids suggests a similarity in the structure of the DNA and thus a close relationship between the organisms.

The pitch of ta vibrating string depends on all of the following except? a. the length of the string b. the amplitude of the vibration c. the thickness of the string d. the frequency of the vibration

B: Observed pitch is independeent of the amplitude of the vibration. The amplitude is related to the loudness of the sound. The pitch of a string is affected by thickness, length, and frequency.

An organism with chloroplasts in its cells is probably. a. a heterotroph b. an autotroph c. an herbivore d. a primary consumer

B: Organisms that have chloroplasts in their cells carry on photosynthesis and are therefore autotrophs. Autotrophs make their own food.

The diffusion of water through a semipermeable membrane is called? a. pinocytosis b. osmosis c. active transport d. transpiration-tension

B: Osmosis is the diffusion of water from areas that contain a high concentration of water to area that contains a low concentration of water.

The compound H2SO4 is an example of? a. a salt b. a base c. an acid d. a halogen

C: H2SO4 is sulfuric acid. Acid may be defined as substances that release hydrogen ions in solution as the only positive ions. they may also be seen as protons donors. Bases release hydroxide ions, and salts have neither H+ or OH- ions. Halogens are elements.

A reading on the Celsius scale is 5. On the Fahrenheit scale, the reading would be closet to? a. 225 b. 30 c. 40 d. 95

C: the formula for converting temperature in Celsius to temperature in Fahrenheit is: T Fahrenheit = 9/5 x T Celsius + 32, therefore 5 degrees Celsius convert to 41 degrees Fahrenheit. 9/5 x 5 = 9 9 + 31 = 41F

while shopping, you find a shirt that is marked 30% off. If the regular price is $45, which of the following is the reduced price? a. $13.40 b. $15 c. $31.50 d. $33.75

$31.50 how? 30% change to decimal which gives you 0.30. then multiply by 45 gives you 13.50. last, 45-13.50= $31.50

What is the cerebellum?

- coordinates muscle control and balance - dense cluster of neurons located at the base of the brain.

adrenal cortex releases:

- glucocorticoids - mineralocorticoids

An employee discovers that 0.0025 of all products predicted at his factory contain defects, which of the following % does this represent?

0.25%

Which of the following is correct comparison of two rational numbers?

0.6>4/7

A rectangular block of a certain substance is 4 cm wide, 5 cm long, and 2 cm high. The block has a mass of 30 grams. What is the density of this substance? a. 0.75 g/cc b. 1.33 cc/g c. 7.5 g/cc d. 40 cc

0.75 g/cc why? The volume of the block is: 4cm X5cm X 2cm= 40cm3 (cc) The mass is given as 30 g. calculate density: 30g/40cc= 0.75g/1cc

A student rounds a number to 12.9. Which of the following possible value is the original number?

12.939

Which of the following is the number of 3/4 inch strips of caulk it will take to completely cover a length of 10 1/2 inches?

14

Mercury is a liquid at room temperature. It has a boiling point of approximately 357 degree Celsius and a latent heat of 70 cal/g. a. 1020 calories b. 13643 calories c. 14000 calories d. 14357 calories

14,000 calories why? The question asks for the amount of heat necessary to turn 200 g of mercury at 357 Celsius into a gas. You are told that 357 Celsius is mercury's boiling point and its latent heat is 70 cal/g. recall the formula for calculating the change in heat of a substance undergoes phase change: H=mL H= 200 g X 70 cal/g = 14,000 calories.

which of the following is 25% of 680 pounds? a. 1,700 pounds b. 510 pounds c. 68 pounds d. 170 pounds

170 pounds how? change 25% to a decimal by dividing by 100 which equal 0.25 then 0.25 multiply 680 will equal 170

X-5=32 solve the equation above. which of the following is correct? a. 22 b. 37 c. 27 d.42

37

Which of the following reactions describes iron being reduced? a. 4Fe + 3O2 ---- 2 Fe2O3 b. Fe + O ---- FeO c. 3Fe2O. + CO ---- 2 Fe3O4 + CO2 d. 2Fe2O3 + 3H2O ---- Fe(OH)3

3Fe2O. + CO ---- 2 Fe3O4 + CO2 Why? This reaction, iron(III) oxide is losing oxygen to CO, and iron is going from Fe(III) to Fe(II). Therefore, iron is reduce.

Which of the following percent is equivalent to 31/50?

62%

A driver uses 35.5 gallons of gas to drive from the Enanol canyon in the golden bridge, a distance of 787.25 miles. How many gallons of gas would the driver need from the golden gate bridge to the national park, a distance od 170.6 miles?

7.26 gallons of gas

Some Ca(OH)2 is dissolved in water. Which of the following could be the pH of the solution? a. 0 b. 4 c. 7 d. 8

8 why? Ca (OH)2 will separate into Ca+ and OH- ions in water. Because of the subscript 2, there will be twice as many OH- ions as Ca+ ions. Predict that the pH will be a number greater than 7.

What is single replacement reaction?

A chemical reaction when one element or molecular ion in a molecule is replaced by a different element or molecular ion

Which of the following best supports the author's point of view that hummingbirds are "marvels of speed, agility, and beauty"?

A citation from a zoologist

What is electron configuration?

A description of the shells and of all the electrons of an atom

What is a control group?

A group identical to the experimental group but without the altered independent variable.

Which of the following lists support the main idea of the passage?

A metal lined chimneys, chimney tower and reforestation efforts will ultimately results in growth of chimney swifts population.

Which of the following has a similar theme to the hummingbirds passage?

A nature documentary highlighting the many different varieties of peacock spider in Australia and their unique traits

The endocrine system is a collection of ductless glands that produce which of the following response? A. a quick initiating but prolonged response on other body organs. B. A slowly initiating and short response on other body systems C. A quickly initiating and short response on other body systems D. A slowly initiating but prolonged response on other body systems

A slowly initiating but prolonged response on other body systems

I before E:

Achieve belief Chief

Which of the following can be inferred about the element Br? a. Br has a strong affinity for free electrons. b. Br has more neutrons than protons. c. Br has more than one isotope. d. All of the above can be inferred from the information in the periodic table.

All of the above can be inferred from the information in the periodic table.

What attacks an antigen?

Antibodies

platelets in blood

Are removed by the spleen and liver

"Sarah and George traveled to Boston via the train." In this sentence, what part of speech is the word "traveled"? A. adverb phrase B. verb C. proper noun D. noun

B

3. Change this sentence from passive to active voice: "Over the course of the year, many mistakes were made." A. If mistakes were made, we are unaware of them. B. We made many mistakes over the course of the year. C. Many mistakes have been made throughout the year. D. Many mistakes have been made throughout the year.

B

The fan oscillated back and fourth, cooling down the entire room. What does the word oscillated mean? A. Blew B. Swung C. Turned D. Toppled

B

Cell responsible for the production of soluble antibodies?

B-cell (the production of antibodies by b-cell is part of the humoral response to antigens)

Which of the following antibody- secreting cells is triggered to proliferate upon vaccination? a. erythrocytes b. B=lymphocytes c. leukocytes d. T-lymphocytes

B-lymphocytes

The presence of which hormone would NOT cause an increase in water retention? A Renin B ANP C ADH D Aldosterone

B. ANP

Steven sneaked into the graveyard but immediately regretted his decision to do so. The sights and sounds of the graveyard at night were unfamiliar and frightening. The graveyard groundskeeper unexpectedly appeared and then walked away. Steven left the graveyard, making the decision to lose the wager with his friend.

Based on his daily routine, he assumed he could easily win the bet.

A survey of 500 teenagers was taken to see which

Basketball 120:football:190: hockey,90: soccer 80: other 20

example of past perfect:

Before I walked to the store, I had walked to the library

According to her parents, the book that I found was her's. She told me, however, that she didn't want it back. Which of the following is an error in the sentence above? A. Improper use of commas around "however" B. Improper use of a comma after the word "parents" C. Improper use of an apostrophe in the word "her's" D. Improper use of an apostrophe in the contration "didn't"

C.

Mustache: Which of the following statements can be inferred from the phrase, "However, it is in the last several centuries that mustaches have undergone a more creative transformation"? A. Individuals are creating more mustache styles than existed in ancient history. B. People are now more creative than people who lived during ancient history. C. Mustaches are easier to style in modern times than in ancient times. D. Facial hair is more prominent in modern times than it was in ancient times.

C. Mustaches are easier to style in modern times than in ancient times.

An organism that has jointed legs, an exoskeleton, and a specialized segmented body belongs to the phylum of? a. mollusks b. echinoderms c. arthropods d. chordates

C: An example of an arthropod is a lobster.

what is endothermic?

Consumes/ absorbs heat

Hypodermis (subcutaneous layer)

Contains subcutaneous fat as well as deeper blood vessels

Which of the following occurs to skeletal muscle as a result of acetylcholine released at the neuromuscular junction? a. relaxation b. peristalsis c. contraction d. eversion

Contraction

What does the brain function as?

Control center for the body and occupies the cranium, or skull.

13.which of the following sentences is grammatically correct? A. The student wrote too slow to finish the exam, but he has always been a slowly writer. B. The student wrote too slow to finish the exam, but he has always been a slow writer. C. The student wrote too slowly to finish the exam, but he has always been a slowly writer. D. The student wrote too slowly to finish the exam, but he has always been a slow writer.

D

The president invited Senator Smith to the White House for a meeting before preparing his address to the United States congress. Which of the following is an error in capitalization in the sentence above? A. The word "president" should be capitalized B. The word "Senator" should be lower case C. The word "House" should be lower case D. The word "congress" should be capitalized

D

Which of the following options best demonstrates sentence fluency? A. The city looked beautiful in that it was lit by the sun which I was walking down the street. B. Walking down the street in the sunshine, the city looked beautiful. C. The city looked beautiful as I was walking, lit by the sun, down the street. D. Walking down the street, I noticed how beautiful the city looked in the sunshine.

D

The respiratory system __________ oxygen and _________ carbon dioxide. a. Inhales' exhales b. Delivers; expels c. creates; absorbs d. exhales; inhales

Delivers; expels

stimulus 1 of 3which of the following actions should the cook take first when following this recipe ( Jo's Slow cooked chili)

Drain the beef and unions

What is the Law of Segregation?

Each inherited trait is defined by a gene pair. Parental genes are randomly separated to the sex cells so that sex cells contain only one gene of the pair. Offspring therefore inherit one genetic allele from each parent when sex cells unite in fertilization.

In addition to oxygen and carbon dioxide, the circulatory system is the primary delivery system for which?

Endocrine hormones

What is the name of the structure that prevents food from entering the airway? a. trachea b. esophagus c. diaphragm d. epiglottis

Epiglottis

Greek Theater is what kind of passage?

Expository

Stimulus 3 of 3 which of the following is the mode of the passage? (Jo's slow cooked chili

Expository

What kind of passage is King Henry VII?

Expository

stimulus 1 of 2 which of the following describes the writing in the passage? ( RUSSIAN BLUE CAT)

Expository

What kind of passage is Frida Kahlo? they will ask you to put her life in chronological order and the answer to that is

Expository/ informative - first, she got hit by a train next, she started drawing pictures of herself then, she married Diego

Estrogen

Female sex hormone production is affected by diseases that harm the uterus and ovaries

What is intermolecular forces?

Forces that cause particles to be attracted to one another or to repulse one another.

stimulus question #1 of 2 which of the following is a likely suggestion to the city manager's office will receive from the public?

Fremont should increase available parking spaces downtown

Editorial (Fremont has struggled over recent years...) Which of the following is a likely suggestion the city manager's office will receive from the public?

Fremont should increase available parking spaces downtown.

Free eats in dogs are controlled by dominant allele (F), and attached ears are controlled by the recessive allele (f). In addition, short dogs is due to a dominant allele(S), and long hair is due to a recessive allele (s). Which genotype of the gods will have free eats and short hair?

FsSs

What is the function of the mitochondria?

Generate ATP

example of transitive verb:

He plays the piano/The piano was played by him

The label appears on a bottle of over the counter cold medicine. Which of the following statements best summarizes the warning label?

If you have a respiratory problem, take this medication only if directed by a doctor.

Which of the following is a logical conclusion based on the passage?

Infection rates within a community may affect that community's understanding of diseases.

Which of the following information is included in the advertisement?

Information about a special deal for new customers

What function does the myelin sheath perform for a nerve cell?

Insulation, increase speed, and protection

Islet cells (in the pancreas):

Insulin Glucagon

Chili Recipe

It is Expository

Which organs remove nitrogenous waste from the body?

Kidney

Which of the following describes how RNA and DNA are used in mitosis?

Leads to the synthesis of proteins that assist in cell replication

Stimulus questions 3 of 6 which of the following lists supports the main idea of the text passage (CHIMNEY SWIFTS)

Logging and deforestation, changes in the ways we heat our buildings, and chimney capping have robbed the chimney swift of its preferred roosting cites

Which valve separates the left atrium and left ventricle:

Mitral Valve also known as bicuspid valve

A cross between two heterozygous F1 plants produces a ratio of 15:1 in the the F2 offspring. Which of the following best describes the ration? a. Modified monohybrid ratio b. Modified dihybrid ratio c. Normal monohybrid ration d. Normal dihybrid ration

Modified dihybrid ratio

What transport is gas exchange?

Passive Transport

What produces antibodies?

Plasma cells (differentiated from WBCs called B lymphocytes also called B cells)

According to the passage ,which of following events occurred second.

President Reagan took office in 1981

corpus luteum

Remnant of the grasfian follicle it secreted progesterone to prepare the uterus for the pregnancy

What are the 3 main function of the nervous system?

Sensory input, integration, and motor output

Semicolon is use between closely connected independent clauses that are not connected with a coordinating conjunction:

She is outside; we are inside. You are right; we should go with your plan.

What is a solution?

Substance dissolved into another

Which of the following sentences is considered common knowledge and would not require a citation within a text?

The Amazonian rainforest is disappearing

On each side of the body, the brachiocephalic veins are formed by the union of which two veins? external jugular veins and subclavian veins external jugular and internal jugular veins internal jugular veins and subclavian veins common carotid and subclavian veins

The internal jugular and subclavian veins

The cerebrum:

The largest part of the human brain. it is associated with higher brain function such as thought and action.

CITY POLICE SCHOOLS . Which of the following parts of the graph below best helps the reader interpret the information the author wants to convey?

The legend, which identifies how to determine pupil spending and income.

Which punctuation marks used incorrectly in the sentence: Otto, somewhat abruptly, got up put of his chair, and just like that; headed into the next room.

The semicolon after "that"

The law of conservation of mass states that ____. The total mass of all products of a chemical reaction is independent of the total mass of all the reactants. They are not related. The total mass of all products of a chemical reaction is more than the total mass of all the reactants. The total mass of all products of a chemical reaction is less than the total mass of all the reactants, because some mass coverts to energy during reaction. The total mass of all products of a chemical reaction is equal to the total mass of all the reactants.

The total mass of all products of a chemical reaction is equal to the total mass of all the reactants.

What is an ionic bond?

The transfer of electrons

This plan was developed based upon previous years' data, tracking trends in revenues and expenses. It includes a five-month track of sales, accounts receivable and payable, and inventor from a year-end benchmark. Which conclusion can be reached about the writer?

The writer is a professional in a business environment.

A student newspaper publishes an editorial supporting the expansion of campus recreation facility, which of the following most likely the intended audience of the editorial?

Undergraduates

Which of the following substances is excreted by sweat glands in response to the breakdown of proteins and the formation of ammonia?

Urea

Which of the following is the tube that carries both sperm and urine for release outside the body? a. ureter b. urethra c. vas deferens c. epididymis

Urethra

Which of the following structures is severed and tied off as part of a vasectomy procedure?

Vas deferns

B

Which aspect of Mary Shelley's Frankenstein most clearly indicates that Romanticism influenced the story? A. Natural philosophy is what drives Frankenstein's fate and future. B. Frankenstein looks upon his monstrous creation with horror and disgust. C. Frankenstein is undisturbed by reality as he seeks to discover the secret of life. D. Frankenstein compares his home life of nature, books, and love to his madness.

A

Which of the following two themes are most developed in Frankenstein? Choice Feedback A. The need for companionship and the consequences of revenge B. The sorrow of death and the joy of new life C. The price of greed and the determination of conquest D. Psychological torture and the desire for aesthetics

singular second person pronoun?

You, your, yours

2) While playing basketball, a player injures her Achilles tendon. Which of the following types of tissues was injured? a) Connective b) Epithelial c) Nervous d) Muscle

a

4) Which of the following is a response to the release of antidiuretic hormone (ADH)? a) Fluid volume in the body increase.

a

Which of the following is the first step to writing an informative essay? a. Selecting a topic b. Developing an outline c. Taking notes on the sources d. Writing a thesis statement

a

Which of the following is the number of 3/4 - inch strips of caulk it would take to completely cover a length of 10 ½ inches? (assessment) a. 14 b. 9 ¾ c. 7 7/8 d. 8

a

Which of the following pairs of fractions with a common denominator are equivalent to ¼ and 2/5? (assessment) a. 5/20, 8/20 b. 1/20, 2/20 c. 1/9, 2/9 d. 5/20, 4/20

a

Which of the following sentences correctly uses a compound sentence structure? a. He liked to cook at home, but she preferred to eat at restaurants b. After they played basketball all day, the girls were very tired c. Although he would have gone to the movie that his friends recommended, it started d. The professor enjoyed reading, cycling, and listening to jazz music

a

what is estrogen?

a female reproductive hormone, produced primarily by the ovaries in the non-pregnant woman. it promotes the maturation and release of an ovum in every menstrual cycle. it is also produced by the placenta during pregnancy.

example of codominance:

a flower with red and white alleles would have a red and white patches.

example of incomplete dominance:

a flower with red and white alleles would yield a pink flower.

The bulbourethral or Cowper's glands secrete:

a fluid into the urethra to neutralize the acidity in the urethra.

what is cystic fibrosis?

a genetic disorder that affects the lungs and other organs, characterized by difficulty breathing, coughing up septum, and lung infections.

What are the three parts of nucleotide?

a phosphate group, a five-carbon sugar ring, and a nitrogenous base.

The scrotum is:

a sac of skin and smooth muscle that houses the testes and keeps the testes at the proper temperature for spermatogenesis.

What does nucleotides also function as?

a source of energy in the form of ATP (adenosine triphosphate)

What is an enzyme?

a special category of proteins that catalyze specific biological reactions

Myosin has two important roles:

a structural one, as the building block for the thick filaments. a functional one, as the catalyst of the breakdown of ATP during contraction and in its interaction with actin as part of the force generator of muscle.

There are many tall tales from old world sailors and farmers, but some of them about weather predictions are true. Here are three general rules you can follow to predict precipitation. If there is a ring around the moon at night, rain or snow may be coming. Rain or snow may be imminent when puffy cumulus clouds change and begin to flatten. And finally, if a north wind shifts in a counter-clockwise manner, rain or snow could be on its way. If the weather forecast suggests that the current north wind will shift in a counter-clockwise direction to a southwesterly wind by afternoon, what might you predict? a. Rain b. Clear skies c. Tornado d. Drought

a. Rain

Which of the following statements rephrases the author's main argument about sleepwalking in the passage? a. Sleepwalking is a perplexing mystery that can be alarming to those who have the condition b. The media should cover more instances of sleepwalking to help with public understanding c. Sleepwalking is well understood by the by the scientific community d. There is no reason to believe that sleepwalking is dangerous

a. Sleepwalking is a perplexing mystery that can be alarming to those who have the condition

What is the meaning of "uncanny," as used in the 4th paragraph of the passage? a. Surreal b. Intimidating c. Disappointing d. Unexpected

a. Surreal

Which of the following best describes the author's purpose in the first sentence of the passage? a. To connect the passage to the reader's frame of reference b. To convince the reader of the need for social action c. To focus on disagreements about global diseases d. To introduce information about HIV/AIDS

a. To connect the passage to the reader's frame of reference

Which is NOT one of the four basic types of tissue? A parenchyma B epithelial C nervous D connective

a. parenchyma

Future perfect:

action that uses the pa sat and the future.

what is a bias?

against something or someone

which substances does not normally pass through the glomerular capsule?

albumin

Follicle-stimulating hormone (FSH), which is responsible for maturation of follicles in the ovary, is produced by which gland in the body? ovary corpus luteum posterior lobe of pituitary gland anterior lobe of pituitary gland

anterior lobe of pituitary gland

TSH is produced by which gland?

anterior pituitary

The systemic circuit includes:

aorta, blood vessels leading to the body tissues, as well as all the veins and venae cavae

compound-complex sentences:

at least 2 independent clauses and at least one dependent clauses.

where does fertilization occurs?

at the end of the Fallopian tube.

What is an isotopes?

atoms of the same element can have different numbers of neutrons.

What do T lymphocytes do?

attack viruses

in which of the following sources is this passage "Henry Vlll" most like to appear?

autobiography

28) (Another test?) Which of the following is generally expected to be true about birth rates in relation to death rates for a country with a growing population and high infant mortality? a) Equal to death rates b) Variable compared to death rates c) Lower than death rates d) Higher than death rates

b

30) Which of the following is the primary function of red blood cells? a) Fighting infection in the body b) Carry oxygen to other body cells c) Helping create blood clots d) Responding to antigens

b

A temperature gauge reads 95 F. which of the following is the correct conversion to degrees in Celsius? (Note: C=5/9[F-32]) a. 21 C b. 35 C c. 63 C d. 113 C

b

According to the prefix "in-" which of the following words defines incredulous? a. Sympathetic b. Disbelieving c. Convinced d. Weary

b

Five hundred forty-eight runners participated in a recent race. Twenty-six runners did not finish the race. Half of those who did finish completed the race in less than 2 hours. Which of the following is the number of runners who completed the race in less than 2 hours? a. 248 b. 261 c. 274 d. 522

b

John followed the doctor's ________ to get more rest. Which of the following options correctly completes the sentence above? a. Reccomendation b. Recommendation c. Reccommendation d. Recomendation

b

Which of the following is the meaning of the word "subalpine"? a. Under the snow b. Below the timberline c. Near the equator

b

|X|≥6 Which of the following graphs represents the inequality above?

b

A student made a culture of yeast cells in a test tube by mixing a pinch of dry yeast with 10 milliliters (mL) of distilled water. This test tube was labeled A. The student then took 1 mL of the resulting solution and placed it in test tube B and with 9 mL of distilled water. The student continued using this method of serial dilution through trial E as shown in the figure above. Examining the solution in test tube E under a microscope, the student found that there were 1,500 yeast cells per milliliter. Which of the following is the concentration of yeast cells per milliliter in test tube c. a) 15 b) 150 c)15,000 d) 150,000

b) 150

Which of the following results from the release of the neurotransmitter dopamine into the nerve synapse? a) the presynaptic neuron fires an action potential b) Dopamine binds to receptor proteins on the membrane of the postsynaptic cell c)Dopamine enters the cytoplasm of the postsynaptic cell d) dopamine cause the postsynaptic nerve cell to contract

b) dopamine binds to receptor proteins on the membrane of the postsynaptic cell

what is incomplete dominance?

both heterozygous alleles are expressed.

Barriers to infection include:

both physical barriers that block entry of pathogens and proteins that impede pathogen replication.

what makes up the central nervous system?

brain and spinal cord

steps in prewriting?

brainstorming, outline, diagram, stream of consciousness

what is osteoblast?

builds bone

10) Which of the following is correct regarding the pH scale? a) A substance with a pH of 3 is 10 times as alkaline as a substance with a pH of 4 b) A substance with a pH of 3 is twice as acidic as a substance with a pH of 4 c) A substance with a pH of 3 is 10 times as acidic as a substance with a pH of 4 d) A substance with a pH of 3 is twice as alkaline as a substance with a pH of 4

c

11) Lithium has an atomic number of 3 and a mass number of 7. Which of the following is the number of protons in a lithium atom? a) 4 b) 12 c) 3 d) 7

c

52) Which of the following occurs when the diaphragm contracts? a) Air is forced out of the lungs b) Alveoli decrease in size c) The thoracic cavity increases in volume d) The ribcage contracts

c

7) A student is comparing four different types of cells and observes that one contains many more mitochondria than the others. The student should conclude that the cell with the greatest number of mitochondria is most likely which of the following? a) Epidermal cell b) Sebaceous gland cell c) Muscle cell d) Fat cell

c

Which of the following percentages is the discount for a textbook if the original price is $107.00 and the discount of $62.10? (Round to the nearest whole percentage.) a. 42% b. 45% c. 58% d. 72%

c

Nails are made of a protein called ____ a) collagen b)vinculin c)keratin d) elastin

c) keratin

Which pf the following explains why gases are termed "compressible" and liquids are not? a) particles have fixed volume and are able to occupy the contained in which they are contained b) particles do not have much empty space between them, thus making them rigid c) particles have a large amount of empty space between them d) particles are held closely together by intermolecular forces

c) particles have a large amount of empty space between them

Which of the following describes the function of the ribosome in a living cell? a) production of ATP molecules from energy in nutrient molecules b) Disposal and recycling of damaged or unneeded cell parts c) Use of genetic information to build specific protein molecules d) regulation of molecules entering and leaving the cell

c) use of genetic information to build specific protein molecules

what are the 2 major parts of the nervous system?

central and peripheral

Which of these represents the correct order of some of the main energy changes that take place in a coal-fired power station? chemical, electrical, heat, kinetic chemical, heat, kinetic, electrical kinetic, heat, electrical, chemical heat, chemical, kinetic, electrical

chemical, heat, kinetic, electrical

14) Which of the following hormones regulates blood calcium levels? a) Melatonin b) Growth hormone c) Norepinephrine d) Parathyroid hormone

d

6) Which of the following describes the function of a neuron? a) Stores potassium ions b) Produces electrolytes c) Creates electrical impulses d) Transmits sensory information

d

8) Blood clotting involves which of the following proteins? a) Trypsinogen b) Hemoglobin c) Albumin d) Fibrinogen

d

Afterward, the children were trying to talk to their mother at the same time; as a result, they were sent to their rooms despite the excitement? Which of the following phrases from the sentence indicates cause and effect? a. At the same time b. Despite the excitement c. Afterward, the children d. As a result

d

adrenal medulla gland hormones:

epinephrine norepinephrine

Phalanges are bones present in which parts of the body? fingers only cranium/skull ears fingers and toes

fingers and toes

What causes a moving body to resist a change in its state of motion? its speed its inertia its acceleration its momentum

its inertia

what are the functions of frontal lobe?

motor strip location, impulsivity, short term memory, emotion, voluntary movement, social functioning, creativity, expressive language.

muscle

movement ( not typically a connective tissue)

the respiratory tract is lined with:

mucus to trap incoming pathogens.

what is an example of a disease where the immune system attacks itself?

multiple sclerosis

Rapport

mutual understanding and harmony

what is the functional unit of the kidney is:

nephron

The dermis, or middle layer of the skin contains:

nerve endings, hair follicles, sweat and oil glands, and capillaries.

A large amount of energy is produced in the center of sun by which phenomenon? nuclear fusion nuclear fission radioactive decay chemical reaction

nuclear fusion

The volume of air that remains present in the lungs, even after forceful expiration/exhalation, is called ____ and is about ____ liters. tidal volume, 2.4 liters residual volume, 1.2 liters vital volume, 2 liters residual volume, 0.5 liters

residual volume, 1.2 liters

Vitamin A, also known as ____, is a ____ vitamin. Its deficiency can lead to ____. calciferol, fat-soluble, osteoporosis ascorbic acid, water-soluble, night blindness retinol, water-soluble, osteoporosis retinol, fat-soluble, night blindness

retinol, fat-soluble, night blindness

Large lymphatic trunks unite to form which two main vessels of the lymphatic system? left lymphatic duct and thoracic duct thoracic duct and superior vena cava right lymphatic duct and thoracic duct right lymphatic duct and left subclavian vein

right lymphatic duct and thoracic duct

what do oil glands, or sebaceous glands secrete:

sebum, a mixture of fats and proteins that prevents the skin and hair from drying out.

pituitary gland

secretes hormones that control others gland

what are the 3 main functions of the nervous system?

sensory input, integration, and motor output

what are the functions of the parietal lobe?

sensory strip location, perception, touch(pain & temperature), ability to draw, reading and writing, calculations.

which bone type is embedded in the tendons?

sesamoid bones

RNA consists of what kind of strand?

single strand

melatonin hormones:

sleep cycles, reproductive cycles in many mammals.

where are nutrients absorbed in the body? a. esophagus b. small intestine c. large intestine. d. colon

small intestine

In a chemical reaction: sulfuric acid + substance X = salt + water + carbon dioxide. What could substance X be? magnesium hydroxide copper(II) oxide sodium carbonate potassium

sodium carbonate

what does unsaturated mean?

some carbon molecules in the chain are bound to one hydrogen and double-boned to the adjacent carbon.

What is Expository?

something you find in an academic journal, a recipe, etc.

what is the correct term to describe the amount of energy needed to change the temperature of 1 kg of a substance by 1 degree Celsius:

specific heat capacity

The follicle-stimulating hormone in the male reproductive system stimulates:

spermatogenesis

Hair follicles warming the body:

stand erect

what does amylase break down?

starch

what does carbohydrates convert?

starch and glycogen into sugar.

function of estrogen:

stimulates egg maturation, controls secondary sex characteristics.

Hyperthermia:

the body temperature is elevated above normal, the integumentary system helps to reduce body temperature through sweat and vasodilation of the blood vessels.

what is progesterone produce by?

the corpus luteum in the ovary.

what is the sympathetic system responsible for?

the fight or flight response

Maturation and development of T cells takes place in which region? spleen bone marrow thymus gland lymph nodes

thymus gland

Thyroxine

thyroid hormone production is associated with goatees and diseases that affect the thyroid gland

which organs needs iodine?

thyroid.

mRNA then leaves the nucleus and is ________into protein by ribosomes.

translated.

what are the pores in the nuclear envelope used for?

transport molecules, also regulate import and export of material between nucleus and cytoplasm

What is a compound sentence?

two or more independent clauses with no dependent clauses. usually, the independent clauses are joined with a comma and a coordinating conjunction, or they can be joined with a semicolon.

Which pyrimidine base is only found in RNA? uracil guanine cytosine adenine

uracil

sweat can also contain trace amounts of:

urea or other waste products in the blood, like alcohol or lactic acid.

Sweat glands, or sudoriferous glands, are responsible for the excretion of:

water and minerals from the body, in particular sodium, chloride, and magnesium.

Graveyard Walls (Steven walked through the graveyard every day...) Which of the following details best supports the main idea of the passage, "he must stay here a full hour to win the bet"?

"An hour in this place just wasn't worth it."

A home improvement is store is selling lawn chairs. There are 520 chairs in the inventory of those 2/5 are green. Find the number of green chairs in the inventory.

$208

Each month, an employee withholds $468 from her gross paycheck of $2,350. Which of the following is her annual net salary?

$22,584

example correlative conjunctions:

(Either) you are coming, or you are staying. He ran (not only) three miles, (but also) swam 200 yards.

what are the germ layers?

- Endoderm - Mesoderm - Ectoderm

The robbery passage( Dog and Humans are best friend) is what kind of passage? when it asks how would you rename this passage answer is:

- Persuasive - Jewel Thief on the Lose)

what is lysosomes?

-Digest proteins, Carbs, and lipids. -Gets rid of worn out particles.

How many organ systems are in the human body? a. 12 b. 15 c. 9 d. 11

11

The mass of a sample is taken

110.2

The ratio of the width of the length of a rectangle is 1:3. Which of the following ratios represents a rectangle with an equivalent ratio?

21:63

The ratio of the width to the length of a rectangle is 1:3. Which of the following represents a rectangle with an equivalent ratio?

21:63

A pudding recipe for 50 people calls for 4 cups of sugar. Each bag of sugar contains 6 cups. How many bags of sugar will be needed to make this recipe for 300 people?

4 bags

at what age does the thymus shrink?

65

blood vessel that carries nutrients directly from the small intestine to the liver:

Hepatic portal vein

endometrium

Highly vascularized tissue of the uterine lining

Which of the following is found in breast milk? a. IgE b. IgA c. melanin d. insulin

IgA

DNA can copy itself quite accurately while RNA cannot because only DNA is a) In double helix form b) A polymer of nucleotides c) Confined to the nucleus d) Surrounded and supported by Justine proteins

In double helix form

OUCH is an:

Interjection: a word that expresses strong emotion

Fat in the blood

Is taking

What is the part of the small intestine between the duodenum and ileum?

Jejunum

The arteriole entering the glomerulus, the afferent arteriole is?

Larger than the arteriole exiting the glomerulus

A doctor explains that a patient has pulled a muscle in his back, which of the following muscles could have been injured in the patient's back?

Latissimus dorsi

A researcher is gathering large amounts of data about patients who have a common disease and the effectiveness of various medications in comparison to placebos. Which of the following types of graphs is the best way for the researcher to represent data?

Line graph

What is catalyst?

Speed up the rate by providing a shortcut that lowers the amount of energy required for the reaction to occur.

Nerve impulses are transmitted from the extremities of the body to the brain via?

Spinal cord

The brain stem connects the cerebrum to the?

Spinal cord and controls critical involuntary body functions

Enzymes speed up the rate of reaction by lowering what?

The activation energy, or energy required for the reaction to take place.

What artery supplies oxygenated blood to the reproductive system?

The gonadal

Fill in the blank: The hepatic portal vein carries ____, ____ blood from the ____ to the ____. oxygenated, nutrient-rich, intestines, liver deoxygenated, nutrient-rich, liver, stomach deoxygenated, nutrient-rich, intestines, liver deoxygenated, nutrient-poor, liver, intestines

The hepatic portal vein carries deoxygenated, nutrient-rich blood from intestines to the liver.

The shear number of people in the audience overwhelmed the actress as she peeked through the curtain before scene one. Which is the error in the sentence?

The homophone "shear" should be "sheer" (unqualified)

What is causality in scientific measurements and relationships?

The relationship between two events or states such that one brings about the other, with one variable being the cause and the other the effect.

Stimulus 4 of 6 which of the following sentences as used in the passage indicates a "compare and contrast" text structure (Steven walled through the graveyard)

Things were different this evening"

Which of the following describes the structure of the passage?

This passage provides facts about hummingbirds.

stimulus question 2 of 2: Which of the following sequences reflect the correct sequence of the memo

This winter will be extraordinarily severe.employees who need equipment to work from home may request it. Employees will be notified by email if the building closes.

what is thrombocytosis?

Thrombocytosis is a disorder in which your body produces too many platelets (thrombocytes), which play an important role in blood clotting.

What is the nucleus?

Where DNA is located and tells cells their functions.

what does seminal vesicles secrete:

alkaline fluids with proteins and mucus into the ejaculatory duct.

what connects the large intestine to the appendix?

cecum

correlative conjunctions:

either.....nor neither....nor not only.....but also

Mineralcorticoids (salt):

electrolyte balance in the adrenal cortex.

mono and disaccharides are important sources of energy that are found in:

fruits (sucrose and fructose) and milk products ( lactose).

Bursitis

inflammation of a bursa usually caused by a blow or friction; the fluid-filled sacs in your joints

Function of Calcitonin?

inhibits the release of calcium from the bones

Which ventricle is the most muscular?

left venticle

Which if the following is the meaning of "rapport"

mutual understanding

when multiple amino acids are linked by peptide bonds, they form what kind of chain?

polypeptide chain, often referred to as the protein's primary structure.

The atrium receives blood:

returning to the heart from other areas of the body.

What does salivary glands secrete?

saliva

water that is saline contains significant amounts of dissolved_____.

salt

What contains 2 reactants that are uniting to form a single product:

synthesis chemical reaction

Which of the following correctly rounds the value below to the hundredths place? 1782.96471

1,782.96

Which of the following correctly rounds the value below to the hundredths place? 1782.96471

1,782.96

what are the bone healing steps:

1. Inflammation & clotting. 2. Fibroblasts secrete matrix forming cartilage for new bone to build off it. 3. Osteoblasts will secrete new bone spindle, where osteocytes will maintain the bone. 4. clasts will dissolve cartilage and replace it with bone.

What are the fives steps in scientific method?

1. Observation 2. Hypothesis 3. Experiment 4. Data analysis 5. Conclusion

A student is conducting an experiment using a ball that is attached to the end of a string on a pendulum. The student pulls the ball back so that it is at an angle to its resting position. As the student releases the ball, it swings forward and backward. The Student measures the time it takes the ball to make one complete period. A period is defined as the time it takes the ball to swing forward and back again to its starting position. This is repeated using different string lengths. The student formed the following hypothesis: Lengthening the string of the pendulum increases the time it takes the ball to make one complete period. 1. What correction would you have the student make to the hypothesis? a. Turn it into an "if/then" statement b. Add the word "will" in the middle after the word "pendulum" c. Switch the order of the sentence so that the phrase about the phrase about the period comes first, and the phrase about the string's length is last. d. No corrections are needed. 2. what would be an appropriate control variable for this experiment? a. the period b. the length of the string c. the mass of the ball d. the color of the ball

1. Turn it into an "if/then" statement 2. the mass of the ball

(½)(10t)=50,000

10,000

which of the following is the correct value of 3+ 2*6-4 a. 32 b. 10 c. 11 d. 26

11 How? 2*6=12+3=15-4=11

A body of mass 30 kg is traveling in space at a speed of 10 m/s. Given that the formula for kinetic energy is KE=0.5⋅m⋅v2, and m is mass and v is speed of an object, what is its kinetic energy? 4600 J 120 J 1000 J 1500 J

1500 J Kinetic Energy =0.5⋅30⋅10⋅10=1500J

how many amino acids are there?

20

A student measures 2,000 ml water into a cylinder. If water has a mass of 1g/ml, which of the following is the approximate mass of the water? a. 200mg b. 2g c. 200cg d. 20hg

20hg

Five hundred forty-eight runners participated in a recent race.twenty-six runners did not finnish the race. Half of those who did not finish completed the race in less than two hours. Which of the following is the number of runners who completed the race in less than two hours?

261

Curtis is taking a road trip through Germany, where all distance signs are in metric. He passes a sign that states the city of Dusseldorf is 45 Kilometers away. Approximately how far is this in miles? a. 42 miles b. 37 miles c. 28 miles d. 16 miles

28 miles 1m=0.62km 1m/0.62km=X/45=0.62*45=27.9 rounded to 28 miles

Lithium has an atomic number of 3 and a mass number of 7. - Which of the following is the number of protons in a lithium atom?

3

A student has a brass cube with the following mark: 300 cg. What is another way to express the weight of the cube? A. 3 mg B. 3 cm C. 3 grams D. 3 decagrams

3 grams

A vertical 1-meter stick casts a shadow of 0.4 meters. if a tree casts a shadow of 12 meters at the same time, how tall is the tree?

30 meters

Simplify the expression 2/7 + 3/5

31/35

5/7 - 2/9 simplify the expression above. Which of the following is correct?

31/63

The image below is a basketball court. Which of the following is the area of the entire court?

4,700 square feet

Clyde and James both own fast food franchises. They own a total of 1,200 franchises between them. If James owns 65% of the fast food franchises, how many franchises does Clyde own? a. 500 franchises b. 480 franchises c. 600 franchises d. 420 franchises

420 franchises How? 1200 X 0.35= 420

Consumer answered a survey question about which type of frozen dessert they preferred: surbet, sherbet, frozen custard, frozen yogurt or ice cream. The results shown in the graph above display the first 1,000 responses.

450

Students are offered a $2.50 discount on all movies that start before 5:00pm.How many different movies could a student see at the discount price?

5

Which of the following point of fraction with a common denominator are equivalent to 1/4 and 2/5?

5/20, 8/20

Dark hair is dominant over red hair. What is the probability that a red-haired mother and a dark haired father with a heterozygous genotype will have a dark haired child? a. 25% b. 50% c. 75% d. 100%

50%

A lab scale weighs 3 pounds, 5 ounces. Which of the following is the scale's weight in (oz)

53 oz

. If the pediatric unit of a hospital has 165 beds, how many rooms are there if each room holds 3 beds?

55

If the pediatric unit of a hospital has 165 beds, how many rooms are there if each room hold 3 beds?

55

All of the 1,020 students at a local high school must enroll in a foreign language course. If 269 students are enrolled in French, 663 are enrolled in Spanish, and the remaining students are enrolled in german, which of the following is the percent of students enrolled in german? - Round to Nearest whole number

65

Seven more than twice a number is equal to twenty one. Which of the following equations expresses the phrase?

7 + 2x=21

The element boron (symbol B) is listed in the periodic table as having an atomic number of 5 and a mass of 10.8 amu. Which of the following could be true? a. 80% of boron isotopes have 6 neutrons. b. Boron never contains more neutrons than protons c. The mass shown in the periodic table is incorrect; atomic mass can only be a whole number. d. Boron contains partial neutrons.

80% of boron isotopes have 6 neutrons.

which of the following fractions is equivalent to 83.1% A. 831/1000 B. 83.1/10 C. 831/10 D. 0.831

831/1000 How? 831/1000= 0.831 X 100=83.1%

A patient drives 19 miles one way to medical facility for treatment. How far does the patient drive round trip in 22 days of treatment?

836 miles

The width of a rectangle is 4 inches (in) and the area of the rectangle is 32 in square. Which of the following represents the length of the triangle

8in

Both Tom and Mary Jo are brown eyed, but their baby, John, is blue eyed. Tom and Mary Jo have free earlobes, but John's earlobes are completely attached. Brown eyes (B) are dominant to blue eyes (b), and free earlobes (A) are dominant to attached earlobes (a). What is the probability that John's baby sister will have brown eyes and free earlobes like her parents? a. 1/16 b. 3/16 c. 7/16 d. 9/16

9/16 why? A child will have the dominant phenotype for both traits if there is at least one B and at least one A in her genotype.

A bakery sells three batches of muffins. On a recent morning, the bakery sold 41 blueberry muffins, 27 bananna nut muffins, and 20 bran muffins. Estimate the total muffins sold.

90 muffins

Which of the following best describes protection from antigens due to passive immunity?

A mother passes antibodies to her child through breast milk

Which of the following is the best source to consult for information about the famous jazz saxophonist Charlie "Yardbird" Parker's childhood in Kansas City in the 1940s?

A published magazine interview with Charlie Parker

The smaller the diameter of the efferent arteriole the resists of?

A steady flow of blood, increasing the BP within the glomerulus

Which of the following is the term for the gap between two nerve cells?

A synapse

Which of the following sentences is correctly punctuated?

A working student's best friend is her phone.

1. Of these four sentences, which is the best example of a compound sentence? A. Carolyn went to the grocery store, but Annie decided to go to the market instead. B. Andy worked hard to complete his exam. C. Because Gerald was late for school, he was unable to take the test. D. Although Amy and Annie wanted to ride the bus to class, they did not have the required bus fare.

A.

convert each decimal to a percent. A. 1.22 B. 0.384

A. 1.22 X 100= 122% B. 0.384 X 100= 38.4%

convert each fraction to a decimal. A. 19/10 B. 13/50

A. 1.9 B. 0.26

The code of a gene is delivered to the protein-producing region of a cell by? a. m-RNA b. t-RNA c. DNA d. ATP

A: Messenger RNA (m-RNA) carries the information contained in DNA molecule to the ribosome where proteins are made. t-RNA carries amino acids to the ribosome. ATP is involved in the storage of energy and not genetics.

In which of the following structures of the respiratory system does the majority of gas exchange between the circulatory and respiratory systems occur?

Alveoli

Which of the following data supports the author's argument about why Americans lack awareness about the dangers of malaria around the world?

An epidemiology report finds there have been no cases of malaria originating from the US in the last 60 years.

Which of these patterns of the periodic table is not correct? Densities and melting points increase down any group. In non-metallic groups, reactivity decreases down a group. In metallic groups, reactivity increases down a group. Atomic size increases down a group and across a period (from left to right).

Atomic size increases down a group and across a period (from left to right).

The nurse, who was late for work, put the manual into a large blue tote bag getting out of the car. Which of the following is misplaced in the sentence above? A. who was late for work B. getting out of the car C. into a large blue tote bag D. put the manual.

B

Mustache: Which of the following primary sources would someone consult to learn more about mustache styles that were popular during the 1920s? A. Personal letters from Charlie Chaplin to Hollywood friends. B. A magazine article about the world Mustache. C. Tournament photographs of people at stage in 1924. D. A population census conducted in 1920.

B. A magazine article about the world Mustache.

2. As a true bibliophile, she spent a great deal of the time in the library. Which of the following defines the word 'bibliophile'? A. A university professor B. A lover of books C. A research scientist D. A collector of rare maps

B. lover of books

Many chemical reactions occur more rapidly with platinum as a catalyst. At the end of the reaction, the platinum is found to be? a. increased in quantity b. unchanged in weight c. changed into another state d. combined into the final product

B: A catalyst only affects the rate of a reaction and is not changed or consumed during it.

A cell is placed in a drop of distilled water. The cell swells and bursts. The most likely cause of this reaction is that? a. active transport of salts occurs b. water moves from an area of low particle concentration to an area of high particle concentration c. water moves from an area of high particle concentration to an area of low particle concentration d. salts from the distilled water enter the cell

B: Osmosis refers to the diffusion of water from areas that contain a high concentraion of water and low concentration of solute (outside the cell) to areas that contain a low concentration of water and high concentration of solute (inside the cell).

A 36-W lam is operated by a 9-V battery. How many amperes of current are flowing through the lamp? a. 0.25 A b. 4 A c. 27 A d. 45 A

B: Power = Voltage x current; 36w = 9v x current; current = 4a

If a plain Soda cracker is chewed slowly, after a while it begins to taste more? a. salty b. sweet c. bitter d. bland

B: The enzyme amylase, which is found in saliva breaks down the starches present in food, such as crackers into sugars.

Stimulus question 1 of 3 which of the following inferences can the reader draw according to the passage? (Three candidates show up to an interview)

Being on time for a job interview and dressing appropriately is important to securing a job

If damage occurs to the alveoli of the lungs, which of the following is true?

Blood will not be oxygenated

1. Understanding which flavors complement each other can take years of practice. 2. Following these basic tenets will allow you to produce recipes that your friends and family will love to share. 3. Once you've mastered flavor combinations, however, there's no end to the tasty experiments you can think up. 4. The key to developing a truly delicious and user-friendly recipe is to focus on flavor combinations and simplicity. 5. For kitchen experiments to become usable recipes, you should keep the user in mind and focus on clear, concise instructions. 30.Using the sentences above, which of the following sentence orders best forms an organized, logical paragraph? A. 1, 3, 4, 5, 2 B. 2, 1, 3, 5, 4 C. 4, 1, 3, 5, 2 D. 5, 2, 1, 4, 3

C

11. Which of these sentences is not an example of subject-verb agreement? A. Four boys were teasing a group of girls in their class. B. Cierra's blanket collection was getting out of control. C. The students was driving the teacher crazy by screaming out answers. D. Behavior in children is most often determined by parental behavior.

C

A coach shows up to football practice one day and all the equipment is missing, before he notifies the principal he tries to figure out the most plausible reason for this. Which is the most logical conclusion based on the four answers listed below.

C) the school budget is cut

What cell produce Pepsin?

Chief cell (chief produce pepsinogen and when combined with HCL produce pepsin.)

Stimulus question 2 of 6 which of the following conclusions can the reader make from the passage(CHIMNEY SWIFTS)

Chimney swifts can easily adapt roosting behaviors to any structure made of the proper material

Which of the following conclusion can the reader make from the passage?

Chimney swifts can easily adapt roosting behaviors to any structure made of the proper material

Which of the following conclusions can the reader make from the passage?

Chimneys swifts are can easily adapt roosting behavior to any structure made up of proper material.

Which element within the respiratory system is responsible for removing foreign matter from the lungs? a. Bronchial tubes b. Cilia c. Trachea d. Alveoli

Cilia

Which of the following shows cause and effect?

Consequently

11.which of the following is an example of accidental plagiarism? A. Submitting a paper that was written by a friend. B. copying information from a source and identifying it as your own C. Summarizing information without including a source credit D. Forgetting to include an in-text citation for information

D

14. Which of the following sentences would most likely to be found in an email from a professor to her biology students at a collage? A. Traffic on Highway 32 will be rerouted to Main Street during renovation of the 7th Street Bridge. B. I didn't get any constructive feedback on my paper during the peer review last week. C. We want to go out for dinner, but we haven't decided where to eat yet. D. Research groups from the 10:30 a.m. Tuesday class should meet in the library this week.

D

9. which of the following sentences correctly uses subject-verb agreement? A. computers has made our jobs easier. B. Those two girls in the first row is best friends. C. The dogs in our backyard growls late at night. D. The tennis balls roll into the parking lot.

D

This sentence is an example of what common mistake in sentence structure? "Sarah and Tommy danced at the wedding, Sarah's mom took pictures." A. sentence fragment B. run-on sentence C. compound sentence D. comma splice

D

Which laboratory reagent turns starch blue-black? a. phenolphthalein b. Benedict's solution c. nitric acid d. Lugol's solution

D: Lugol's solution (iodine plus potassium iodide) is an indicator that turns blue-black in the presence of starch.

A mixture such as mayonnaise is best described as? a. a substance b. saturated c. a solution d. an emulsion

D: Mayonnaise is made up of oil, vinegar and eggs which form a type of heterogeneous mixture called an emulsion. An emulsion will eventuallly separate, while a solution will not. The terms "saturated" and "unsaturated" refer to solutions. Mixtures are not substances.

Which of the following is considered a principal ecological role of green plants in the environment? a. providing shade to regulate the environmental temperature b. using the mineral content of the soil c. providing nesting places for animals d. manufacturing food from simple materials

D: Plants undergo photosynthesis, which allows them to make glucose from the simple materials carbon dioxide and water.

Offspring of parents who eat a diet low in omega-6 fatty acids have fewer health problems than offspring of parents who consume diets rich omega-6 fatty acids. Which of the following statements provides the most logical extension of this observation? a. The offspring of parents who were starved will be healthier than offspring of well-fed parents. b. Diets rich in omega-6 fatty acids are poor nutritional choices for long-term family health. c. Any high-calorie diet that omits omega-6 fatty acids is beneficial to long-term family health. d. Health outcomes of a person's diet are visible in the health of offspring rather than the individual.

Diets rich in omega-6 fatty acids are poor nutritional choices for long-term family health.

What is the pancreatic lipase function?

Digest lipids and fats in the small intestine

Trysin (ogen) proteases?

Digest proteins in the small intestine

when two monosaccharides undergo dehydration synthesis, they form what?

Disaccharide

According to the prefix "in-" which of the following words defines "incredulous"?

Disbelieving

An experiment is designed to determine the effect of a new drug on controlling blood sugar in people with diabetes. Volunteers of similar ages and diets are enrolled in the study and randomly assigned to two groups: one group receives the drug, and one does not. Following one month of treatment, blood is collected from the subjects to measure blood glucose concentrations. Which of the following parameters is the independent variable? A. Blood glucose concentration B. Drug Treatment C. Age D. Diet

Drug Treatment

Which of the following best describes matter in which the components cannot be broken down into simpler substance? a. molecule b. element c. mixture d. compound

Element

Where does fertilization begin?

End of the Fallopian tube

The Central Park passage is what kind of passage?

Expository/ informative - the central park was in New York

If a hydrocarbon chain ends in a carboxyl group is is called a?

Fatty acid.

What is the function of the lymph nodes in mammals?

Filter debris, lymphocytes, and pathogens from intercellular fluid

what hormones are found in the anterior pituitary gland?

Growth hormone GH Prolactin PRL Follicle stimulating hormone FSH Luteinizing hormone LH Adrenocorticotropic hormone ACTH Thyroid stimulating hormone TSH

The terrible conditions prompted Richard to become politically involved. By the end of the year,he__to organize a grassroots campaign in the neighborhood. Which of the following options completes the sentence correctly?

Had began

Each successive shell represents a?

Higher energy level, and as the energy level increases, the electrons are, on average, further away from the nucleus.

which of the following types of bonds link base pair nucleotides within a double strand of DNA?

Hydrogen

example of a declarative sentence:

I can fly a kite. The plane left two hours ago.

. Did you know that just 10% of the population is left-handed?... Which of the following statements restates the main idea of the passage?

Identical twins are more likely to have the same dominant hands.

A

In A Room of One's Own, Woolf points out that women feature prominently in literature, but are seldom mentioned in history. Which best states the claim that this information helps to develop? Choice Feedback A. Literary writers might pay lip service to women, but historians ignore them. B. Both literary writers and historians have suffered for their craft. C. Few women have accomplished anything in history that is worth mentioning. D. Women prefer to influence history by supporting the men who make it.

second section of the small intestine where majority of absorption occurs?

Jejunum

Example of a compound-complex sentences

John is my friend who went to India, and he brought souvenirs for us. You may not know, but we heard the music that you played last night.

The nephridium in worms has a function most similar to which organ in humans

Kidney nephridia in segmented worm operate similarly to the nephron of the kidney

Which of the following best rephrases the author's point of view about media coverage of sleepwalking?

Media coverage of sleepwalking is sensationalized.

Which of the following is NOT an organic substance? A. C6H12O6 B. MgSO4 C. C2H6O D. C12H22O11

MgSO4 Why? organic substances contains both carbon and hydrogen.

Once the blood has been oxygenated, it travels through the pulmonary veins, through the left atrium, and then through the _______ before entering the left ventricle. a. Tricuspid valve b. Mitral valve c. Pulmonary arteries d. Aorta

Mitral valve

Pharynx

Muscular region at the intersection of the respiratory and digestive system

Which of the following resources would help a reader gather information about economic trade during World War 1?

News articles and textbooks

Heterotrophic with a genome stored in DNA

Organisms that obtain their nutrients or food from consuming other organisms.

The function of the Golgi apparatus is to

Package proteins

stimulus 6 of 6 which of the following inferences can be made from the following section from the passage? Carter was ahead of the times. He set this example for the American people as part of a campaign to conserve energy

President carter did had different views than many of his contemporaries

Mucin (mucus)

Produced by the lung cell that absorbs water

A patient in a sleep club has a prescription for a continuous positive airway pressure machine for sleep apnea. She would like a lifetime warranty. She is deciding between the four models listed in the chart above. Which of the following is the most economical buy for her situation?

Product C

Which is the last step of the writing process? A. Proofreading B. Brainstorming C. Citing primary sources D. Identifying a thesis

Proofreading

Which of the following organic compounds are used to manufacture steroid hormones in the body?

Protein

What produces ammonia by deamination in the liver?

Proteins

Globular proteins contains what kind of group?

R groups that allow them to be soluble in water.

C

Read the following excerpts from two speeches given by Franklin D. Roosevelt at the onset of America's entrance into World War II in 1941. The first one was given to Congress and the second one was a radio address to the nation: 1. As Commander in Chief of the Army and Navy, I have directed that all measures be taken for our defense. But always will our whole nation remember the character of the onslaught against us. . . . [T]he American people in their righteous might will win through to absolute victory. 2. And in the difficult hours of this day — through dark days that may be yet to come — we will know that the vast majority of the members of the human race are on our side. Many of them are fighting with us. All of them are praying for us. But, in representing our cause, we represent theirs as well — our hope and their hope for liberty under God. Which most accurately describes a major difference in the emphasis between the two speeches? Choice Feedback A. Speech #1 conveys a less certain tone concerning future victory than Speech #2. B. Both speeches pronounce a dark, uncertain future for the United States. C. Unlike Speech #1, Speech #2 uses emotional language to remind Americans that they are not alone. D. Unlike Speech #1, Speech #2 employs logos to indicate that war is necessary.

A

Read this excerpt from Franklin Delano Roosevelt's "Fireside Chat 19: On the War with Japan": In 1931, ten years ago, Japan invaded Manchukuo — without warning. In 1935, Italy invaded Ethiopia — without warning. In 1938, Hitler occupied Austria — without warning. In 1939, Hitler invaded Czechoslovakia — without warning. Which sentence most accurately evaluates the rhetoric of Franklin Delano Roosevelt's speech? Choice Feedback A. The president uses repetition to emphasize the suddenness of the events. B. The president relies on repetition to ensure that his listeners remember the date of each event. C. The president uses incendiary language to evoke anger from the American listeners. D. The president mentions the dates to emphasize his credibility with the listeners.

A group of researchers is designing a study to test the effect of a specially formulated, vitamin-infused drink on the duration of the common cold in the general population. The researcher recruit 200 participants, 165 women and 35 men. They randomly assign 100 participants to the control group and 100 participants to the treatment group. During the trial, the members of the control group will receive a placebo drink, and the members of the treatment group will receive the vitamin-infused drink. In what way could the design of this experiment be improved? A. Give more participants the vitamin-infused drink B. Give the more participants the placebo drink C. Recruit more men for the experiment D. Recruit more women for the experiment

Recruit more men for the experiment.

What are the steps for bone healing formation?

Remodeling hematoma forms Fibrocartilage callus formation Bony callus forms Bone remodeling occurs compact bone lay down

Which of the following best describes the relationship between the circulatory system and the integumentary system? a. Removal of excess heat from body b. Hormonal influence on bp c. Regulating of blood's pressure and volume d. Development of blood cells within marrow.

Removal of excess heat from the body

The kidneys receive blood from the left and right?

Renal arteries

The blood-filtering portion of the nephron comprised of the glomerulus & the Bowman's capsule?

Renal corpuscle

DNA is a molecule that must under go what?

Replication before cell division can occur.

After working all afternoon, Sara has to return books to the library. Before she can go to work, though, she needs to go to the post office, take a shower, and make a plan for dinner. - Which of the following actions will Sara take when she leaves work?

Return books to the library

After working all afternoon. Sara has to return books to the library. Before she can go to work, though, she needs to go to the post office, take a shower, and make a plan for dinner. Which of the following actions will Sara take when she leaves work?

Return books to the library.

a scientist has noted a possible relationship between a certain chemical substance found in fish and the occurrence of kidney failures in humans. Which of the following preliminary steps should the scientists take before conducting a controlled experiment?

Review the literature to determine what other research is available to the topic

An individual call is able to make copies of its genetic information but is unable to produce encoded proteins. This indicates a possible problem in the_______.

Ribosome

stimulus 1 of 2: which of the following is the main idea of the passage( Did you know that 10% of the population)

Right- or left- handedness doesn't really effect the outcomes of people's lives

The system of interconnected tubes involved in protein production is called

Rough endoplasmic reticulum

Donna is an excellent nurse, but her patients find her bedside manner to be somewhat "churlish" and unpleasant. What "churlish" means?

Rude

Which of these is known as the pacemaker of the heart? SA node cardiac plexus (nerve fibers) AV node muscle fibers of the atria

SA node

Copy the letter above on a piece of paper. Cross out the second and last letters. Replace all vowels with the letter C. Insert an O before the second C. cross out the first and third letters. Double the second letter. Which of the following letter sequences remains oidson the page?

SFFMOC

_P W S A F M I X_ Copy the letter string above on a piece of paper. Cross out the second and last letters. Replace all vowels with the letter C. insert an O before the second C. cross out the first and third letters. Double the second letter. - Which of the following letter sequences remains on the page?

SFFMOC

Which of the following valves prevent backflow of blood from the arteries into the ventricle? a. Bicuspid b. Tricuspid c. Mitral D. Semilunar

Semilunar Valve

what is metaphase?

Spindle moves to center of cell and chromosome pairs align along center of spindle structure.

Function of parathyroid hormone:

Stimulates the release of calcium from the bones.

What is the function of the Golgi apparatus?

Stores in and packages everything for the body.

what does the subclavian artery do?

Supplies blood to the right and left side of the body.

What is specific heat capacity?

The amount of energy needed to change the temperature of 1 g of a substance by 1 degree Celsius

Based on the passage, which of the following statements is true?

The author provides facts to support the main idea

A high school football team arrives to practice one day to find that all of their equipment is missing. The coach wants to make sure he has considered every possibility before he the principal. Which of the following pieces of evidence would be relevant to help explain what happened to the equipment?

The equipment failed a recent safety inspection

What is an experimental group?

The group in which the independent variable is changed.

Which gland is responsible for the regulation of calcium levels? a. The parathyroid glands b. The thyroid glands c. The adrenal glands d. The pancreas

The parathyroid glands

Which of the following distinguishes endocrine and exocrine glands from one another?

The presence of absence of ducts

Cooking with Super Fire Charcoal... Which of the following is correct according to the directions?

The user should read the lighter fluid directions before grilling.

Based on the passage, which of the following inferences can be made about the woman's sleepwalking event described in the second paragraph?

The woman's sleepwalking experience became a popular news story.

Which of the following sentences is correct?

There in the distance we saw a river, Its crystal-clear water sparkled in the morning sun.

Simple covalent compounds show which of the following properties? They are crystalline solids at room temperature. They are soluble in organic solvents e.g. ethanol. They conduct electricity when molten or dissolved in water. They have high melting and boiling points.

They are soluble in organic solvents e.g. ethanol.

Why did they start off the passage with Obama?

To get a picture of the current event that is happening now.

What is the function of the myelin sheath?

To protect the neuron, provide insulation, and increase the speed of impulse transmission.

In order to comprehend, you must identify the?

Topic or subject of the passage or text.

The walls of_____are thinner than those of arteries because they do not have to carry blood under high pressure contain valves to prevent back flow of blood:

Veins

Which of the following features of an almanac would be most useful to a person trying to decide whether to throw a fourth of July beach party next summer?

Weather forecasts

Once you have identified the topic of a text, you can easily identify the purpose for the reading and answer?

What am I reading about? and What is important to know about it?

A

Which aspect of Mary Shelley's Frankenstein most clearly indicates that the Industrial Revolution influenced the story? A. Recent scientific breakthroughs inspire Frankenstein to study the nature of life. B. Frankenstein decides to abandon his belief in the ancient philosophers. C. Characters cling to their faith in God and deny the newer scientific revelations. D. Frankenstein feels guilty for allowing Justine to be wrongfully convicted.

What happens when a piece of zinc is placed in a copper(II) sulphate solution? Zinc displaces copper from copper(II) sulphate solution. The original blue color of the solution persists. Zinc displaces copper from copper(II) sulphate solution. Sulphur dioxide is liberated. No reaction occurs because zinc is less reactive metal than copper.

Zinc displaces copper from copper(II) sulphate solution

What is the first stage of gestation?

Zygote

20) Which of the following is the sphincter muscle that regulated the movement of chyme into the small intestine? a) pyloric b) Esophageal c) Ileocecal d) Urethral

a

We wanted to do something different on our vacation last summer, so we chose backpacking in the wilderness. On the second day, we came to a lookout tower, which we climbed. Lookout towers are often made of metal so that they won't burn. From the top of the tower, I could see that fog covered the whole valley, wrapping it in a cotton-like blanket. To this day, that foggy view remains in my memory? Which of the following sentences does not enhance the meaning of the paragraph above? a. Lookout towers are made of metal so that they won't burn. b. We wanted to do something different on our vacation last summer, so we chose backpacking in the wilderness. c. On the second day, we came to a lookout tower, which we climbed. d. To this day, that foggy view remains in my memory.

a

Singing is my favorite thing to do. what is "singing" in the sentence?

a gerund, or verb that acts as a noun.

what is thrombopoietin?

a glycoprotein hormone produced by the liver and kidney which regulates the production of platelets.

The prostate gland secretes:

a milky white fluid with proteins and enzymes as part of the semen.

What is a topic?

a noun or noun phrase that encapsulates the subject matter of the writing.

Which of the following molecules, if present in urine, indicates a problem with kidney function? a) Albumin b) Creatinine c) Sodium Ions d) water

a) Albumin

Which of the following was the key reason that scientists began working on better treatments for malaria? a. To protect soldiers in combat b. Because the disease is more deadly than HIV/AIDS c. Because many Americans did not know about the risks of malaria d. To prevent the spread of malaria from other parts of the world to America

a. To protect soldiers in combat

Which train described below would have the greatest momentum? A A 9500 kg train car moving at 200 mph B An 8000 kg train car moving at 215 mph C A 9300 kg train car moving at 190 mph D A 8600 kg train car mobbing at 195 mph

a. a 9500 kg train car moving at 200 mph

The pharynx is part of what body system? A respiratory B muscular C lymphatic D circulatory

a. respiratory

what are protein monomers called?

amino acid

what attacks antigens:

antibodies.

What is the name for any substance that stimulates the production of antibodies? a. collagen b. hemoglobin c. lymph d. antigen

antigen

small molecules that are broad spectrum that target & kill many bacterial, viral, & fungal pathogens to prevent infection is:

antimicrobial peptides

Eccrine gland:

appear all over the body and are the primary sweat glands of the body.

23) Which of the following statements best supports the hypothesis that viruses can cause cancer? a) Cancerous and normal cells share genetic sequences b) Cellular DNA has sequences related to viral sequences c) Genes that regulate cell division are found in some viruses d) Viruses and cancer cells both replicate rapidly

b

Read the passage below before answering the question. 1 tbsp olive oil 3 cloves garlic, minced 1 tbsp fresh basil 1 tsp dried thyme 1/2 red onion, diced 1 lb boneless, skinless chicken breast, chopped 12 oz penne pasta 2 tbsp butter 4 oz goat cheese 1. Heat the olive oil in a pan over medium-high heat. 2. Add minced garlic, basil, thyme, and onion to oil and heat until onion is clear and tender. 3. Add chicken to pan and cook for 8 min. 4. While chicken is cooking, bring pasta to a boil in a saucepan. Drain and return to the pot. 5. Add butter and cheese to pasta and stir until melted, creating a sauce. 6. Toss the chicken mixture with the pasta, coating evenly with the cheese sauce. Serve hot. Makes 4 servings According to the recipe above, when should the onion be cooked? a. After the pasta boils b. After the goat cheese has melted c. After heating the oil in the pan After adding the chicken to the pain

c. After heating the oil in the pan

Which of the following is a logical conclusion based on the passage? a. Malaria and HIV/AIDS are the two most deadly diseases around the world b. Eradication efforts always have a high rate of success in eliminating infectious diseases c. Infection rates within a community may affect that community's understanding of diseases d. U.S. scientists' efforts to eradicate malaria were the decisive factor in winning World War II

c. Infection rates within a community may affect that community's understanding of diseases

Which of the following scenarios would NOT give a normal distribution? A distribution of the yearly income of families in the U.S. B The distribution of heights in a large sample of people. C The percentage of toxic wastes and pollution in the atmosphere vs. the lifespan of organisms in the location. D A curve with 97% of the population within the first two standard deviations.

c. The percentage of toxic wastes and pollution in the atmosphere vs. the lifespan of organisms in the location

Which of the following best rephrases the topic of the passage? a. The importance of biodiversity in areas where hummingbirds live b. The reasons for creating laws to protect hummingbird migration routes c. The wide variety of hummingbirds and their attributes d. The slow decline of the hummingbird population in the eastern portion of the United States

c. The wide variety of hummingbirds and their attributes

Which organ system is primarily responsible for regulating metabolism, mood, and growth? A respiratory system B digestive system C endocrine system D lymphatic system

c. endocrine

A patient who has suffered head trauma is not able to recognize people he has met around an hour earlier but is still able to recognize people he met before the injury. He most likely has damage to the: A Medulla Oblongata B Corpus Callosum C Hippocampus D Pituitary Gland

c. hippocampus

What type of bond between the complementary bases of DNA stabilizes the double helix structure? A covalent B ionic C hydrogen D nuclear

c. hydrogen

The addition of a catalyst to a chemical reaction will have what effect? A Increase the energy required for the reaction to take place. B Increase the time required for the reaction to take place. C Increase the rate at which the reaction takes place. D Reduce the amount of products in the reaction.

c. increases the rate at which the reaction takes place

What is the longest phase of the cell cycle? a. mitosis b. cytokinesis c. interphase d. metaphase

c. interphase

Which of the following is a characteristic of the autonomic nervous system? A. It regulates the voluntary control of body movements through the skeletal muscles. B It is the part of the nervous system consisting of the brain and spinal cord. C It regulates involuntary activity in the heart, stomach, lungs, and intestines. D It is thought to be the center of intelligence.

c. it regulates involuntary activity in the heart, stomach, lungs, and intestines

If a cell is placed in a hypertonic solution, what will happen to the cell? a.It does not affect the cell. b.It will stay the same. c. It will shrink. d. It will swell.

c. it will shrink

What is the role of ribosomes? a. storage b. waste removal c. make proteins d. transport

c. make proteins

In a vacuum, why doesn't an elephant accelerate faster than a penny as it falls to the ground? A Vector components of an elephant are more complex than those of a penny. B An elephant is not a projectile, while a penny is. C They both have an acceleration of 9.8 m/s2. D The elephant is greater in mass but is slowed less by the effects of air resistance.

c. they both have an acceleration of 9.8 m/s2

Which base is NOT found in DNA? A guanine B cytosine C uracil D adenine

c.uracil

what are the macromolecules that living organism made of?

carbohydrates, lipids, proteins, and nucleic acids.

I wanted to just set the table and go back to my video game, but my mother precisely arranged each napkin with every dish and utensil until they were perfectly aligned/ - Which of the following words is a synonym for "precisely"

carefully

what happens if acetylcholinesterase is inhibited at the synapse? a. causes a muscle stimulation b. causes a bone stimulation c. causes a bone contraction d. causes a nerve blockage

causes a muscle stimulation

Membrane proteins that contain both water soluble and insoluble components are embedded in?

cell membrane and serve as receptors for signal transfer between cells and transport ions or other materials across membranes.

The globular proteins also serve as?

cellular messages (hormones), in the immune system (immunoglobulins and antibodies), and as enzymes.

Examples of steroids?

cholesterol and sex hormones like estrogen and testosterone that are made from cholesterol

Urethra

connects the bladder to the outside of the body

blood vessels warming of the body:

constrict

what is a decomposition chemical reaction?

contains a single reactant that is broken down into two

what are the 2 major regions of the kidney?

cortex and medulla

Mucus can be either expelled through:

coughing, eliminating harmful microbes from the body, or swallowed.

The pumpkin crop was incredibly small this year because of the excessive rain in the spring and heat in the summer. Which of the following parts of speech describes the word "incredibly" as used in the sentence above? a. Adjective b. Preposition c. Pronoun d. Adverb

d

The ratio of the width to the length of a rectangle is 1:3. Which of the following ratios represents a rectangle with an equivalent ratio? (assessment) a. 33:100 b. 6:9 c. 5:7 d. 21:63

d

When an enzyme is degraded into its monomeric structural units, the monomers will be a)monosaccharides b)fatty acids c)nucleotides d)amino acids

d) amino acids

Read the passage below before answering the question. 1 tbsp olive oil 3 cloves garlic, minced 1 tbsp fresh basil 1 tsp dried thyme 1/2 red onion, diced 1 lb boneless, skinless chicken breast, chopped 12 oz penne pasta 2 tbsp butter 4 oz goat cheese 1. Heat the olive oil in a pan over medium-high heat. 2. Add minced garlic, basil, thyme, and onion to oil and heat until onion is clear and tender. 3. Add chicken to pan and cook for 8 min. 4. While chicken is cooking, bring pasta to a boil in a saucepan. Drain and return to the pot. 5. Add butter and cheese to pasta and stir until melted, creating a sauce. 6. Toss the chicken mixture with the pasta, coating evenly with the cheese sauce. Serve hot. Makes 4 servings At which of the following times does it make the most sense to mince garlic? a. After the onion is clear and tender b. After tossing the chicken mixture with the pasta c. While the pasta boils d. Before adding oil to the pan

d. Before adding oil to the pan

Based on the passage, which of the following inferences can be made about the women's sleepwalking event described in the second paragraph? a. The woman's experience is typical of most instances of sleepwalking. b. The woman was never in danger during her sleepwalking episode. c. The woman frequently climbs trees while awake. d. The woman's sleepwalking experience became a popular news story.

d. The woman's sleepwalking experience became a popular news story.

Which of the following sentences best summarizes the passage? a. When you think of a hummingbird, you most likely think of a very small creature with furiously beating wings. b. So it should be no surprise that the hummingbird family contains the smallest bird in the world. c. They are the only birds that can fly backward. d. They are marvels of speed, agility, and beauty.

d. They are marvels of speed, agility, and beauty.

Which of the following is true about eukaryotic cells? A They do not contain lysosomes. B They contain one chromosome. C They do not contain a nucleus. D They contain mitochondria.

d. They contain mitochondria

Which of the following statements from the passage contains only fact? a. It should be no surprise that the hummingbird family contains the smallest bird in the world. b. One thing that all hummingbirds have in common is their ability to amaze. c. Despite their diminutive stature, these birds can do many incredible things. d. When newly hatched, three of the birds can fit inside a nest the size of a large thimble.

d. When newly hatched, three of the birds can fit inside a nest the size of a large thimble.

A scientist wants to write an equation using the variables X, Y, and Z. What is the proper relationship between the variables if X represents time, Y represents energy, and Z represents power? A 2XY = Z B XZ = 2Y C Z = X/Y D Z = Y/X

d. Z= Y/X

Which of the following is an example of deductive reasoning? A Only 21% of a town's population have expressed a desire to ban all cars. Therefore, the new proposition banning cars in the state is not likely to pass. B People who brush their teeth regularly have fewer instances of heart disease. Therefore, good dentistry can prevent heart attacks. C All soccer players who play professionally can kick well. Therefore, any future soccer player who is recruited to play professionally will be able to kick well. D All secretaries are good typists. Cynthia is a secretary. Therefore, Cynthia is a good typist.

d. all secretaries are good typist. Cynthia is a secretary. Therefore, Cynthia is a good typist

The rate of a chemical reaction depends on all of the following except a. surface area. b. temperature. c. presence of catalysts. d. amount of mass lost.

d. amount of mass lost

In a cell, the Golgi apparatus is found in what location? A Inside the nucleus B Inside the mitochondria C In the cell wall D In the cytoplasm

d. in the cytoplasm

Which of the following statements is true regarding trophic levels? A Tertiary consumers acquire more energy content than secondary consumers. B Producers acquire less energy than primary consumers. C Secondary consumers acquire more energy than primary consumers. D Tertiary consumers acquire less energy than primary consumers.

d. tertiary consumers acquire less energy than primary consumers

Which of the following layers of the atmosphere is the closest to space? A troposphere B stratosphere C mesosphere D thermosphere

d. thermosphere

Which of the following is exchanged between two or more atoms that undergo ionic bonding? a. transitory electrons b .neutrons c. electrical charges d .valence electrons

d. valence electrons

what dos the spleen do?

filter blood and help fight infections

what do lymph nodes do? a. carry blood away from the heart b. filter debris from intracellular spaces c. create RBCs d. filter the blood

filter debris from intracellular spaces

The skin is inhabited by:

flora

what is technical passage?

following steps to complete something.

Nephron (kidney)

functional unit of the kidney contain a collecting tube that aids in urine production

where is bile store?

gallbladder

where is glycogen store?

in the liver and muscle tissue as a form of energy.

what is linking verb?

linking subject of a sentence to a noun or pronoun, or they link a subject with an adjective.

where is bile produce?

liver

what are polysacchrides made of?

long, repeated strands of monomers that can be linear or branched.

Triglycerides are used in the body as a for of ?

long-term energy storage in adipose tissue.

What kind of tissue is the spinal cord?

nervous

What is a complex sentence?

one independent clause and one or more dependent clauses.

A network of capillaries that drain toxins and wastes away from the body tissues into the blood & plays a vital role in monitoring & removing foreign entities in the body:

open circulatory system or lymphatic system

which disease is likely to result in height loss?

osteoporosis

Which organs is responsible for secretion of the hormone insulin?

pancreas

what are the functions of the occipital lobe?

perception, vision

which type of molecule is water comprised of:

polar

What is progesterone?

pregnancy changes such as uterine growth, attachment of embryo to uterine wall, formation of placenta, holding foetus on uterine wall, and growth of secretary alveoli in mammary glands.

function of progesterone:

prepares the uterus to receive a fertilized egg.

Which of the following is the purpose of valves in the venous circulatory system? a. facilitating stasis of blood in the veins b. propelling blood through the vena cava c. preventing backward flow of deoxygenated blood d. promoting movement of oxygenated blood toward the heart.

preventing backward flow of deoxygenated blood

plants store energy in the form of:

starch, formed from the glucose polymers amylose and amylopectin, which can be broken down into glucose during the digestive process.

Cardiac muscles exhibit which of these features? striated, involuntary, intercalated discs present striated, involuntary, intercalated discs not present striated, voluntary, intercalated discs present unstriated, involuntary, intercalated discs present

striated, involuntary, intercalated discs present

The shape of an enzyme is specifically suited to bond to one type of?

substrate at the enzyme's active site

What is the function of long bones?

support the weight of the body and facilitate movement.

which of the following mechanisms does the body use to increase heat loss? a. sweating b. vasoconstriction c. shivering d. panting

sweating

which of the following nouns represents the correct plural form of the word syllabus? a. syllabus b. syllaba c. syllabi d. syllabis

syllabi

Which part of the autonomic nervous system is responsible for accelerating heart rate?

sympathetic nervous system

which part of the autonomic nervous system is responsible for accelerating heart rate?

sympathetic nervous system

what is a narrative passage?

telling a story and entertain

where are gametes produced? a. testes, ovaries b. uterus c. nodes of ranvier d. egg yolk sack

testes, ovaries

Which layer of the skin contains hair follicles, sweat glands, and nerves? the epidermis the subcutaneous layer the basal layer the dermis

the dermis

What is adrenaline:

the fight or flight activated by the somatic nervous system from the adrenal medulla

Deoxyribonucleic acid (DNA) is found in chromosomes and stores what?

the genetic information of an organism.

Tiny muscles attached to the hair follicles relax so that:

the hairs lie flat against the surface. this increases air flow next to the skin and aids in radiative and evaporative heat loss.

What does Mendel's Law of Segregation describes?

the separation of the alleles of the parent genotype in the formation of gametes.

Example of Mendel's Law of Independent Assortment:

the sorting of alleles for eye color is not affected by the sorting of alleles for earlobe attachment.

what is a theme?

the subject to talk, a piece of writing, a person's thoughts

The integumenary system plays a vital role in:

thermal homeostasis, or regulating eh body's internal temperature, by controlling how the body interact with the surrounding environment.

what is the main function of the skin? a. nutrient absorption from the air and sun b. thermoregulation/homeostasis c. protect our inner organs d. to promote hair and nail growth

thermoregulation/homeostasis

plural third person pronoun?

they, them, their, theirs

what is the function of the progesterone?

to prepare the endometrium for the reception and development of the fertilized ovum. it also suppresses the production of estrogen after ovulation has occurred.

what is the function of neurons?

to process and transmit information.

What is the function of myelin sheath?

to protect the neuron, provide insulation, and increase the speed of impulse transmission.

Amber was distraught when she discovered that her wallet was missing form her purse.What is the meaning of "distraught" as used in the sentence above?

upset

Which of the following nitrogenous bases is found in ribonucleic acid (RNA) but not in deoxyribonucleic acid (DNA)? a. Guanine b. adenine c. uracil d. thymine

uracil

what is a main idea?

usually in the first sentence, overall idea of the paragraph.

what is an example of active immunity?

vaccination

Which is the women's outer organ?

vagina

Transitive verb:

verb whose action points to a receiver.

Larynx (voice box)

vocal cords used to generate sound

A change in the gravitational field affects which property of a body? temperature weight volume mass

weight

What is a chemical equation?

when chemical reactions are written out

Example of double replacement:

when silver nitrate and potassium combine, they form silver chloride and potassium nitrate.

Goose bump:

when the contracted muscles can be seen as bumps beneath the skin.

putting pen to paper or fingers to keyboard is considered ___ step in writing process

writing

Which of the following correctly represents the phrase "five less than a number is greater than twenty"?

x-5>20

solve for: 3x=10

x=10/3

A vertical meter stick casts a shadow of 0.4 meters. If a tree casts a shadow of 12 meters at the same time, how tall is the tree?

x=30 meters

plural second person pronoun?

you, your, yours

Examples of prepositions:

-The napkin is (in) the drawer. -The Earth rotates (around) the Sun. -The needle is (beneath) the haystack. -Can you find me (among) the words.

Solve for: x+7=-3X

-x+-3x=-4x 7=-4x -7/4=x

solve the following problem: (12-2)/(11-6)

2 How? 12-2= 10 11-6= 5 10/5=2

Simplify the expression (x+3)(2x^2-3x+4)

2x^3 + 3x^2 - 5x+12

A survey was given to a random sample of people in a town

3000

A width of a rectangle is 4 inches and the area of a rectangle is 32 in ^2. Which of the following represents the length of the rectangle?

8 in

Simplify the expression above, which one is correct? (2x + 5)(4x-7)

8x^2 + 6 - 35

What causes osteoporosis?

A decline of osteoblast activity while osteoclast activity continues at expected levels

Did you know that just 10% of the population is left-handed?... Which of the following details from the passage supports the main idea?

A growing number of experts believe handedness is random.

Which of the following is a logical conclusion based on the passage?

A hummingbird may attack other birds that come near a hummingbird feeder.

A scientist predicts that germinating beans will use more oxygen than non-germinating beans. This prediction is referred to as which of the following?

A hypothesis

Stimulus question 5 of 6 which of the following definitions matches the meaning of the word "threshold' as it is used in this passage? (Steven walked through the graveyard)

A level in which one starts to feel or react to something

What is a compound?

A substance composed of molecules containing atoms of different elements.

Which of the following is correct regarding PH scale

A substance with a ph of 3 is 10 times more acidic as a substance with a ph of 4

In a field of pure white asters, a grower noticed a single blue aster. How could the grower test whether this change affected the gamete-producing cells of the plant? a. self-pollinate the flower, and plant the resulting seeds b. examine the blue petals under an electron microscope c. preform a chemical analysis to find the nature of the blue pigment d. preform vegetative propagation on a cutting of the plant

A. Self pollinate the flowers, and plant the resulting seed

10. Because of all your running around, their clients are confused about whose in charge, and the company is loosing its competitive edge. Which of the following contractions should be used to correct the error in the sentence above? A.Who's B.You're C.They're D.It's

A. Who's

Which of the following parts of a compound microscope is used to adjust the amount of light? a. diaphragm b. objective lens c. eyepiece d. coarse adjustment knob

A: The diaphragm adjusts the amount of light in a microscope.

The alimentary canal is associated with the? a. spinal cord b. digestive system c. urinary tract d. birth canal

B: The alimentary canal begins at the mouth and ends at the anus.

which part of the brain is responsible for speech?

Broca area

Which of the following sentence correctly follow the rules of punctuation? A. The peach's are ready for harvest. B. The peaches' are ready for harvest . C. My son's peaches are ready for harvest. D. My sons's peaches's are ready for harvest

C

Ozone is an allotrope of? a. hydrogen b. water c. oxygen d. uranium

C: Ozone (O3) is an allotrope of oxygen. It is a molecule made up of more than two atoms of the same element.

As light passes obliquely from air to water, it is bent. This bending is called? a. diffraction b. reflection c. refraction d. dispersion

C: Refraction is the bending of light as it passes obliquely from one medium to another.

Which of the following elements is a nonmetal? a. Na (sodium) b. Hg (mercury) c. S (sulfur) d. Mn (manganese)

C: Sulfur is the only element listed that is not a metal.

Which is the following is the primary function of red blood cells?

Carrying oxygen to other body cells

The deposition of a liquid or a solid from its vapor is?

Condensation

Which bonds are involved in S2O versus H2O?

Covalent bond; H2O has a stronger intermolecular bond

Which of the following sentence orders best forms an organized, logical paragraph? 1. Planning for a test involves determining the time needed for studying, reviewing notes and previous assignments, and prioritizing the content matter to be studied. 2. Students should determine the format of questions on the test and practice answering those types of questions. 3. Each student should understand his or her learning style and utilize study aids that reflect personal strengths. 4. Planning and practicing may not be fun, but they can build confidence and lead to test-taking success. 5. Test-taking success can be greatly improved by taking the time to organize a test preparation strategy. A. 1, 4, 2, 3, 5 B. 2, 3, 5, 1, 4 C. 4, 1, 5, 2, 3 D. 5, 1, 3, 2, 4

D

Which of the following sentences is an example of first person narrative voice? A. You were simultaneously exhausted and exhilarated following the cross-country trip. B. The cross-country trip was simultaneously exhausting and exhilarating for him. C. She was simultaneously exhausted and exhilarated following the cross-country trip. D. I was simultaneously exhausted and exhilarated following the cross-country trip.

D

Mustache: Which of the following would be an appropriate title for this passage? A. "Mustaches and Cinema" B. "The History of Human Expression" C. "Mustaches and Modern Technology" D. "The History of Mustaches"

D. "The History of Mustaches"

Mustache: Which of the following statements can be inferred from the final paragraph of the passage? A. People who have mustaches tend to have more prestige. B. Mustaches are an indication of social standing. C. People in leadership roles usually have mustaches. D. Different mustaches can evoke different interpretations

D. Different mustaches can evoke different interpretations

A substance with a ph of 8 is a? a. strong acid b. weak acid c. strong base d. weak base

D: A substance with a ph between 8 and 14 is basic. The closer the substance gets to a ph of 14, the more basic it is.

What are the two types of nucleic acids?

DNA & RNA

Monthly Household Expenses Which of the following can be determined from the graphic data representation shown below?

In the households represented, people sent more for housing than for childcare and transportation combined

Which of the following can be determined from the graphic data representation shown below? MONTHLY HOUSEHOLD EXPENSE

In the households represented, people spent more for housing than for childcare and transportation combined

Autotrophic

Organisms that make their own food

D

Read the following passage from Joseph Stalin's radio broadcast shortly after Germany invaded the USSR in 1941: Non-aggression pacts are pacts of peace between two States. It was such a pact that Germany proposed to us in 1939. Could the Soviet Government have declined such a proposal? I think that not a single peace-loving state could decline a peace treaty with a neighboring state even though the latter was headed by such fiends and cannibals as Hitler and Ribbertrop. Which argument is most clearly developed by the rhetoric used in the passage? Choice Feedback A. Russia should have joined forces with the Allies. B. Russia signed a peace treaty with Germany. C. Germany's army is not invincible. D.. Germany's leaders cannot be trusted

PWSAFMIX

SFFMOC

The left hemisphere is responsible for?

Speech and lies in part of the brain Broca's

Example of compound sentence:

The time has come, and we are ready. I woke up at dawn; then I went outside to watch the sun rise.

Tonsils

are masses of lymphatic tissue that filter interstitial fluid

Apocrine glands:

are not active until puberty and are found in the armpits, nipples, and groin.

Which of the following cell types produce antibodies? a) t- cells b) dendritic cells c) b- cells d) natural killer cells

c) b- cells

Which of the following organs stores bile? a) stomach b)liver c)small intestine d) Gallbladder

d) gallbladder

Which of the following medications is in a form other that a pill? a. DO NOT CHEW SWALLOW WHOLE b. DO NOT CRUSH c. DISSOLVE UNDER THE TONGUE OR IN THE MOUTH AS DIRECTED BY YOUR DOCTOR: DO NOT CHEW OR SWALLOW WHOLE d. SHAKE WELL BEFORE USING CLEAN MOUTHPIECE AFTER EACH USE WITH RUNNING WATER

d. SHAKE WELL BEFORE USING CLEAN MOUTHPIECE AFTER EACH USE WITH RUNNING WATER

Hydrophobic?

do not dissolve in water

Rib

flat bone

deciding thesis or the main idea and purpose of writing is in what step of writing process?

prewriting

what is solute?

substance that dissolves in a solvent in order to form a solution. Such as water.

One of the question in the Chili Recipe

to rinse/drain the beans first

An investor owns 50 shares of a stock worth $15.30 per share. Which of the following is the amount of money the investor would receive if he cashed half of the shares of this stock?

$382.5

colon is used for greeting in a formal letter, to show hours and minutes, and to separate a title and subtitle:

- Greeting in a formal letter: Dear Sir:/To whom it may concern: - time: 3:14 a.m. - title: The essay is titled " America: A Short Introduction to a Modern Country"

example of subordinating conjunctions:

- I am hungry (because) I did not eat breakfast. - He went home (when) everyone left.

Example coordinating conjunctions:

- The rock was small, (but) it was heavy. - She drove in the night, (and) he drove in the day.

The appendicular skeleton consists of:

- limb bones - scapula - clavicle - pelvis and enables movements.

What are the functions of the salivary gland?

- lubricates mouth -contain salivary amylase, which breaks down carbohydrates and starches. - contain salivary lipase, which breaks down fats.

myoid cells:

- surrounds each seminiferous tubule - may squeeze sperm and testicular fluids out of testes.

What are the functions of the skin?

-Protects from injuries - Barriers to regulate what comes in & out -Regulates body temperature. -Synthesizes & stores vitamins -Sensory functions

What is autonomic sympathetic system responsible for?

-Regulating involuntary body functions such as: heartbeat, blood flow, breathing, and digestion. -Fight or Flight response

what does the axial skeleton consists of?

-Skull -Vertebrae -Rib cage -Hyoid bone -and provides the general scaffold of the body.

What is the function of the neuron?

-Transmit information -Created electrical impulses

During hypothermia the muscles of he hair follicles contract to:

-lift the hair upright, trapping air against the surface of the skin and helping to insulate the body.

Autonomic sympathetic system is responsible for what?

-regulating involuntary body functions such as heartbeat, blood flow, breathing, and digestion. - Fight or Flight response.

What is the seminiferous tubules responsible for?

producing sperm cells

What is the first stage of mitosis?

prophase

immune system

protects the body from pathogens (infectious agents) using a combination of white blood cells and antibodies

Which of the following produces ammonia by deamination in the liver? a. proteins b. carbohydrates c. nucleic acids d. lipids

proteins

what does the enzyme protease break down?

proteins into amino acids

Enzymes

proteins that act as biological catalysts

Ribonucleic acid (RNA) translates the DNA into a form that can be read to create what?

proteins; has a sugar back hone of ribose; and has nucleotides that contain bases adenine, guanine, cytosine, or uracil (A,G,C,U).

Which circuit contains the blood vessels that carry blood to and from lungs:

pulmonary circuit.

heart

pumps blood

aldosterone hormones:

regulates sodium content in the blood.

what is altar?

religious worship occurs such as sacrificing.

Center of the kidney where urine collects before moving to the uterus is:

renal pelvis

Which of these substances is an end product of protein digestion? a. amino acids b. lipid c. fatty acids d. cholesterol

A: Proteins are made up of chains of amino acids.

A scientist is interested in studying how caffeine affects heart rate. He gathers a group of 100 college students and randomly assigns them to two groups. One group is given a caffeinated soda and the other is given a caffeine free soda with the same sugar content. Before starting the experiment, the researcher measures the heart rates of all participants. One hour later, he measures their heart rates again. Which of the following represents the dependent variable in this experiment? A. Caffeine B. Heart rate C. Sugar D. Age

Heart rate

Which types of cell stimulates other immune cells to attack and destroy foreign agents?

Helper T-Cells

The capillary network of the digestive tract carries blood to the liver through the?

Hepatic portal vein

which part of the brain is responsible for memory?

Hippocampus

Semicolon is use between items in a series that has internal punctuation.

I have visited New York, New York; Augusta, Maine; and Baltimore, Maryland.

Singular First person pronoun?

I, me, my, mine

An element's chemical characteristics are most strongly influenced by? a. the amount of the element b. its physical appearance c. its group in the periodic table d. its period in the periodic table

Its group in the periodic table

Which of the following digestive system structures releases amylase?

Salivary glands

29) Once ovulation occurs, the corpus luteum forms. Which of the following describes the function of the corpus luteum? a) The corpus luteum secretes progesterone to thicken the endometrium of the uterus b) The corpus luteum secretes estrogen to widen fallopian tubes c) The corpus luteum secretes estrogen to for the secondary oocyte d) The corpus luteum secretes progesterone to increase the chances of fertilization of the egg by the sperm

a

what is a stereotypes?

a characteristics ascribed to groups of people involving gender, race, origin, etc.

1) A scientist predicts that germinating beans will use more oxygen than non-germinating beans. This prediction is referred to as which of the following? a) An observation b) A hypothesis c) Data collection d) A conclusion

b

It has been hypothesized that chloroplasts originated from photosynthetic cyanobacteria that became symbiotic inside a host cell. Which of the following observations provides the strongest evidence for this suggestion? a) microscopically, chloroplasts appear similar to cyanobacteria b) chloroplast DNA shows clear similarities to cyanobacteria DNA c) Both chloroplasts and cyanobacteria are green and perform photosynthesis d) Photosynthesis cyanobacteria and fungi form a mutualistic association in lichen

b) chloroplast DNA shows clear similarities to cyanobacteria DNA

Which of the following is the form of energy contained in nutrient molecules and in ATP's high-energy bonds? a) mechanical energy b)potential energy c)kinetic energy d)radiation energy

b) potential energy

Which of the following of appropriate units of measure to use when finding the mass of a pencil?

gram

During meiosis, the original cell divides twice and the four resulting cells each contain a single copy of each chromosome, a _______.

haploid (23)

what are the functions of the temporal lobe?

hearing, long term memory, verbal and written recognition memory, receptive memory, music, initiation of verbal.

Helper T cells

help cytotoxic T-cells and other immune cells.

lipids contain long strands of hydrogen and carbon atoms called what?

hydrocarbon chains

What is the function of microvilli?

include lactase and other dissaccharides that break down lactose and other simple sugar . nucleases that break down nucleic acids and peptidases that complete protein digestion and convert trypsinogen to trypsin

The epidermis is the:

outermost layer of the skin. it provides a barrier to infection from environmental pathogens and regulates the amount of water lost to the body's surroundings. it is constantly being replaced with new skin cells.

what is the function of the follicle-stimulating hormone?

stimulates the development of the maturing ovarian follicle and controls ovum production in the female, and sperm production in the male.

epinephrine hormone:

stimulates the fight response in "fight or flight" response

what does bile neutralized?

stomach acid

Where is melanin located? a. stratum corneum b. stratum granulosum c. stratum lucidum d. stratum basale

stratum basale

which layer of the skin is only visible through the skin:

stratum luicidum

Cortisol and glucocorticoid the role of:

stress inflammation immunity and increase glucose.

which system carries oxygenated blood away from the left ventricle of the heart and returns deoxygenated blood to the right atrium:

systemic circuit

What does leydig cells produces?

testosterone in the presence of LH

the luteinizing hormone in the males reproductive system stimulates:

testosterone production

What does Mendel's Law of Independent Assortment suggests?

that different genes sort into different gametes, independently of each other.

what does Mendel's Law of Dominance states?

that one factor in a pair of traits dominates the other in inheritance, unless both factors in the pair are recessive.

what is a topic?

that part of a sentence about which something is said.

Which of the following is the best source to consult for information about the famous Jazz Saxophonist Charlie "Yard bird" parker's childhood in Kansas City in the 1940s?

the ''Arts'' section of 1940 issue Kansas City newspaper

present perfect:

the action started in the past and continues into the present

which of the following sentences is grammatically correct? a. the student wrote too slow to finish the exam, but he has always been a slowly writer. b. the student wrote too slow to finish the exam, but he has always been a slow writer. c. the student wrote too slowly to finish the exam, but he has always been a slowly writer. d. the student wrote too slowly to finish the exam, but he has always been a slow write.

the student wrote too slowly to finish the exam, but he has always been a slowly writer.

which of the following sentences correctly uses subject-verb-agreement? a. computers ahs made our jobs easier b. those two girls in the first row is best friends c. the dogs in our backyard growls late at night. d. the tennis balls roll into the parking lot.

the tennis balls roll into the parking lot.

Cooking with super fire charoal

the user should read the lighter fluid directions before grilling

when chyme enters the intestine, it signals the pancreas & gallbladder to release:

their digestive secretions into the duodenum.

which gland shrink with age?

thymus

Stimulus 2 of 3 which of the following is the meaning of the word "brown" as used in the directions? (Jo's slow cooked chili)

cook

Pulmonary artery carry:

deoxygenated blood to the lung

kidney

filters blood

16. Which of the following is an example of second-person narrative voice? A. You go to the window because someone is knocking insistently on the front door. B. We are planning to trip to Florida for our vacation in January. C. She won first place at the country fair with my homemade blueberry pie recipe. D. I wanted to visit the art museum to see the new sculpture exhibit.

A

Stimulus 2 of 2: Which of the following details from the passage supports the main idea?

A growing number of experts believe handedness is random

What is osteoclasts?

Absorb/ Breaks down

Which of the following relationships represents a positive correlation between two variables?

As the amount of water a plant receives increases, the growth rate of the plant increases

What are the 2 main division of the skeletal system?

Axial and Appendicular

example of glucocorticoids hormones:

Cortisol helps regulate metabolism and helps your body respond to stress

What is an example of a disease where the immune system attacks itself?

MS= multiple sclerosis

Which of the following details from the passage supports the main idea?

Many Theories exist about what determines handedness

Which of these statements is false regarding blood cells? Mature red blood cells are bi-concave discs that have a central nucleus and long life span. White blood cells are larger and less abundant than red blood cells. Mature red blood cells are oval, bi-concave discs with a lifespan of 120 days. Phagocytes and lymphocytes are the most numerous types of white blood cells.

Mature red blood cells are bi-concave discs that have a central nucleus and long life span

Which of the following substances contains an ionic bond? a. C b. N2 c. NaCL d. CO2

NaCL why? Sodium gives up electrons easily and chlorine has a strong affinity for them, these atoms form an ionic bond.

Why is water capable of maintaining relatively large amounts of table salt (NaCL) in solutions? a. Salt is lightweight b. NaCl easily separates into Na+ and Cl- ions c. Heat is created when NaCl dissolves in water D. NaCl is a nonpolar molecule when dissolved

NaCl easily separates into Na+ and Cl- ions

what kind of passage is UFO?

Narrative

20. Which of the following fractional representations is equivalent to 0.897? a. 879/1000 b. 897/1 c. 897/100 d. 897/10000

a

21. If the pediatric unit of a hospital has 165 beds, how many rooms are there if each room holds 3 beds? a. 55 b. 83 c. 162 d. 495

a

28) Which of the following best describes protection from antigens due to passive immunity? a) A mother passes antibodies to her child through breast milk b) After primary exposure to an antigen, memory T cells remain to respond to a secondary e c) Immunization leads to activation of B-Cell proliferation d) Exposure to a virus leads to the production of antibodies that can bind to the virus

a

Which of the following enzymes breaks down complex carbohydrates? a) amylase b) pepsin c)lactase d)lipase

a) amylase

What are the 3 principal layers of the skin?

epidermis, dermis & hypodermis.

sperm delivered to exterior through system of ducts:

epididymis to ductus deferens to ejaculatory duct to urethra

Which element is essential for the normal production of the hormone thyroxine? iodine magnesium calcium and vitamin D phosphorus

iodine

Vertebrae

irregular bone

When two closely related species of flour beetles, T. confusum and T. castaneum, were placed in a culture in equal numbers, T. castaneum increased in numbers while T. confusum experienced population decline. The growth difference in the changes of population most likely indicates that the energy consumption efficiency of T. castaneum is ________ T. confusum. Which of the following correctly completes the sentence above? a. less than b. less variable than c. greater than d. more variable than

greater than

what is the role of villi? a.increase depth b. decrease depth c. increase surface area absorption d. decrease surface area absorption.

increase surface area absorption

What is glucocorticoids (sugar):

increases blood sugar (adrenal cortex)

Saliva, tears, and mucus all contain the enzyme lysozyme, which degrades bacterial walls and causes them to:

lyse, or burst.

Liver

makes bile and help regulate blood sugar levels

what is persuasive passage?

making you believe something or to do something.

Which of these groups of physical quantities consists only of scalars? time, velocity, force mass, velocity, acceleration mass, speed, time speed, weight, acceleration

mass, speed, time

What does the triple beam balance measure?

measurements of weight

The innate immune system is quick to respond to incoming pathogens but is unable to form:

memory, or the ability to remember a pathogen that has been previously encountered.

Dermis

middle layer of skin contains blood capillaries,hair shaft,nails roots and sweat glands

Ricardo pours sand into water and shakes the container. The sand stays suspended in the water for a while but eventually settles at the bottom of the container. Ricardo created a: a. mixture b. substance c. solution d. solute

mixture

carbohydrate monomers care called?

monosaccharides

Dilated blood vessels in the dermis allow:

more blood to get near the surface of the skin, carrying excess heat from the body's core. this heart is then released to the external environment as thermal radiation.

Where is bicarbonate produced?

pancreas

what enzymes are found in the pancreas?

pancreatic proteases ( trypsin and chymotrypsin) which help to digest proteins. pancreatic amylase which helps to digest sugars ( carbohydrates) pancreatic lipase which helps to digest fat.

cells that produce acid in the stomach?

parietal cells

glucocorticoids hormones:

part of stress response, increase blood glucose levels and decrease immune response.

what kind of transport is gas exchange? a. kinetic b. active c. passive d. molecular

passive

Scientists rely on which of the following to provide critical feedback when revising scientific explanations?

peer review

which is a digestive system enzyme? a. pepsin b. pepsi c. hydromorphic d. bile

pepsin

amino groups are linked by what kind of bond?

peptide bond that forms with the removal of a water molecule from the carboxyl and amino groups.

what is follicle-stimulating hormone produce by?

pituitary gland during the 1st half of the menstrual cycle.

Fatty acids are consumed in the form of what?

plant oils and animal fats

Interjections:

exclamation that is used alone or as a piece to the sentence.

what is combustion?

A type of oxidation reaction

A mass of food that has been chewed and swallowed is called?

bolus

ureter

connects kidney to bladder

author's purpose:

reason to write a specific topic

The spaces between neurons are called? a. synapses b. dendrites c. inter-neurons d. cell gaps

A: An impulse travels down the axon of a neuron and reaches the terminus. There, chemicals are released in response, which travel across the synapse to the next neuron. Thus the impulse is transmitted to the next neuron.

Which of these substances forms an alkaline solution in water? a. Mg(OH)2 b. H2SO4 c. MgSO4 d. NaCl

A: Magnesium hydroxide is the only base of the four choices. Thus in water, magnesium hydroxide releases hydroxide ions, forming a basic or alkaline solution.

Stimulus 1 of 6 which of the following sentences best summarizes the text?( WHITE HOUSE SOLAR PANEL)

Diverse presidential philosphoies have effected white house energy resource use

Binary fission in paramecia and budding in yeast cells are examples of? a. asexual reproduction only b. sexual reproduction only c. spontaneous generation and sexual reproduction d. both sexual and asexual reproduction

A: Binary fission and budding are both examples of asexual reproduction, because cell division in both cases depends on mitosis.

what is exothermic?

Give off heat

What cycle is where fertilization happens?

Meiosis

Fibrous proteins do not dissolve in?

water; they are found in structural compounds like hair and nails (keratin) and connective tissue (collagen).

example of stereotypes:

◾Domestic Behaviors: Women are supposed to cook and do housework. Women are better at raising children. Stay-at-home mothers are better than working mothers. On the other hand: Men are better at household repairs. Men cannot cook, sew or care for their children. Men always tell their wives what to do.

What is the cause of emphysema?

-Chemical pollutants, -Smokes

Which of the following is a good estimate of the length of a standard bed pillow

0.5m

A student starts working for $12 per hour. After a year he gets a raise and earns $13.50 per hour. Which of the following represents the increase in the students wage?

12.5%

Consider two parents who are both heterozygous for sickle cell anemia, a recessive genetic disease. What is the probability that an offspring will have the disorder? a. 25% b. 50% c. 75% d. 100%

25%

Spinal muscular atrophy (SMA) is an autosomal recessive disorder. Knowing this information, what are the chances that a child will not have the recessive allele if both of his parents are carriers? 50% 0% 25% 100%

25%

The skin has a moderately acidic pH, between:

3&5 which discourages replication of most pathogens.

A fragment of mRNA with the sequence 5'ACGCAUGGCAAAAAAAA 3' will have a template DNA sequence that reads

3' TGCGTACCGTTTTTT 5'.

A bucket can hold 3 L. How many mL can the bucket hold?

3,000 mL

Which of the following percentages is the discount of a textbook if the original price is $107.00 and the discount is $62.10? (round to the nearest whole percentage)

45%

A glass that holds 0.5 liters (L) will hold how many milliliters (ml)

500ml

Cheetahs, rainbow trout, and inchworms belong to the same A Kingdom B Phylum C Class D Order

A. kingdom

A specific isotope has an atomic number of 51 and a mass number of 122. How many electrons are contained in the neutral atom? a. 51 b. 71 c. 122 d. 173

A: The mass number is the sum of the number of protons and neutrons. The atomic number is the number of protons, which is always equal to the number of electrons in a neural atom.

The first step by which a mutation my occur is a change in the ? a. location of a nitrogen base in DNA b. messenger RNA c. transfer RNA d. amino acid in a protein synthesis

A: The sequence of messenger RNA, transfer RNA, and amino acids in protein synthesis all depend on the sequence of DNA found in the nucleus. Thus the fist step in which a mutation may occur is at the level of the DNA, which is made up of a specific sequence of basis.

Which of the following cavities is lined by the connective tissue peritoneum? A. Cephalic B. Thoracic C. Abdominal D. Pelvic

Abdominal

Where does gas exchange occur?

Alveoli

A patient walks into a clinic with a sore leg. The patient says she is required to carry heavy objects at her job, and she believes her leg has been affected by the weight. In addition to the soreness, the patient's entire leg appears to be swollen and tender. The intake clinic nurse speculates this patient has bursitis. - Which of the following is the definition of "bursitis" in the passage above?

An inflammation of a joint

What is a decomposition reaction?

When a reactant molecule is broken down into its component parts.

sufactant

Are lipopolysaccharides that have a hydrophobic and hydrophilic layer( they keep the lungs inflated) preventing lungs from collapsing

demal papillae

Are wavy projections of the dermis into the epidermis that lock the two layers together

subclavian artery

Artery that carries oxygenated blood to the upper limbs (shoulder)

common carotid artery

Artery that supplies oxygenated blood to the face, head, and neck.

7. The job applications were shown to an anteroom, where they waited to be called for their interviews. Which of the following is the meaning of the prefix "ante-"as used in the sentence above? A.Beyond B.Before C.Against D.After

B. Before

Which of the following conclusions can the reader make from the passage? A. Metal flumes are inserted in masonry chimneys to prevent birds, such as chimney swifts, from nesting in them B. Chimney swifts can easily adapt roosting behaviors to any structure made of the proper material C. Chimney swifts are nocturnal birds whose large numbers are evident in the size of their colonies D. Moving in large chipping flocks is a natural protection against predators

B. Chimney swifts can easily adapt roosting behaviors to any structure made of the proper material

The first paragraph in the passage on chimney swifts includes this description: "...large colonies of hundreds, or even thousands, of individuals form tornado-like funnels." Why does the author include this in the passage? A. It proves the author is an expert in bird studies B. It provides the reader with an engaging image C. It establishes the passage as a work of fiction D. It offers supporting evidence for the claim

B. It provides the reader with an engaging image

Which of the following is the muscle action that results in exhalation and causes the air pressure inside the lungs to be higher that outside the chest cavity? A. Contraction of internal intercostal B. Relaxation of the diaphragm C. Contraction of the diaphragm D. Relaxation of the external intercostal

B. Relaxation of the diaphragm

In which of these structures does the normal development of a human embryo occur? a. ovary b. uterus c. vagina d. oviduct

B: The ovary releases the egg into the oviduct, or fallopian tube where it is fertilized. If fertilization occurs, the zygote travels to the uterus, where it implants itself and develops.

The tiny projections in the small intestine adapted for absorption are called? a. venules b. villi c. alveoli d. nephrons

B: Villi are tiny projections that are barely visible to the naked eye.

Consider the unbalanced equation __N2+__H2 > __NH3. What is the coefficient of ammonia in the balanced equation that has the smallest whole-number coefficient? a. 1 b. 2 c. 3 d. 4

B: When balancing an equation, the number of atoms of each element must be the same on both sides of the equation: N2+3H2 > 2NH3

A 4-N (-newton) and an 18-N force are acting in opposite directions on the same point of an object. The magnitude of their resultant is? a. 0N b. 14N c. 18N d. 22N

B: When two forces act in opposite directions, the total force is determined by subtracting the smaller force from the larger one.

Which statement regarding ball and socket joints is true? Ball and socket joints are uni-axial joints, which are highly mobile, allowing rotation around a central axis. Ball and socket joints are bi-axial joints, which are mobile, allowing movement in two planes. Ball and socket joints are multi-axial joints, which are highly mobile, allowing movement in multiple axes and planes. Ball and socket joints are uni-axial joints, which are mobile, allowing movement in a single axis and plane.

Ball and socket joints are multi-axial joints, which are highly mobile, allowing movement in multiple axes and planes.

At which of the following times does it make the most sense to mince the garlic?

Before adding to the pan

Hydrogen can be prepared in the laboratory by combining Zn and HCl. In the resulting reaction, the metallic zinc (Zn). a. is changed to another element b. reacts with the water in the acid c. replaces the combined hydrogen in the acid d. serves as a catalyst

C: In the reaction, zinc replaces hydrogen and produces zinc chloride. Thus, hydrogen gas is released. Zn+2HCl > ZnCl2+H2 This is a single replacement reaction.

Which of the following particles are nucleons? a. neutrons and electrons b. electrons, protons and neutrons c. neutrons and protons d. electrons, photons, and neutrons

C: Neutrons and protons are contained in the nucleus of an atom, whereas electrons reside outside the nucleus.

Many people believe that some trees drop their leaves in the late fall because the temperature is near freezing. Which of the following statements least supports this idea? a. Maple trees in northern Minnesota drop their leaves in late fall. b. Maple trees growing near city lights stay green longer than those far from the lights. c. Maple trees pass through a time of near freezing before they bud d. Maple trees in southern Florida drop their leaves in late fall

D: The statement supports the idea that leaves are dropped due to the temperature, because the temperature in southern Florida in late fall is not freezing, yet the leaves are dropping. The key word in the question is "least".

The numerous villi in the small intestine serve to? a. secrete enzymes for digestion b. absorb water from dissolved foods c. secrete hydrochloric acid to dissolve food d. provide greater surface for absorption

D: The villi are tiny projections in the small intestine that increase the total surface area of the intestine for absorption of digested nutrients.

Vinegar is a common antidote for the ingestion of lye. What is the chemical process underlying this treatment? a. digestion b. alkalinization c. oxidation d. neutralization

D: Vinegar, an acid, neutralizes lye, a base.

What property of water allows someone to fill a glass slightly above the rim without the water flowing over? a. specific gravity b. capillarity c. opacity d. surface tension

D: Water molecules are polar. Due to the fact that O2 draws the electrons in the molecule toward itself.

What must undergo replication before cell division can occur?

DNA

a runner is keeping track of her daily running activity. Based on the graphic below, on which day did she run the fastest

Day 9

Which skin layer contains keratin? a. epidermis b. dermis c. hypodermis d. subcutaneous

Dermis

Which layer of the skin contains follicles hairs?

Dermis layer

_which of the following processes causes most of the carbon dioxide from the blood to move to the alveoli?_

Diffusion down a concentration gradient

Estrogen stimulates :

FSH hormone of the anterior pituitary, it is typical steroid hormone. growth, maturation and functions of female secondary sex organs like uterus, Fallopian tubes and the duct system of mammary glands.

lymph nodes

Filter out pathogens from interstitial fluid (lymph)

What is Solid?

Fixed shaped Fixed Volume Not easily compressible Highly ordered particles of any states of matter due to the strong attractions between its particles. Strong forces even though the particles that make up a solid are in constant motion, typically vibrate together with little change in their relative distance form one another. solids typically have fixed shapes and volumes. Most highly ordered type of matter is Crystal.

When Lucretia spilled some gasoline into a puddle of water, she noticed that the gasoline did not mix with the water. Instead, the gasoline formed a colorful slick on top of the puddle. Which of the following statements explains what she observed? a. Gasoline has a lower boiling point than water. b. Gasoline is made up primarily of nonpolar molecules. c. Gasoline is made up of hydrocarbons with polar molecules. d. Water is sometimes called the "universal solvent."

Gasoline is made up primarily of nonpolar molecules.

Which of the following is a polysaccharide stored in the liver? a. amylose b. cellulose c. glycerol d. glycogen

Glycogen

What is Diffusion?

Goes from an area of high concentration to an area of low concentration.

The dense granules, containing reactive oxygen compounds and cytokines, in their cytoplasm and include: basophils, eosinophils, and neutrophils is:

Granulocytes

what are long bones?

Hollow, Filled with marrow, and longer than they are wide.

Which of the following composes the rings that support the trachea? A. Spongy bone B. Fibrous ligaments C. Elastic tendons D. Hyaline cartilage

Hyaline cartilage

What kind of bonds are in nucleotide in the double strand DNA?

Hydrogen bond

The thyrotrophic-releasing, growth hormone-releasing, corticotrophin-releasing, and gonadotropin-releasing hormones are released by which endocrine system glands:

Hypothalamus

Which of the following is a portion of the brain that integrates nerve signals and hormonal secretions? a. hypothalamus b. adrenal gland c. nucleus accumbens d. medulla oblongata

Hypothalamus

Which of the following sentences is the most formal in style?

I am applying to this internship to gain experience with your company.

Example sentence of subordinate conjunction:

I am hungry because I did not eat breakfast. He went home when everyone left.

example of present perfect:

I have walked to the store three times today.

semicolon are use between independent clauses linked with a transitional word:

I think that we can agree on this; however, I am not sure about my friends.

Colon is use after an independent clause to make a list:

I want to learn many languages: Spanish, French, German, and Italian.

Which of the following sentences is punctuated correctly

I was at the mall yesterday when I met some friends. "Hey what are you doing here?" I asked

How does an impurity affect the melting point and boiling point of a substance? Impurity lowers the boiling point and raises the melting point. Impurity lowers the boiling point and lowers the melting point. Impurity raises the boiling point and lowers the melting point. Impurity raises the boiling point and raises the melting point.

Impurity raises the boiling point and lowers the melting point.

D

In A Room of One's Own, Woolf describes the life of Shakespeare's hypothetical sister whose attempts at achieving the greatness of her brother leave her desperate, ostracized, and eventually suicidal. Which best states the claim that this information helps to develop? Choice Feedback A. It is proper that women behave more subtly and indirectly than men. B. Attempts by women to repeat the success of men are doomed to failure. C. Behind every great man is a great woman with the true talent. D. Throughout history, society has worked against ambitious women.

A

In A Room of One's Own, Woolf explains that many families in the late 1400s arranged marriages. She later tells a story about Shakespeare's hypothetical sister who had his genius but whose family would not allow her to develop it. Which best states the claim that this information helps to develop? Choice Feedback A. Historically, women had little control over how they lived their lives. B. Historically, women were content to be both creative and servile. C. Historically, women have always been as intellectually capable as men. D. Historically, women were happy to marry whom their parents chose.

In your garden, you noticed that the tomato plants did better on the north side of your house than the west side and you decided to figure out why. They are both planted with the same soil that provides adequate nutrients to the plant, and they are watered at the same time during the week. Over the course of a week, you begin to measure the amount of sunlight that hits each side of the house and determine that the north side gets more light because the sunlight is blocked by the house's shadow on the west side. What is the name of the factor in your observations that affected the tomato plants growth. a. The control b. The independent variable c. The dependent variable d. The conclusion

Independent variable

to determine the insurance premium of a car, an insurance company considers the following determinants: the age of the car, the model of the car, and the milage of the car which of the following is the dependant variable?

Insurance premium

When writing a paper for publication in a medical journal, which guideline should be used for citing sources? A. Only cite sources for direct quotations B. Use the citation style that seems most appropriate to the subject matter. C. Look at previous copies of the journal to see what citation style is preferred. D. Use the citation style teachers have preferred on previous papers

Look at previous copies of the journal to see what citation style is preferred.

A researcher is designing an experiment to compare a new conductor material to the most commonly used conductor in transistors. Which procedure is likely to lead to the most reliable results? A. Measure the resistance of the new conductor 10 times and compare it to the known resistance of the existing conductor. B. Measure the resistance of the new and old conductors 10 times each under the same conditions and compare the two results. C. Measure the resistance of the new conductor 50 times and compare it to the known resistance of the existing conductor. D. Measure the resistance of the new and old conductors 50 times each under the same conditions and compare the two results.

Measure the resistance of the new and old conductors 50 times each under the same conditions and compare the two results.

Which of the following tools would measure the volume of an object roughly the size of a tennis shoe most accurately? A. Measuring tape B. Electronic balance C. Triple=beam balance D. Volumetric pipette

Measuring tape

Triglycerides also cushion what?

Nerves as an important component fo the myelin sheath and insulate the body.

Which of the following resources would help a reader gather information about economic trade during world war 1?

New articles and textbooks

What is Liquid?

No fixed shaped Fixed volume Not easily compressible Forces between the particles are strong enough to keep them together bet weak enough to allow the particles to move around one another fluidly. Liquids change shape to fit any container or surface. Liquid remains fairly constant, this state of matter has a defined volume.

What is gas?

No fixed shaped No fixed volume Easily compressible Particles move apart easily and disperse to fill an available space, gases do not have a fixed volume. The volume of a gas at any given time depends on the amount of pressure that is being exerted on its particles to keep them together. Gases can e easily compressed or compacted, while solid and liquid can not.

Stimulus question 1 of 3 which of the following words most closely matches the meaning of the word "infamous" as used in the passage? (Henry Tudor)

Notorious

Which of the following is sentences in the passage is a fact

Of all the candidates, only one was appropriate for the job

Epidermis

Outer layer of skin contains living and dead keratinocytes,melanocytes,as well as dendritic and tactile cells

Casino Passage is what kind of passage?

Persuasive why? it's trying to explain how it is better to go to a Casino then buying a lottery ticket. Buying a lottery ticket, will lower your chances of winning than going to a Casino and play.

Zip codes were introduced... A student reads the above passage in an encyclopedia and wants to learn more about the US postal service. In which of the following magazine articles would the student most likely find relevant information about Zip codes?

Postal Code Zones of the U.S Postal Service"

Zip Code

Postal Code Zones of the U.S. Postal Service

Zip codes were introduced by the U.S postal service July 1

Postal code zones of the U.S. postal service"

Which of the following inferences can the reader draw according to the passage?

Preparing for a job interview can compensate for appropriate business attire

Which of the following inferences can be made from the following section from this passage? Carter was ahead of the times. He set this example for the American people as part of a campaign to conserve energy

President Carter's conservation efforts significantly reduced waste in America

B

Read the following passage: I would like to honor the courageous soldiers who sacrificed their lives to ensure that all nations may benefit from our country's ideals. Without their sacrifice, our great nation would not have been able to bring freedom and justice to these savage lands. Which phrase from the passage most clearly shows an imperialistic attitude? Choice Feedback A. courageous soldiers B. savage lands C. freedom and justice D. their sacrifice

Assuming each macromolecule is of equal weight, which of the following macromolecules would have the greatest number of C-H bonds? a. polypeptide b. saturated fatty acid c. polysaccharide d. unsaturated fatty acid

Saturated fatty acid why? both saturated and unsaturated fatty acids contain hydrocarbon chains, but saturated fatty acids contain the maximum amount of hydrogen atoms.

Which of the following describes the likely behavior of a person who consumes two alcoholic drinks, resulting in a BAC of .08 A. She will experience no changes in her behavior. B. She will have some difficulty seeing or hearing, even though she might be in a good mood. C. She will have some difficulty with walking, although she will think she is fine. D. She will appear confused about her surroundings and her ability to function.

She will have some difficulty seeing or hearing, even though she might be in a good mood.

Epiglottis

Shuts off the tracheal opening,diverting food into esophagus

What method does the lungs use to diffuse oxygen and carbon dioxide?

Simple diffusion against concentration gradient

Which of the following statements rephrases the author's main argument about sleepwalking in the passage?

Sleepwalking is a perplexing mystery that can be alarming to those who have the condition.

Based on the advertisement above, which of the following attorneys provides the nesccessary experience to pursue her claim and would ensure the employee the lowest overall cost.

Smith & Jones

Which of the following is a logical conclusion based on this passage?

Solar energy is the best alternative fuel source available

Stimulus question 6 of 6 which of the following is a logical conclusion one could make based on the passage (Steven walked through the graveyard)

Steven becomes frightened by the atmosphere of the graveyard at night

Graveyard Walls Which of the following is a logical conclusion one could make based on the passage?

Steven becomes most frightened by the atmosphere of the graveyard at night.

Graveyard Walls Which of the following options most accurately summarizes the passage?

Steven sneaked into the graveyard but immediately regretted his decision to do so. The sights and sounds of the graveyard at night were unfamiliar and frightening. The graveyard groundskeeper unexpectedly appeared and then walked away. Steven left the graveyard, making the decision to lose the wager with his friend.

stimulus question 1 of 6 which of the following options most accurately summarizes. ( Steven walked through the graveyard)

Steven went to the graveyard after school because he had bet the town bully $5 that he would stay there for an hour. The ominous sights anf sounds in the graveyard indicated that it was haunted, and Steven was very frightened. Just when he was most scared , he saw a mysterious figure that could have been a walking corpse

What is a solvent?

Substance with dissolving capability.

which of the following is the contraction phase of the cardiac cycle?

Systole

Which part of the skeletal system runs along the body's midline axis and is made up of 80 bones in the following regions: skull, hyoid, auditory ossicles, ribs, sternum, and vertebral column

The Axial skeleton

. Chimney Swifts Based on this passage, which of the following statements is true?

The author provides facts to support the main idea.

Which reaction occurs during thermoregulation to increase body temperature?

The body to shivered

Capital City Chargers Points Per Game Graph 1995 Ross-2015 Which of the following statements is true about the graph above?

The chargers had the greatest increase of points per game between 2013-2014 and 2014-2015 scores

Once ovulation occurs, the corpus luteum forms.Which of the following describes the function of the corpus luteum?

The corpus luteum increases progesterone to increase chances of fertilization of the egg by sperm.

One way of classifying types of clouds is by the way in which they are formed... Which of the following is the main idea of the paragraph?

The correct categorization of clouds

one way of classifying types of clouds is by the way in which they are formed. Two categories are cumulus and stratus. Cumulus clouds are formed when areas of moist air are warmed and raise quickly. Cumulus clouds are buffy or billowy and resemble piles of cotton. Stratus clouds are formed when warm moist air rises over cold air. Stratus clouds resemble a blanket spread over the sky/ - Which of the following is the main idea of the paragraph?

The formation of clouds

What does the hypothalamus controls?

The hormones secreted by the pituitary gland.

which of the following body systems is the primary system involved in preventing an infection when a cut in the skin allows bacteria to enter the body?

The immune system

Which section of the digestive system is responsible for water reabsorption? a. the large intestine b. the duodenum c. the small intestine d. the gallbladder

The large intestine

Which of the following parts of the graph below helps the reader interpret the information the arthur wants to convey? CITY PIBLIC SCHOOL

The title, which identifies the subject under consideration

Which sentence uses formal language? A. This sandwich is really delicious. B. I wish I could stay up and talk, but i'm beat. C. Let's catch that movie when it comes to town. D. He seems like a really cool teacher.

This sandwich is really delicious.

Martin and Sons: job injuries, Medical Malpractice and Personal Injury... An employee believes she was injured on the job due to an unsafe situation. Based on the advertisements above, which of the following attorneys provides the necessary experience to pursue her claim and would ensure the employee the lowest overall cost?

Thorn and Thomas

Editorial (Fremont has struggled over recent years...) Which of the following is thepurpose of the editorial about downtown? Fremont

To engage residents in the discussion about increasing business revenue.

What are the pores in the nuclear envelope used for?

Transport molecules, regulate import and export of material between nucleus and cytoplasm.

Color blindness is a hereditary condition present at birth. The human eye normally has three types of cone cells, and each type senses either red, green, or blue light. Inherited color blindness happens when an individual lacks or has a malfunction in one of these cone cell types. He or she might not see one of the basic colors, see a different shade of that color, or even see a different color entirely. Thus, faulty red cones means that an individual will be unable to see colors containing red clearly. A person with malfunctioning blue-sensing cone cells would most likely have difficulty distinguishing which of these color? a. pink b. turquoise c. brown d. olive

Turquoise

Which of the following is a response to a release in antidiuretic releasing hormone (ADH)?

Water reabsorptions increases at the collecting duct

Which sentence is correctly punctuated?

We plan to take some trips this summer to Wellfleet, Massachusetts and Newport, Rhode Island.

Which of the following sentences is written correctly?

Wearing a warm coat, I walked to the store.

Which of the following features of an almanac would be most useful to a person trying to decide whether to throw a Fourth of July beach party next summer?

Weather forecasts

B

Which aspect of Mary Shelley's Frankenstein most clearly shows a Gothic influence in the story? A. The novel begins with a series of letters written by a British explorer of the Arctic. B. Frankenstein suffers mental torture from his guilt while his monster tortures others for vengeance. C. The monster demands a life of peaceful existence in South America with a companion. D. Justine is wrongfully accused and convicted for the death of another character, William.

A

Which sentence most accurately evaluates the spoken tone of Churchill's "Their Finest Hour"? Choice Feedback A. Churchill's tone comes across as unmotivated and is not likely to inspire the audience. B. Churchill's cautious approach lends credibility to the idea that Britain will be victorious. C. Churchill's emphasis on carrying on the war challenges the British to fight bravely. D. Churchill's surprised tone makes him seem as if he is caught off-guard.

A

Which sentence most accurately evaluates the spoken tone of Churchill's "Their Finest Hour"? Choice Feedback A. Churchill's tone sounds depressed despite his words calling for determined duty. B. Churchill's emphasis on Britain's inflexible resolve makes victory seem like a foregone conclusion. C. Churchill's sarcasm makes it appear that he's daring Germany to attack. D. Churchill's bold tone encourages everyone to stand up and be counted.

Which of the following chemical equations describes a reaction that has a metal and an acid as the reactants and a salt and a gas as the products? a. Zn+2 HCL---- ZnCL2+H2 b. ZnCL2 + H2 ----- Zn+2 HCL c. 2 AL(NO3)3 + 3 MgS ----- 3 Mg(NO3)2 + AL2S3 d. Li + NaCL ----- Na + LiCL

Zn+2 HCL---- ZnCL2+H2 Why? Zinc is a metal and HCL is an acid

common prepositions:

about, after, against, among, around, at, before, beneath, between, beyond, by, down, during, for from, in, of, off, on, over, past, through, to, toward, under, until, up, with, within,without

Which of the following options uses parentheses correctly? a. As the horses broke from the gate, the betting favorite stumbled slightly, eliciting gasp (and likely some curse words) from the crowd. b. The betting favorite made a strong push down the stretch, but the lead was too much to overcome (even for such a strong horse) c. The announcer (cleared his throat and) prepared to call the race from the historic press box at Churchill Downs. d. When the leaders reached the midway point (that several experts tagged) as a good lone-shot bet began to pull away from the pack

a

Which of the following processes explains why an individual that has suffered a broken bone and is immobilized for an extended period of time experiences bone mass loss? a) demineralization b)muscle atrophy c) ossification d) kyphosis

a) demineralization

A first degree burn involves which of the following layers of skin? a) epidermis only b) epidermis and part of the dermis c) epidermis and all of the dermis d) epidermis, dermis, and some muscle and bone

a) epidermis only

where are sebaceous found:

all over the body with the exception of the soles of the feet and palms of the hand.

35) Which of the following occurs when the thyroid gland releases calcitonin? a) Osteoclast activity is enabled b) Building of the bones occurs c) The bones break down d) The bones release calcium

b

HCl + Mg(OH)2 --> MgCl2 + H2O Balance the equation above. Which of the following describes the number of moles of each product that are formed by this chemical reaction after is has been balanced? a) one mole magnesium chloride, one mole water b) one mole magnesium chloride, two moles water c) two moles magnesium chloride, one mole water d) two moles magnesium chloride, two moles water

b) one mole magnesium chloride, two moles water

Seaweed --> Snail--> Fish--> Dolphin Using the food chain above, which of the following terms best describes the trophic level of snails? A. Producer B. Primary consumer C. Secondary consumer D. Tertiary consumer

b) primary consumer

Anxiety about an upcoming final exam causes a student to experience an increase in blood pressure, dilation of the pupils, and a dry mouth. Which of the following nervous system responses is responsible for these physical symptoms? A. Voluntary B. Sympathetic C. Limbic D. Cognitive

b) sympathetic

Which of the following statements from the passage provides evidence that hummingbirds have the "ability to amaze"? a. "Despite their diminutive stature, these birds can do many incredible things." b. "Their wings beat faster than those of any other bird - from 60 to 70 times per second." c. "In the eastern United States, the Ruby-throated Hummingbird is the only kind commonly found north of Florida." d. "Many people hang hummingbird feeders containing sugar water, and hummingbirds flock to them."

b. "Their wings beat faster than those of any other bird - from 60 to 70 times per second."

Students are offered a $ discount on all movies that start before 5:00 p.m. How many different movies could a student see at the discount price? a. 4 b. 5 c. 6 d. 12

b. 5

Which of the following is the atomic mass of an atom containing 42 protons, 42 electrons, and 37 neutrons? A 42 B 79 C 84 D 121

b. 79

Read the memo below before answering the question.​ TO: All Staff FROM: Human Resources RE: Award Nominations As the year comes to a close, please consider nominating a coworker for the employee-of-the-year award. This award is sponsored by the company and is intended to recognize an employee who has exhibited consistent excellent performance throughout the year. Please keep the intended purpose in mind when considering a nominee. Once all nominations have been received, a committee of managers and employees will make the final selection. Please submit your nominations no later than Friday, December 5th. What is a logical conclusion based on the information given in the memo? a. Most of the employees will be nominated for the award. b. An employee will be selected from the nominee pool shortly after December 5th. c. The company will continue this award for years to come. d. The person who wins Employee of the Year will immediately receive a promotion.

b. An employee will be selected from the nominee pool shortly after December 5th.

Which taxonomic rank is the next-smallest after Order? A Phylum B Family C Species D Genus

b. Family

Which of the following best rephrases the author's point of view about media coverage of sleepwalking? a. Media coverage of sleepwalking is confusing. b. Media coverage of sleepwalking is sensationalized. c. Media coverage of sleepwalking is mundane. d. Media coverage of sleepwalking is inaccurate.

b. Media coverage of sleepwalking is sensationalized.

There are many tall tales from old world sailors and farmers, but some of them about weather predictions are true. Here are three general rules you can follow to predict precipitation. If there is a ring around the moon at night, rain or snow may be coming. Rain or snow may be imminent when puffy cumulus clouds change and begin to flatten. And finally, if a north wind shifts in a counter-clockwise manner, rain or snow could be on its way. What is the main idea of the paragraph? a. Cloud patterns indicate a change in the weather. b. Precipitation can often be forecasted by a few simple observations. c. Old world sailors predicted weather. d. The night sky often gives clues about imminent precipitation.

b. Precipitation can often be forecasted by a few simple observations.

Which of the following is the main idea of the third paragraph? a. The tendency to sleepwalk runs in families b. Sleepwalk is common phenomenon c. Authorities keep accurate statistics about the occurrence of sleepwalking d. The women's sleepwalking was the most amazing incident that day

b. Sleepwalk is common phenomenon

Which of the following describes the structure of the passage? a. This passage tells a story about hummingbirds. b. This passage provides facts about hummingbirds. c. This passage describes how to care of hummingbirds. d. This passage argues for the protection of hummingbirds.

b. This passage provides facts about hummingbirds.

Which is the following is the author's main purpose in the third paragraph? a. To entertain b. To inform c. To persuade d. To analyze

b. To inform

A scientist found that when an invasive species was introduced into a river, the population of salmon native to the region decreased. The type of correlation between these two populations could best be described as A a direct relationship. B an inverse relationship. C a direct and indirect relationship. D no decisive relationship.

b. an inverse relationship

In a lab experiment designed to test the rate at which plants grow under artificial light, the kind of light used is which type of variable? A dependent B independent C random D responding

b. independent

Which of the following subatomic particles are found inside of an atom's nucleus? A electrons, neutrons, protons B neutrons, protons C electrons, protons D electrons

b. neutrons, protons

What characteristic of an element determines its specific isotope? A Number of protons. B Number of neutrons. C Number of electrons. D Number of quarks.

b. the number of neutrons

19) A scientist has noted a possible relationship between a certain chemical substances found in fish and the occurence of kidney failure in humans. Which of the following preliminary steps should the scientist take before conducting a controlled experiment? a) Establish a cause and effect relationship between the substance and the outcome b) Distribute letters of consent to human subjects who will be used in the research c) Review the literature to determine what other research is available on this topic d) Write a research grant proposal to the appropriate federal funding agency

c

24) Which of the following chemicals is released by one type of immune cell to directly activate another type of a) Perforin b) Granzymes c) Cytokines d) Lysozymes

c

38) (Another test?) If damage occurs to the alveoli of the lungs, which of the following is true a) Blood would not be sent from the heart to the body tissues b) Blood would not contain carbon dioxide c) Blood would now be oxygenated d) Blood would not return from the lungs to the heart

c

39) In a hypertonic solution, water flows through aquaporins embedded in the plasma membrane of the cell. This type of transport is best known as which of the following? a) Osmosis b) Active transport c) Facilitated diffusion d) Diffusion

c

If the amount of hydrogen in a balloon of fixed size is doubled, what happens to the mass and density of the balloon? A The mass and density both decrease. B The mass increases, but the density decreases. C The mass decreases, but the density increases. D The mass and density both increase.

d. the mass and density both increase

Which of the following sentences is the most formal in style? a. I'm applying to be an intern to gain experience with your company b. I am applying to this internship to gain experience with your company c. I'd like to apply for this internship in order to gain experience as a part of your company d. I want to gain experience with your company, so I am applying to be an intern

c

Which of the following is the iron- containing protein in red blood cells ? a)plasma b) deoxyribonucleic acid c) hemoglobin d) myoglobin

c) hemoglobin

Which of these terms specifically means the intake and expulsion of air using the lungs? a) inspiration b) aeration c) Ventilation d) Oxygenation

c) ventilation

A blood pressure reading of 120/70 mm HG refers to arterial pressures recorded during a) atrial contraction/ventricular contraction b) atrial contraction/ ventricular relaxation c) Ventricular contraction/ Ventricular relaxation d) Ventricular relaxation/ atrial relaxation

c) ventricular contraction/ ventricular relaxation

An unknown element is found to contain 45 protons and an atomic mass of 64, what is its atomic number? A It cannot be determined. B 19 C 45 D 64

c. 45

Which of the following data supports the author's argument about why Americans lack awareness about the dangers of malaria around the world? a. 95% of American adults in a survey believe HIV/AIDS is more dangerous worldwide than malaria. b. A financial study finds that U.S. churches donated over $10,000 last year to supply mosquito nets to impoverished countries to prevent the spread of malaria. c. An epidemiology report finds there have been no cases of malaria originating from the United States in the last 60 years. d. A World Health Organization investigation reports that over half of the world's population is in danger of malaria.

c. An epidemiology report finds there have been no cases of malaria originating from the United States in the last 60 years.

Which of the following sentences from the passage presents the topic of the passage? a. In fact, estimates for the number of sleepwalkers in the United States range into the millions. b. This woman's feat may have been the most spectacular sleepwalking performance that June night, but it was far from the only one. c. Despite the fact that sleepwalking, or noctambulation, occurs frequently, this odd ailment remains a baffling, uncanny - if not frightening - phenomenon. d. Reports have shown that sleepwalkers perform remarkable feats of memory; climb upon steep roofs; write letters; solve intricate mathematical problems; play the piano or some other musical instrument; compose music; and commit robbery, suicide, or murder.

c. Despite the fact that sleepwalking, or noctambulation, occurs frequently, this odd ailment remains a baffling, uncanny - if not frightening - phenomenon.

What are groups of cells that perform the same function called? a. molecules b. organs c. plastids d. tissues

d. tissues

Two weeks ago, I went fishing on the Kaw River with my grandfather. We found a secluded spot where the fish were abundant. After a while, several police officers appeared on the opposite shore. Grandfather hollered across and discovered that they were looking for a stolen car that had been reportedly dumped in the Kaw. I suddenly had a large tug on my line, but I could not pull the fish in without breaking my line. A police officer in scuba gear volunteered to help. After several moments, the diver surfaced and said, "You have a whopper, but he is in one of those old cars at the bottom. I would have better luck helping you, but every time I approach him, he rolls up the window!" Which of the following is the purpose of this paragraph? a. To discuss b. To inform c. To entertain d. To express feeling

c. To entertain

Read the memo below before answering the question.​ TO: All Staff FROM: Human Resources RE: Award Nominations As the year comes to a close, please consider nominating a coworker for the employee-of-the-year award. This award is sponsored by the company and is intended to recognize an employee who has exhibited consistent excellent performance throughout the year. Please keep the intended purpose in mind when considering a nominee. Once all nominations have been received, a committee of managers and employees will make the final selection. Please submit your nominations no later than Friday, December 5th. What is the main purpose of the memo? a. To introduce a company-sponsored award for employees b. To meet a specific deadline c. To request nominations of potential candidates for an annual award d. To describe the qualifications of the Employee of the Year

c. To request nominations of potential candidates for an annual award

sudoriferous glands warming the body:

cease sweating.

3) A fragment of mRNA with the sequence 5' ACGCAUGGCAAAAAAAA 3' will have a template DNA sequence that reads A. 3' UGCGUACCGUUUUUUUU 5'. B. 3' TTTTTTTTGCCATGCGT 5'. C. 3' UUUUUUUUGCCATGCGT 5'. D. 3' TGCGTACCGTTTTTTTT 5'.

d

Which of the following is an example of a tissue? a. mammal b. hamstring c. liver d. xylem

d. xylem

The hypodermis is the:

deepest layer of the skin and contains blood vessels and adipose fat tissue.

What does saturated mean?

every carbon molecule is bound to 2 hydrogens

If a cell becomes infected, it will secrete:

interferon

what is luteinizing hormone?

it is produced by the pituitary gland of the brain. it stimulates the ovaries to produce estrogen and progesterone. it triggers ovulation , and it promotes the development of the corpus luteum.

what is cellulose?

linear ploysaccharide that is found in the structural material of plaints; the human body lacks the enzyme necessary to break the bonds of cellulose, so it is not digestible.

Humerus

long bone

what is Gonadotropin-releasing hormone?

produced by a part of the brain called hypothalamus. when it circulated in the blood, it causes the release of two important hormones called follicle-stimulating hormone and luteinizing hormone.

Having been discovered, Rover looked up at his owner with puppy-dog eyes. _which group of words is misplaced in the sentence above?

puppy dog eyes

what word below is spelled correctly? a. receive b. recieve c. receve

receive

John followed the doctor's___to get more rest - Which of the following completes the sentence above?

recommendation

The region of large intestine in which feces is stored before elimination:

rectum


Kaugnay na mga set ng pag-aaral

AP Government Unit 1: Foundations of American Democracy

View Set

Fire Officer Chapter 5 Fire Department Administration

View Set

Great Britain Practice Questions

View Set

BUS101 - Module 1 Week 1 Role of Business

View Set

Biomedical Pharmacology HSAD 020 Examination #4

View Set